MJDF MCQ Collection
MJDF MCQ Collection
MJDF MCQ Collection
Rev. Dr nagham
BY DR.ABDULRAHMAN ALMUALM
MJDF MCQS WITH ANSWERS
Dr.Almualm
BY DR.ABDULRAHMAN ALMUALM
MJDF MCQS WITH ANSWERS
5 )Which is the most appropriate image to use when assessing third molars?
a) Bitewing radiograph
b) Long cone CT
c) Long cone periapical radiograph
d) Orthopantomogram (OPG)
e) PA mandible
Ans is D
7. A Female patient with history of hip fracture and osteoporosis taking oral
bisphosphonates
from about 2 years, no other medications. She want to extract a tooth:
a) Extract with no alteration but advice to keep her mouth clean
b) Stop the drug 3 days before extraction after consultation with GP
c) Consider 3 months drug holiday after consultation with GP
d) Never to extract, RCT and crown amputation
e) Extract under antibiotic cover
BY DR.ABDULRAHMAN ALMUALM
MJDF MCQS WITH ANSWERS
11) Child aged 10 with 1 mm pulp exposure of upper c;entral ,which is the preferred
treatment option
-='
a) Cvek pulpotomy
b) Direct pulp capping
c) Indirect pulp capping
d) Complete pulpotomy
A
BY DR.ABDULRAHMAN ALMUALM
MJDF MCQS WITH ANSWERS
14 ) The biting load of a denture base to the gingival tissue compared to teeth are
a) Ten times more
b) Ten times less
c) equal
Ans B
16 Patients comes to you with an ulcer on the lateral border of his tongue. He wears
a
lower full
denture and is complaining that his denture is loose and sore. How would you
manage?
a) Refer the patient as an emergency case
b) Refer the patient to the oncologist
c) Relieve the denture and ask the patient to come in for a review
d) Send the patient to the Oral Maxillofacial surgery department
e) Take biopsy
Ans is C
BY DR.ABDULRAHMAN ALMUALM
MJDF MCQS WITH ANSWERS
extraction
c) Require antibiotic prophylaxis with3g Amoxicillin provided they are
not allergic to penicillin
d) Most probably have metastatic bone disease or osteoporosis
E) Must have their bisphosphonate medication stopped 24 hours
prior to dental treatment
D is the answer
19) Which of the following has highest five year survival rate for class V restoration?
A. Amalgam
B. Bonded amalgam
C. Composite (flowable)
D. Conventional GIC
E. Resin modified GIC
Ans is E
22) A pt presents with a history of clicking from their tmj. This click occurs midway
through the
BY DR.ABDULRAHMAN ALMUALM
MJDF MCQS WITH ANSWERS
opening cycle and is consistent. There is some preauricular pain and the lateral
pterygoid muscle on
the affected side is tender to resisted movement test. There is no trismus and the
click is not present
when the patient opens from a incisor edge to edge relationship ,instead of her
normal class-1
occlusion. The patient would like treatment, the most appropriate occlusal splint for
this patient would
be
a) Stabilisation splint
b) localised occlusal interference splint
c) Bite raiser
d) Soft biteGuard
e) Anterior Repositioner Splint
ans is e
26 A study was conducted with the following objective: To determine the individual
and combined
BY DR.ABDULRAHMAN ALMUALM
MJDF MCQS WITH ANSWERS
28What is the usual range of orthodontic force for bodily tooth movements?
• A. 25-50g
• B. 50-75g
C.75-100g
D.100-150g
E.>150g
Ans is D
BY DR.ABDULRAHMAN ALMUALM
MJDF MCQS WITH ANSWERS
30) Which of the following is regarded by British Dental Association as risk factor for
oral cancer?
A Age
B Candida infection
C Chronic sepsis in mouth
D Excessive exposure to sunlight and radiation
E Immune deficiency disease
Ans is D
31) Attached gingiva is the thinnest in the mouth in the following region of the
mouth?
a) Maxillary anterior buccal region
b) Maxillary molar buccal region
c) Mandibular anterior buccal region
d) mandibular anterior lingual region
E mandibular molar lingual region
Ans is C
32 Question about AED and what you are gonna do aftrer fierst given shock..
• A) give another shock
• B)check ABCDE and start CPR
• C) recovery position
• D) give two breath
Ans is B
33 Patient had his history of epilepsy and presents to your practice with a painful
dental abscess. She is currently on Carbamazepine. Which of the following antibiotic
is contraindicated?
a. Amoxicilin
BY DR.ABDULRAHMAN ALMUALM
MJDF MCQS WITH ANSWERS
b. Erythromycin
c. Clindamycin
d. Metrodinazone
e. Tetracycline
Erythromycin interferes with liver microsomal metabolism of carbamazepine,
causing decreased carbamazepine clearance, an elevated
serum carbamazepine level, and clinical toxicity. Erythromycin-
inducedcarbamazepine toxicity usually manifests as drowsiness, lethargy, ataxia,
vomiting, and nystagmus.
39) 12.After consultation with the patient’s cardiologist regarding the risk of infective
endocarditis, antibiotic prophylaxis was advised, which of the
BY DR.ABDULRAHMAN ALMUALM
MJDF MCQS WITH ANSWERS
40A 55 year old lady has attended requesting extraction of lower right first
molar which has a questionable restorability. Her medical history reveals
that she has diabetes mellitus, post-menopausal osteoporosis, and
rheumatoid arthritis. She has been taking the following medications orally
since 2009; metformin, alendronic acid and methotrexate. What would be
the risk of having MRONJ from the extraction of this tooth?
a. High.
b. Moderate.
c. Low.
d. Negligible.
Ans is A
BY DR.ABDULRAHMAN ALMUALM
MJDF MCQS WITH ANSWERS
48 68) Patient has an over jet of above 9mm, which IOTN band it will be
a) Band 1
b) Band 2
c) Band 3
D)Band 4
e)Band 5
Ans is C band 3 |
IOTN 1 and 2 --> no claim
IOTN 3 with aesthetic component 1-4 --> no claim
IOTN 3 with aesthetic component 5-10 --> band 3
IOTN 4 and 5 --> band 3
Overjet 6-9 is IOTN 4
Both IOTN 4 and 5 are eligible for NHS orthodontic treatment under band 3
BY DR.ABDULRAHMAN ALMUALM
MJDF MCQS WITH ANSWERS
56.diabetic with abscess complain of failure of root canal therapy and during the
examination the filling was leaking? What the reason
. a) Lack of coronal seal
b) Lack of apical seal
c) Medical condition
BY DR.ABDULRAHMAN ALMUALM
MJDF MCQS WITH ANSWERS
Ans is a
BY DR.ABDULRAHMAN ALMUALM
MJDF MCQS WITH ANSWERS
BY DR.ABDULRAHMAN ALMUALM
MJDF MCQS WITH ANSWERS
c) Methotrexate
d) Systemic Corticosteroids
e) Fluoxetine
Ans is e
72. ) You have just completed conventional endodontics on UL7. The tooth had
originally been
restored with an extensive MOD amalgam and is now significantly further weakened.
What is the best
immediate restoration for this tooth?
a) Direct compositewith cusp reduction
b) Direct composite with full cuspal coverage
c) Indirect composite or porcelain onlay
d) MOD amalgam with Nayyar core capping all cusps
e) Porcelain fused to metal crown on amalgam core
Ans D
73) Which is the best cantilever bridge for missing max canine, the abutment on
a) Both premolars
B) Lateral and Central Incisors
C) Lateral incisor
D) First premolar
Ans is A
BY DR.ABDULRAHMAN ALMUALM
MJDF MCQS WITH ANSWERS
Ans is C
75. according to the recent radiation protection recommendations, The annual dose
limit for non-classified workers shouldn't exceed:
a) 20 MSV (classified workers)
b) 50 MSV
c) 6 MSV
d) 26 MSV ans is C || general public =1 msv
76. Which odontogenic pathology is associated with Afro-Caribbean men?
a) Ameloblastoma
b) Odontomes
c) Solitary bone cyst
d) Dentigerous cyst
e) Keratocyst
Ans is A
77. What percentage of the population has a missing wisdom tooth? a) 0.5%
b) 2%
c) 5%
d) 10%
e) 20%
Ans e
78. A 78-year-old woman presents for the extraction of her lower left 6. She
is deaf with a ‘cotton wool’ bone appearance on OPG. Which one of the
following is the diagnosis?
a) Paget’s disease
b) Giant cell granuloma
c) Cherubism
d) Fibrous dysplasia
e) Osteogenesis imperfecta
ans is A
79. 20. Which one of the following syndromes is associated with multiple tori?
a) Eagle’s syndrome
b) Frey’s syndrome
c) Gardner’s syndrome
d) Golin–Goltz syndrome
e) Crouzon’s syndrome
Ans is C
80.. Which one of these is not an NHS justification for implant placement?
a) Prosthetics tolerance issues
b) Trauma
c) Hypodontia
d) teaching
e) Oncology
BY DR.ABDULRAHMAN ALMUALM
MJDF MCQS WITH ANSWERS
Ans is D
83. 6. Moved buccally while pushing coronally, but frequently need a varied
rocking movement to extract:
a) Upper 1, 2, 3
b) Upper 4, 5
c) Upper 6, 7
d) Lower 4, 5
e) Lower 6, 7
ans is C
BY DR.ABDULRAHMAN ALMUALM
MJDF MCQS WITH ANSWERS
a) Ankylosis.
b) Subluxation.
c) Osteoma of condyle.
d) Recurrent dislocation.
e) TMJ pain dysfunction syndrome.
Ans is A
BY DR.ABDULRAHMAN ALMUALM
MJDF MCQS WITH ANSWERS
93. A 15 year old female patient has a 8mm overjet and bilateral impacted canines
a) The patient would be placed in the Index of Orthodontic Treatment Need DHC
grade 5
b) The patient is at an appropriate age to commence functional appliance therapy
c) The canine position is best assessed by using a lateral cephalometric radiograph
d) The canines are almost certainly buccally placed
e) Bilateral impacted canines are seen more frequently then unilateral impacted
canines
Ans is A
94. During the restoration of a deep carious cavity in UL6, to minimise the risk of
bacteria gaining access to the pulp, the dentist may
a) Carry out direct pulp capping
b) Carry out indirect pulp capping
c) Remove the caries from the floor of the cavity before the cavity wall
d) Give the patient a course of antibiotics for a week
e) Ask the patient to rinse with a fluoride mouthwash one hour prior the cavity
ppreparation
Ans is B
95. A 35 year old female patient requires replacement of 6 upper anterior crowns
with a conformative occlusal scheme. What would be the
most appropriate way to articulate study and working casts?
a) Average value articulator with incisal guidance table.
b) Fully adjustable articulator
c) Plane line articulator.
d) Semi adjustable articulator with facebow transfer and custom incisal guidance
table.
e) Semi adjustable articulator with facebow transfer.
Ans is D
96. Regarding root fractures, which of the following statements are LEAST
appropriate
a) Vertical root fractures occur most commonly in sound teeth as a result of direct
physical trauma
b) They occur more commonly in endodontically treated teeth
c) They could occur due to corrosion of metal posts
d) Wedging effects during post cementation are known to cause vertical root fracture
e) Horizontal root fractures are most commonly the results of direct physical trauma
to the dentition
Ans is A
BY DR.ABDULRAHMAN ALMUALM
MJDF MCQS WITH ANSWERS
98. Secondary research papers in which all the primary studies on a particular
subject have been hunted out and critically appraised
according to rigorous criteria are called
a) Randomised controlled trials
b) Case control studies
c) Systemic reviews and meta-analysis
d) Cross sectional surveys
e) Cohort studies
Ans is C
BY DR.ABDULRAHMAN ALMUALM
MJDF MCQS WITH ANSWERS
BY DR.ABDULRAHMAN ALMUALM
MJDF MCQS WITH ANSWERS
BY DR.ABDULRAHMAN ALMUALM
MJDF MCQS WITH ANSWERS
117. In the inferior alveolar block the needle goes through or close
to which muscles
A. Buccinator and superior constrictor
B. Medial and lateral pterygoid
C. Medial pterygoid and superior instructor
D. Temporal and lateral pterygoid
E. Temporal and medial pterygoid
Ans is A
118. A patient attends with pain of four days duration in a carious upper molar tooth.
The pain is constant and is not relieved by paracetamol. Sleep has been disturbed
by the pain. The tooth is tender to percussion and gives a positive response to Ethyl
Chloride. What is the most likely diagnosis? .,
A-Pericoronitis
B-Apical periodontitis
C-Marginal periodontitis
D-Reversible pulpitis
E-Irreversible pulpitis
Ans is E
119.Lady in 40s got TMJpain for about few months, she got divorced recently what is
the first line treatment
BY DR.ABDULRAHMAN ALMUALM
MJDF MCQS WITH ANSWERS
A.amitrptyline
B.physiotherapy
C.occlusal splint
Ans B ( first line of treatment.)
121. Q4 A 30 year old man with unknown allergy to latex goes into anaphylactic
shock whilst being treated in the dental surgery.
A. Hydrocortisone - orally
B. Chlorphenamine – intramuscularly
C. Chlorphenamine - orally
D. Epinephrine – intravenously
E. Epinephrine – intramuscularly
Ans is e
122. days after preparation and filling of a shallow class I
amalgam cavity the patient complains of pain on biting. You
would:
A.Perform vitality test.
B.Replace filling.
C.Check for premature contacts.
D.Remove all occlusal contacts from this filling
E.Tell the patient to wait 2-4 weeks, the pain will go away.
Ans is c
BY DR.ABDULRAHMAN ALMUALM
MJDF MCQS WITH ANSWERS
Ans is A
129 the pulp horn most likely to be exposed in the preparation of large cavity in
permanent molar tooth is
A. Mesio-lingual in upper first molar
B. Mesio-buccal in upper first molar
C. Disto-buccal in lower first molar
D. Mesio-lingual in lower first molar
E. Mesio-buccal in lower first molar
Ans is B
130
BY DR.ABDULRAHMAN ALMUALM
MJDF MCQS WITH ANSWERS
When preparing class III for composite restoration; which situation acid Etching
should be placed
133 .The technique of placing Gutta-Percha cones against the root canal walls
providing space for additional Gutta Percha is termed
A. Lateral Condensation
B. One major Gutta Percha point
C. Laterally above condensed
Ans is A
A. Tapering
B. Long path of insertion
C. Pin retention
Ans is B (long path of insertion means more length of abutment
135
What is the most sensible approach to treatment?A patient reports that his post
crown has fallen out. This crown had been present for many years. You note that
there appears to be a hairline vertical fracture of the root. The tooth is symptomless :
A. Replace the post crown using a resin-reinforced glass ionomer material
B. Replace the post crown using a polycarboxylate cement
C.Replace the post crown using a dentine bonding agent and a resin-reinforced
glass ionomer material
D.Replace the post crown using a resin composite luting agent
E.Arrange to extract the tooth
BY DR.ABDULRAHMAN ALMUALM
MJDF MCQS WITH ANSWERS
Ans is E
136 While attempting to cement a PFM crown, the crown has accidently
slipped into the patient’s throat. The patient has suddenly grasped his
throat with both hands and he is unable to speak. How would you manage
this situation?
a. Use McGill forceps to extract the crown.
b. Encourage the patient to cough.
c. Back slaps only.
d. Abdominal thrusts only.
e. Alternating back slaps with abdominal thrusts.
Ans is E (medical emergency : helmich manuever
Ans is D
A.False
B.True
Ans is A
139 Which of the following materials has the greatest resistance to wear in the oral
environment?
A. Compomers
B. Conventional glass ionomer cements
C. Resin composites
D. Resin modified glass ionomer cements
E. All have equal water resistance
Ans is C
140 A patient attends your clinic complaining of the appearance of his discolored
root filled upper right lateral incisor.which one of the following options is the most
appropriate?
BY DR.ABDULRAHMAN ALMUALM
MJDF MCQS WITH ANSWERS
142 The minimal labial tooth reduction for satisfactory aesthetics with porcelain fused
to metal crown is,
A. 1mm
B. The full thickness of enamel
C. 1.5 mm
D. 2.5mm
E. One third of the dentine thickness
Ans is C
144 Which should be covered by the denture base of a mandibular distal extension
RPD?
A) All undercuts areas and engage them for retention
B) Hammular notch
C) retromolar pad
D) pterygomandibular raphe
Ans is C????
.145 patient presents with a radiolucency in the posterior maxilla region. You suspect
an ameloblastoma. Which of the following feature
might support your suspicion.
a) The patient is 17 years old
b) The radiolucency is multilocular
c) There is evidence of calcification within the radiolucency
d) There is no evidence of tooth resorption
e) There is parasthesia in the region of the distribution of the infraorbital nerve
BY DR.ABDULRAHMAN ALMUALM
MJDF MCQS WITH ANSWERS
ans is B
146) Which of the following does not affect the elasticity of retentive clasp?
a) Length of the arm1
b) The cross section of the shape
C) The material used
D) The undercut area
Ans is D
147A pt on examination was found to have swollen gingiva around a crown that had
been present for
several years.The papillae were particularly enlarged, The most important feature of
the crown
responsible for this is
a) Material of the crown
b) The occlusion
c) Proximal contour
d) labial contour
e) Surface finish
ans is C
BY DR.ABDULRAHMAN ALMUALM
MJDF MCQS WITH ANSWERS
151) All the following drugs should be present in emergency drug box in dental office
except
a) Adrenalin
b) Midazolam
c) Corticosteroids
d) Salbutamol
e) Aspirin
ans is C
154. Reasonof failure canine eruption without early loss of deciduous teeth?
1. Supernumerary
2. Ectopic
3. Crypt
4. Primary failure
5. supplementary
Ans is 3
.156 30 yr-old patient attends complaining of pain from the lower left quadrant.
Clinical
examination reveals a dentition with generally good oral hygiene. There is no
significant periodontal pocketing other:.than an isolated defect in the region of the
furcation of lower left first molar which is non-mobile. The gingival tissue in thiS area
appears erythematous and slightly hyperplastic with a purulent exudate on probing.
From the list below, which is the most appropriate next step?
A-Obtain a radiograph
BY DR.ABDULRAHMAN ALMUALM
MJDF MCQS WITH ANSWERS
157 A 58 year old male presents at your surgery complaining of a sharp pain of no
more than 30 minutes duration arising from his upper left molar region. The pain is
brought on by cold stimuli but persists after-the stimulus is removed. It does not
seem to occur spontaneously. He has tried taking paracetemol and this does
temporarily stop the pain from recurring. The upper left 6' reacted to a lower current
on electronic pulp testing than the upper right 6, upper left ~ or the lower left molars.
What is the most likely cause of the patient's pain? -
A-Acute/reversible pulpitis
B-Dentine sensitivity
C-Chronic/irreversible pulpitis
D-Periapical periodontitis
E-Trigeminal neuralgia
Ans is A
BY DR.ABDULRAHMAN ALMUALM
MJDF MCQS WITH ANSWERS
A-Proportional unit
B-Modulus of elasticity
C-Stress/ stra i n
D-Ultimate tensile strength
Ans is B
163When preparing the Apical Zone, the use of the files sequentially from apex
to backwards (lower the size of instrument) - what is the best distance to
achieve good apical area preparation a 0.5mm
b.1.0mm
C. l.5mm
d 2.5mm
e. 3.5mm
Ans is C ?
164. Best material to use in the root caries and buccal caries in elderly patient.
AHybrid Composite
bAmalgam
C. Silicate cement
D. Zinc phosphate
E .RMGIC
Ans E
165 Six-year-old child with fluoridated water of 0.6ppm , what is the fluoride
supplement dose given
A.0.25 mg
B. 0.5 mg
C. 1 mg
Ans is B
A.Copy dentures
BY DR.ABDULRAHMAN ALMUALM
MJDF MCQS WITH ANSWERS
B.Hard reline
C.Soft reline Construction of new dentures
Ans is A = good polished surface, minimum occlusal wear
167. What is the least important factor to reduce radiation in dental radiographs?
A-Speed of film
B-Collimation
C-Filtration
D-Cone shape and length
E-Use of lead apron
Ans is E
168. A suddenly swollen upper lip that lasts for 48 hours or more is most likely
A.Haemangioma
B.Agioneurotic oedema
C.Mucocele
D.Cyst
Ans is B .
169. Medical condition of pt who reserved a seat in a dinner meating for his dead
wife
A-Mania
B-Anxiety
C-Depression
D-Schizophrenia
Ans is D
172. what type of cells that proliferate in later stage of her life?
A.Odontoblast
B.cemtoblast
C. rest of malazess
E.undifferentiated mesynchymal cells
Ans is E
BY DR.ABDULRAHMAN ALMUALM
MJDF MCQS WITH ANSWERS
176.A patient comes with a firm, painless swelling of lower lobe of parotid which has
grown progressively for the past year. He complains of paresthesia for the past 2
weeks.
This is most likely to be:
A-Pleomorphic adenoma
B-Carcinoma of the parotid
C-Lymphoma of parotid
Ans is B ?
BY DR.ABDULRAHMAN ALMUALM
MJDF MCQS WITH ANSWERS
180. Normal prothrombin time and elevated partial thromboblastin time is seen in
A.Factor VIII deficiancy
B.Thrombocytic pupura
C.Leukemia
D.Von Willebrand disease
Ans is A
182. A 58 year old male has been treated with radiation for carcinoma of tongue. The
patient complains of pain associated with
poor dentition. The dental management would be:
A-Immediate extraction of any poor teeth under local anaesthetic with antibiotic
coverage
B-Segmental dental clearance and closure to eliminate problems
C-No dental treatment may be due to neuronic of neoplasms
D. Clearance of poor dentition followed by hyperbaric oxygen treatment plus a
primary
closure of wounds under antibiotic
coverage
E. No extraction as radionecrosis is an important sequelae
Ans is D?
183. What effect has placing a sealant over pits and fissures on the progression of
caries?
A.Decreased new caries
B.lncreased new caries
C.Progression of existing caries
D. No effect on existing caries
Ans is A
184. 1 n a radiograph the roots of the upper teeth are too short because of:
BY DR.ABDULRAHMAN ALMUALM
MJDF MCQS WITH ANSWERS
Inadequate horizontal
angulation
A-Too high vertical
angulation
B-Too small vertical
angulation
C-Parallel
Ans is A
185. What is CORRECT in regard to the periodontal surface area in maxillary teeth:
A.central incisor> first premolar> second
premolar
B. Canine> first premolar> central incisor
C.Canine> lateral incisor> second premolar
D. Canine> central incisor> first premola
Ans is B
Ans is C
187. patient attends with pain of four days duration in a carious upper molar tooth.
The
pain is constant and is not relieved by paracetemol. Sleep has been disturbed by the
pain.
The tooth is tender to percussion and gives a positive response to Ethyl Chloride.
What is
the most likely diagnosis?
A-Pericoronitis
B-Apical periodontitis
C-Marginal periodontitis
D-Reversible pulpitis
Ans is B
BY DR.ABDULRAHMAN ALMUALM
MJDF MCQS WITH ANSWERS
188. A 14 year old patient attends with a decayed and hypoplastic LL7. A radiograph
shows
the presence of an unerupted LL8 and the LL6 is sound. What would be the most
appropriate long-term treatment for this tooth?
A-Amalgam restoration
B-Antibiotics
D- Root canal therapy (best is to extract)
E-Sedative dressing
Ans is D since its hypoplastic decayed for longterm treatment best is to extract and
allow 8 to enter to the arch If applicable
189. EDTA (ethylene diamine tetra-acetate) has useful roles in certain situations in
clinical
dentistry. When would you use EDTA?
A-As a pulp capping agent
B-As a root canal irrigant
C-As a mouthwash
D-As a root end filling material
E-As a dentine bonding agent
Ans is B
190. You suspect that there is occlusal caries in the lower right first permanent molar
of a
10 year old child. You wish to confirm your suspicions. Which diagnostic test is most
commonly used in this situation?
A-Bitewing radiography
B-Electro-conductive caries monitors
C-Fibro-optic transillumination
D-Panoramic radiography
E-Visual examination of a dried tooth.
Ans is E// If proximal caries C or A
191. Which one of this restorative method will be LEAST compromised by a core,
A-Amalgam
B-Composite
C-GIC
BY DR.ABDULRAHMAN ALMUALM
MJDF MCQS WITH ANSWERS
D-Cast gold
Ans is D
192.child comes to your clinic with a fractured crown 3mm supra gingival how would
you treat this case
A. Formecorsol pulpotomy
b.calcuim hydroxide pulpotomy
C.pulpectomy
D. Direct pulp capping
E.indirect pulp capping
Ans is B
So its trauma and we can go with pulpotomy (child and trauma )
Formecresol isn't recommended due its carcinogenic concern .
The Caoh is good choice here
But I will give a recap for all materials used in pulpotomy
.ferric sulphate is best recommended by most
Formercrseol still used but recent concerns of mutagencity due to formalin
Caoh is good but time consuming but ferric sulphate is better
MTA show great success rate but expensive
Divitalizing paste (formaldehyde also containing and concern for use )
194. patient in your dental chair shows chest pain, weak pulse and dysponea, what
is
your initial management,
A-Give a nitro-glycerine tablet and keep the patient up seated'
B-Put the patient in supine position
BY DR.ABDULRAHMAN ALMUALM
MJDF MCQS WITH ANSWERS
195. Developer contaminated with other chemical and was not mixed pro perlv. What
is
the effect on the X-ray film?
A-Too dark film
B-Light film
C-Foggy
Ans is C
BY DR.ABDULRAHMAN ALMUALM
MJDF MCQS WITH ANSWERS
Ans is B
200. Immediately after the extraction of lower molar the patient complains of post
operation bleeding and pain, how would
manage this,
A-Prescribe analgesics and ask the patient to follow a strict oral hygiene
B-Administer 5% Marcaine Local Anastatic, prescribe analgesics and pack the
socket
with alvogyl
C-Administer 5% Marcaine Local Anastatic, suture the socket and prescribe
analgesics
D-Suture and give pressure packs
Ans is C
201. 381. The removable partial denture requires relining what is would be the most
appropriate action,
A-take a new impression by asking the patient to occlude on it
B-Provide equal space between denture and gingival tissues.
C-Make sure the framework and retainers are seated in place before taking
impression
Ans is C
203. After obturation and on X-ray you notice the obturation materials are lmm
beyond
apex. What is your first management?
A-Refill the canal
B-Pull the GP cone about lmm out and take a new X-ray
C-Leave it as it
Ans is C or B depend on time that you discovered there is overextention ! If
immediately after placing it you can pull it but if after you did coronal.restoration you
will wait ,,review it periodically to check any changes in preapical area
205.. Four years kid shows at your clinic with open bite as a result of thumb sucking,
you
BY DR.ABDULRAHMAN ALMUALM
MJDF MCQS WITH ANSWERS
206. An 80 year old male presents at your surgery complaining of a sharp stabbing
pain of no
more than 2-3 minutes duration arising from his upper left pre-molar region. The pain
can be brought on by cold stimuli but also occurs spontaneously and has been
sufficiently severe to wake the patient from sleep. He has tried taking paracetemol
but this has been of no benefit. On examination the patient has a heavily restored
upper left 4, which is vital to electrical pulp testing and shows no radiographic
evidence of caries. Blowing cold air onto the tooth produces the pain but the pain
also occurs spontaneously when you are examining the patient. There is no
evidence of a crack or fracture in the tooth itself.
What is the most likely cause of the patient's pain?
A-Acute/reversible pulpitis
B-Dentine sensitivity
C-Chronic/irreversible pulpitis
D-Atypical odontalgia
E-Trigeminal neuralgia
Ans is C
207. A patient complains of a lower incisor which has been mobile for several
months. The
radiograph indicates a normal level of bony support although the periodontal space
has widened. The apical bone appears normal. The tooth is tender to pressure.
Which of the following tests and or examinations would be most likely to provide a
diagnosis?
A-Masticatory muscle palpation
B-Electric pulp test
C-Occlusal examination
D-Ethyl chloride test
E-Hot gutta percha application
Ans is C
208. 9 years old boy come to your clinic with cl 1 occlusion and normal overjet
grossly carious upper left 6 which is
not suitable for restoration. The upper left 7 is very near to eruption. Upper right 6,
lower right 6
and lower left 6 are sound and
fissure sealed. The oral hygiene is good. What is the most appropriate extraction
pattern for this
patient?
A-Extract all four 1st permanent molars
B-Extract upper left 6 and lower left 6
BY DR.ABDULRAHMAN ALMUALM
MJDF MCQS WITH ANSWERS
210. All the films which have been manually processed by an unsupervised new
trainee dental
nurse in your practice are extremely
dark. Which of the following errors could lead to a dark film?
Insufficient exposure
A-Films have been left in the developer for too long a period of time
B-Films have been in the fixer for too long a period of time
C-The concentration of the developer is too dilute
D-The concentration of the fixer is too dilute
Ans is A
211. A new patient attends your practice. At the initial examination, you carry out a
Basic
Periodontal Examination (BPE).
What does a BPE score of 1 signify?
A-Periodontal health
B-Probing depths greater than 3.5mm
C-Presence of overhanging restorations
D-Presence of supragingival calculus
E-Presence of bleeding on probing
Ans is E
BY DR.ABDULRAHMAN ALMUALM
MJDF MCQS WITH ANSWERS
BY DR.ABDULRAHMAN ALMUALM
MJDF MCQS WITH ANSWERS
213.court asked you to testify against another dentist in this case you are :
A. Plaintiff
B. Witness
C.expert witness
D.defendant
E.hostile witness
Ans is c
214. Lignocaine (2%) is widely used in dental procedures. It is most often used in
combination
with epinephrine (1 in 80,000).
In which one of the following patients is the use of epinephrine containing local
analgesia
potentially hazardous?
A-Patient with severe hypertension
B-Patient on tricyclic antidepressants
C-Patient who is an alcoholic
D-Patient on monoamine oxidase inhibitors (MAOls)
E-Patient with Grand Mal epilepsy
BY DR.ABDULRAHMAN ALMUALM
MJDF MCQS WITH ANSWERS
Ans is B
Epinephrine (Primatene, Adrenalin, Ana-Kit, EpiPen, Marcaine) should not be used
with tricyclic antidepressants, since together they can cause severe high blood
pressure.
215.child mother heard that some foods may lead to caries but she isn’t sure what
should she avoid ,from the list below what should she avoid the most:
A.bread
B.cheese
C.ground almonds
D. Fruit juices
E. Vegetable and fruits
Ans is D (check pinkbook)
216. 29 year old man has a prosthetic (mechanical) aortic valve. He had bacterial
endocarditis
five years ago. He now requires
removal of his upper and lower right third molars. Which antibiotic prophylaxis is
recommended
prior to the procedure?
A-Amoxicillin and clindamycin
B-Metronidazole and cephalexin
C-Amoxicillin and gentamycin
D-Amoxycillin and vancomycin
E-Erythromycin and vancomycin
Ans is A ( Amoxicillin 3g for non allergic -clindamycin 600mg for allergic )
217. Pt comes with the career and they have an verbal argue and you have
noticed that caeer were aggressive towards client, what would you do;
A) ignore
B) report to the police
C) ask them politely to book next appointmen
D) report to local safeguarding services
E) report to care home
Ans is D ?
218. patient presents with a history of pain in the right pre-auricular region. There is
an
intermittent click during opening; when
the click is not present the patient can open to a normal range. On examination the
masticatory
muscles, including the lateral
pterygoid, on the RHS side are tender. What is the most likely diagnosis?
A-Myofascial pain
B-Disc Displacement with Reduction
C-Osteoarthrosis
BY DR.ABDULRAHMAN ALMUALM
MJDF MCQS WITH ANSWERS
219. An adult patient complains of her prominent upper front teeth and receding chin.
She says
she did not get 'braces' when she
was younger because the family had to move area a lot. Her oral care and health is
good, and
she has a 9 mm overjet.
Where is the most suitable place to refer her? "
A-A GDP friend who has a special interest in orthodontics and has been trained to
use fixed
appliances.
B-A specialist practitioner who uses a lot of functional
appliances.
C-A private specialist practitioner, because she's too old to
get NHS treatment now.
D-A hospital consultant, as it is likely she will need surgery
now to correct her problem
E-The nearest dental hospital, although it is 70 miles away.
Ans is D
220. 25 year old male attends for the first time complaining of sensitivity of a
number of teeth. On examination, the occlusal surfaces of all the teeth are
worn with obvious wear facets on the canines and premolars. Posterior
amalgam restorations are proud of the surrounding tooth. What would be the
first stage management?
221. 60 year old patient attends your surgery complaining of a sore mouth. He
has Type II diabetes well controlled by diet and metformin. On examination
white patches which cannot be removed are present on his buccal mucosa:
What is the most likely diagnosis?
A ● Frictional keratosis
B ● Leukoplakia
C● Lichen Planus
D● White sponge naevus
BY DR.ABDULRAHMAN ALMUALM
MJDF MCQS WITH ANSWERS
E. Candidosis
Ans is C
A. Temporal nerve .
B.Orbital nerve
C.Zygomatic nerve
D. Buccal nerve
E.Cervical nerve
Ans is B
BY DR.ABDULRAHMAN ALMUALM
MJDF MCQS WITH ANSWERS
4. Left bronchi
5. alveoli
Ans is right bronchi
225. Any patient receiving treatment under IV sedation must have their blood
pressure recorded
as part of their assessment.
What is the maximum blood pressure that is generally regarded as being compatible
with safe
sedation in general dental practice?
A.160/95
B.140/95
C-160/90
D-170/100
E.120/80
Ans is A
226. A 13 year old boy with Down's syndrome attends for the first time. He is
cooperative and
has no relevant medical history. He
is caries free, apart from two small occlusal cavities in his lower second primary
molars. His
mother requests that these teeth are
restored.
Which of the following is the most appropriate management?
A-Arranging to extract these teeth at a future visit.
B-Arranging to place two simple restorations at a future visit.
C-Reassurance that these teeth can be left until they exfoliate naturally, with no
treatment being
necessary.
D-Recommending use of fluoride mouthwash.
E-Take OPG to check on unerupted teeth
Ans is E
227. You are designing a partial denture for a patient with several missing teeth in
the maxilla.
The reason for surveying the model prior to designing the denture is to:
A-Measure and mark out hard and soft tissue undercuts on the casts
B-Relate the intended position of the inter-papillary plane of the patient to the casts
C-Establish the position of the post dam
D-Relate the maxillary and mandibular casts
E-Aid setting up the prosthetic teeth prior to trial insertion
Ans is A
228. A child of 5 years attends with pain from a grossly decayed lower right D which
has a
discharging sinus. He is a hemophiliac.
Which of the following is the most appropriate treatment to relieve his pain?
A-Antibiotics
BY DR.ABDULRAHMAN ALMUALM
MJDF MCQS WITH ANSWERS
B-Extraction
C-Fluoride application
D-Non-vital pulpotomy
E-Vital pulpotomy
Ans is D
229. A 23 year old patient attends complaining of pain in an upper right molar and is
keen to
keep the tooth. The pain is typically
sharp in nature, is triggered by cold and persists after removal of the cold stimulus.
The tooth is
not tender to percussion; a
radiograph of the upper right first molar shows a large radiolucency extending to the
pulp horn
but no peri-radicular changes.
What treatment is most likely needed in this case?
A-Oral hygiene instruction and fluoride application
B-Excavation of caries and placement of a permanent restoration
C-Root-canal treatment
D-Indirect pulp cap and restoration
E-Direct pulp cap and restoration
Ans is C
230. An obese 40 year old who takes metformin is seen at the end of a morning
clinic. After
administration of local analgesia in thesupine position the patient complains
of feeling unwell. She is pale and sweating and is
confused. No pulse was detectable. She
was given oxygen and maintained in the supine with no improvement in
condition. What would
be the most appropriate drug to administer next?
A-Glucagon
B-Glucose
C-Adrenaline
D-Hydrocortisone
E-Chlorphenamine
Ans is A.patient is unconscious
231. There has been much debate regarding the effectiveness of fluoride in water for
preventing
tooth decay. A systematic review
of the evidence drew conclusions as to the reductions in decay which can be
expected.
Fluoridation of public water supplies:
BY DR.ABDULRAHMAN ALMUALM
MJDF MCQS WITH ANSWERS
232. The biological process by which the architecture and function of the lost tissue
is completely
restored".
In Periodontology this is a definition of which of the following
A-New Attachment
B-Regeneration
C- Repair
D-Primary Healing
E-Reattachment
Ans is B
235 A 23 year old male presents to your surgery. He lost his upper lateral incisors
some 10
BY DR.ABDULRAHMAN ALMUALM
MJDF MCQS WITH ANSWERS
years ago in a swimming pool accident. Since then he has been wearing a 'spoon'
denture
which he now feels in aesthetically unacceptable. He has sought an opinion on
dental implants but has been told that he would need bone grafting for this to be
successful and
he is not prepared to undergo this. His dentition is excellent with no restorations and
a Class I
occlusion. He wants some advice on what the best treatment might be.
Which option would you put first on your list of possibilities?
● Two fixed - fixed resin bonded bridges using the central and canine teeth
● Two cantilever resin bonded bridges from canines.
● Two conventional fixed - fixed bridges from the canine
● Conventional cantilever bridges from the canines
● Cobalt chrome partial denture
Ans is B
237. The patient wants removable partial denture to restore his missing teethfrom
second premolar
in both sideof lower mandible. What would be the best retention
for this case?
1. Mesial rest, I bar
2. Distal rest, I bar
3. Mesial & distal rest, Ibar
4. Circumferential clasp
5. C clasp withl bar.
Ans is A
238
You want to treat Pagetwith
medication. Which of the following
is theleast possibleto develop
MRONJ?
1. Denosumab
2. Risedronate
3. Calcitonin
4. Etidronate
5. ibandronate
Ans is 3
BY DR.ABDULRAHMAN ALMUALM
MJDF MCQS WITH ANSWERS
1. Vitamin D
2. Vitamin B12
3. Iron
4. Folate
5. Vitamin K ||||| Ans is 1 (Vit d)
240. You were asked to do help your Nurse about her CPD.
Which one of the following is the
minimum requirement?
1. 25 hours
2. 50 hours
3. 75 hours
4. 10 hours
5. 35 hours
Ans is 2
a. Myelogenous leukaemia
b. Infectious mononucleosis /glandular fever/
c. Thrombocytopenic purpura
d. Gingivitis of local aetiological origin
e. Pernicious anaemia /Vitamin B12 deficiency/
Ans is A
242. Symptoms free patient comes to you after four weeks of an endodontic
treatment and you
find on radiograph the canal is over filled with what it seems to be a cone of Gutta
Percha lmm
beyond the apex with a radiolucent small area. What is your initial
management?,
● Start apiectomy through a flap and surgery
● Obturate the root canal
● Ask for a recall and observe in three months time
● Seal the pulp chamber and keep it under observation
Ans is C
BY DR.ABDULRAHMAN ALMUALM
MJDF MCQS WITH ANSWERS
Ans is3 //
244.
Which nerve is affected if a patient is unable to gaze laterally to the left?
-rt abducent
-rt trochlear
-lft abducent
-lft trochlear
Ans is c
248. If your nurse had Needle stick injury, who should she report first?
BY DR.ABDULRAHMAN ALMUALM
MJDF MCQS WITH ANSWERS
1. You, operator
2. Receptionist
3. Manager
Ans is 1
Ans is C
253. the patients didn’t satisfy with your settlement, which he/she
could go to before going to
court?
1. Healthcare commissioner
2. Private inspector
3. Independent service
4. Public inspection service
5. GDC
Ans is 1
254. You want to put stainless
steel on 6yrs old child because
of severe gross caries and
he already
BY DR.ABDULRAHMAN ALMUALM
MJDF MCQS WITH ANSWERS
255. 25yrs old female patient whois quite anxious consult you for extraction of lower
impactedlast
molar. X-ray revealed that there is high risk of nerve injury but the tooth needto go.
You
concerned about patient anxiety and didn't explain about every detail about
complication.After surgery, patient develop paresthesia over lower lip. If she
wanted to sue you, onwhich ground she can?
1. Gross negligent
2. Consent negligent
3. Malpractice
4.civil assault
5.criminal assault
Ans is 2
BY DR.ABDULRAHMAN ALMUALM
MJDF MCQS WITH ANSWERS
259. When you Air dry lower left first molar, you see white
lesion on teeth surface. What would that
be?
1. Enamel demineralization
2. Fluorosis
3. Enamel mineralization
Ans is 1
1. Diplopia
2. Infraorbital paranesthesia
3. Ptosis
4. Subconjunctival hemorrhage
5. trismus
Ans is 5 /// all other signs are for zygomaticocomplex fracture
264.Pt with normal centric occlusion but have interferance in eccentric movement
what is your
treatment:
A-Grinding of lingually inclined maxillary teeth
BY DR.ABDULRAHMAN ALMUALM
MJDF MCQS WITH ANSWERS
265. At which angle to the external surface of proximal cavity walls in a class
II preparation for amalgam should be finished
A. An acute angle
B. An obtuse angle
C. A right angle
D. An angle of 45°
Ans is c
266. A 10½ year old boy with an uncrowded sound dentition attends your
practice.His permanent canines are unerupted and not palpable and primary
canines are retained. On radiographic examination, you find the canines are
mesially inclined and in the line of the arch. What is the most appropriate
management option?
BY DR.ABDULRAHMAN ALMUALM
MJDF MCQS WITH ANSWERS
271. 261. Probe pressure at the sulculus of pocket should be enough to:
a. Feel the top of the crestal bone
b. Balance the pressure between fulcrum and grasp
c. Define the location of the calculus deposit
d. Feel the coronal end of the attached tissues
e. Limit the lateral pressure
ans is D (note probe pressure is 25 g)
272. You are interested in finding out what the risk indicators are for a
rare form of oral cancer and decide to undertake a study to examine this.
What type of study would be the most appropriate for addressing this
issue?
A. Cohort
B. Prevalence study
C. Clinical trial
D. Case-control study
E. Case-series
Ans is a
Types of probes
Nabers probe
North
Carolina p
BY DR.ABDULRAHMAN ALMUALM
MJDF MCQS WITH ANSWERS
273. What enzyme assists microorganisms when they are causing dentine
caries?
a.Collagenase
b.Enolasec.
c. Lactoferrin
ans is B
BY DR.ABDULRAHMAN ALMUALM
MJDF MCQS WITH ANSWERS
●Medial pterygoid
●Superior constrictor and Buccinator
Ans is B
280After the age of 6 years, the greatest increase in the size of the mandible
occurs:
A. At the symphysis
B. Between canines
C. Distal to the first molar
Ans is C
281. Which of the following does state BEST the morphology of periodontal
ligament fibres:
A. Elastic
B. Striated
C. Non striated
D. Levity
E. Wavy
Ans is E
282. Patient whose hands fell warm and moist is MOST likely to be suffering
from:**
A. Anxiety
B. Congestive cardiac failure
C. Thyrotoxicosis
Ans is C
BY DR.ABDULRAHMAN ALMUALM
MJDF MCQS WITH ANSWERS
Overcontouring,
undercontouring ,
overhang.
Fracture
Ans is b (undercountring)
284. 20. You used N-type sterilizer for your instrument with proper packing. How
long can these instruments be stored?
A. 4 months
B. 8 months
C. 12 months
D. 15 months
E. 24 months
Ans is C// B type sterlizer don’t store
285. 26. You realize a problem calibrating delivery gas while treating patient with
Nitrous oxide. Who should you report to?A. Care quality commission
B. Clinical commissioning group
C. Device manufacturer
D. Health and safety executive
E. Medicine and healthcare product regulatory ageny
Ans is E
286. Which of the following has highest five year survival rate for class V
restoration?
A. Amalgam
B. Bonded amalgam
C. Composite (flowable)
D. Conventional GIC
E. Resin modified GIC
Ans is E
BY DR.ABDULRAHMAN ALMUALM
MJDF MCQS WITH ANSWERS
BY DR.ABDULRAHMAN ALMUALM
MJDF MCQS WITH ANSWERS
292.A 46 year old female presents with a slowly enlarging painless firm swelling in
the hard
palate to the left of the midline.
The most likely diagnosis is:
● A dental abscess
● Torus palatinus
● Osteoma
● Pleomorphic adenoma
● Canalicular adenoma
Ans is D pleomorphic adenoma
294. A curette may be inserted to the level of the attached gingiva with
minimal trauma to the tissues because of
A. Has a round base
B. Is easy to sharpen
C. Has rounded cutting edges
D. Provides good tactile sensitivity
E. Has two cutting edges
Ans is C
BY DR.ABDULRAHMAN ALMUALM
MJDF MCQS WITH ANSWERS
296. 52. Which of the following anomalies occurs during the initiation and
proliferation stages of tooth development
A. Amelogenesis imperfecta - histodifferentiation
B. Dentinogenesis imperfecta- histodifferentiation
C. Enamel hypoplasia – apposition stage
D. Oligodontia – initiation stage ( absence of single or multiple teeth)
E. Ankylosis Hypocalcified/hypomineralised – normal organic matrix but
defective mineralization.
Ans is D
297. The management of patients taking corticosteroid for long time with
dental infection, abscess or high temperature, do we need double the
dose of antibiotic or double the dose of
cortisone
ANS double dose of cortisone to prevent
adrenal crisis
BY DR.ABDULRAHMAN ALMUALM
MJDF MCQS WITH ANSWERS
Ans is B
Ans is E
BY DR.ABDULRAHMAN ALMUALM
MJDF MCQS WITH ANSWERS
303. An 80 year old patient presents with an ulcer in the floor of the
mouth. This has been present for several months and has not
responded to conventional treatment. An incisional biopsy is taken.
Which of the following histological changes in the epithelium confirm a
diagnosis of squamous cellcarcinoma?
A. Hyperkeratosis
B. Acanthosis
C. Dysplasia
D. Invasion
E. Discontinuous epithelium
Ans is D
BY DR.ABDULRAHMAN ALMUALM
MJDF MCQS WITH ANSWERS
BY DR.ABDULRAHMAN ALMUALM
MJDF MCQS WITH ANSWERS
c. immune to infection
Ans is A
314. case scenarios where one central incisor is missing in a child who
has had a previous historyof trauma with an avulsed primary incisor, the
cause is
a.scar tissue
b.supernumerary teeth
c.tubercle
d.frenum
Ans is A /trauma lead to dilaceration to developing tooth
315. A mother brings her child to the out of hours clinic suffering
spontaneous excess bleeding from his gingival, what is the
expected diagnosis?
- acute leukemia
- injury
- stress
- scurvey
Ans is A
BY DR.ABDULRAHMAN ALMUALM
MJDF MCQS WITH ANSWERS
318. You suspect the patient suffers hepatitis B symptoms, after investigation,
the test show HBe Ag antibodies, what does that indicate?
- the patient is getting better
- the patient is in a transmitting state of the disease
Ans is B
323. You arrive at a new practice and notice that almost every
radiograph in the patient's notes has turned brown. Your nurse confirms
that this is a widespread problem that no-one has ever remedied and
she also remarks that the films tend to get browner with age. What
corrective action will you take to remedy the problem?
BY DR.ABDULRAHMAN ALMUALM
MJDF MCQS WITH ANSWERS
325. The most accurate way to evaluate the effectiveness of root planning is
by:
a. Inspect the root surface with an instrument for root smoothness
b. Use air for visual inspection
c. Activate a curette against root surface and listen for a high pitched
sound which indicates as mooth, hard surface.
d. Evaluate the soft tissue at the end of the appointment for a
decrease oedema and bleeding
e. Evaluate the soft tissues 10 to 14 days later.
Ans is A
326. The most common type of injury caused by a Non accidental Injury is:
- burnt tip of tongue
- ulcer on the gingiva
- lacerated labial frenum
- neck bruise
Ans is d
BY DR.ABDULRAHMAN ALMUALM
MJDF MCQS WITH ANSWERS
332. 8. Which of the following does not act by interfering with cyclooxygenase
pathway of arachidonic acid
A) A. Aspirin
B) B. Diclofenac
C) C. Ibuprofen
D) D. Dihydrocodeine
E) E. Paracetamol
Ans is D,?
333. 1. Which of the following is cyanotic congenital heart condition?
A) A. artrial septal defect
B) B. coarctation of aorta
C) C. floppy mitral valve
D) D. patent ductus arteriosus
E) E. transposition of great vessels
Ans is E
Von Ebner
Neonatal line
Perikymata
Non of the above
Ans is D
335. When treating a tooth with a non-vital pulp with a fistula presented;fistula
should be treated by
BY DR.ABDULRAHMAN ALMUALM
MJDF MCQS WITH ANSWERS
A. Surgical incision
B. Antibiotic coverage
C. The usual root canal procedures for non-vital teeth and no special
procedures for fistula
Debatable Question some answered A some as C
336. When primary molars are prepared for stainless steel crowns should the
depth for reduction of the proximal surface be similar to the depth of the
buccal and lingual surfaces?
337. Which of these muscles may affect the borders of mandibular complete
denture,
A. Mentalis
B. Lateral pterygoid
C. Orbicularis oris
D. Levator angulioris
E. Temporal
Ans is B
338. Electrical pulp testing is least useful in /or does not detect in some
papers/
, A. Traumatised teeth
B. Just erupted teeth
C. Multi-rooted teeth
D. Capped teeth
E. Necrotic pulp
Ans is A
BY DR.ABDULRAHMAN ALMUALM
MJDF MCQS WITH ANSWERS
342. You are interested in finding out what the risk indicators are for a
rare form of oral cancer and decide to undertake a study to examine this.
What type of study would be the most appropriate for addressing this
issue?
A. Cohort
B. Prevalence study
C. Clinical trial
D. Case-control study
E. Case-series
Ans is D (repeated Qs with different ans )
BY DR.ABDULRAHMAN ALMUALM
MJDF MCQS WITH ANSWERS
c.0.01mSv
d. 1.0mSv
e. 10 mSv
Ans is B
351. What cement u use to glue a alumina core ceramic crown?- GIC-
RMGIC-
COMPOSITE RESIN CCEMENT
BY DR.ABDULRAHMAN ALMUALM
MJDF MCQS WITH ANSWERS
Ans is B
354. An upper deciduous molar has a caries exposure and on X ray the
corresponding 2nd permanent premolar is absent. What treatment would you
do to the deciduous tooth:
A. Pulpotomy
B. Endodontic treatment
C. Pulp capping
Ans is B
355. How many pulp horns are presented in a typical mandibular deciduous
second molar: A. 2
B. 3
C. 4 .
D. 5
Ans is c
BY DR.ABDULRAHMAN ALMUALM
MJDF MCQS WITH ANSWERS
4). Radiographs
5). Staining
359. If amalgam gets contaminated with moisture, the most uncommon result
is: **A. Blister formation
B. Post operative pain
C. Secondary caries D. Lower compressive strength
Ans is A
360.If the sealant of bonding agent is not placed on part of enamel that has
been etched by an acidsolution; you would expect:
A. Arrest of enamel carries by organic sulphides
B. The enamel is to return to normal within 7 days
C. Continued enamel declassification in the etched area
D. Slight attrition of the opposing tooth
Ans is C
361. A major difference between light cured and chemical cured composite is
that during setting or in function the light cures material tends to:
BY DR.ABDULRAHMAN ALMUALM
MJDF MCQS WITH ANSWERS
Ans is A
BY DR.ABDULRAHMAN ALMUALM
MJDF MCQS WITH ANSWERS
neutrophils,
macrophages
ans is D
372. If patient can't open after IDN block which muscle of mastication affected?
A. Medial pterygoid
B. Lateral pterygoid C. masseter
Ans is A.
373. Deep needle insertion during the standard inferior alveolar nerve block may
result in:
A. Premature bone contact
B. . A greater success rate.
C. Contact with the temporal crest of the ramus
. D. Penetration of the parotid capsule.
Ans is D
374. Which one of the following is most commonly used to bleach vital teeth:
A. . Ethyl chloride
B. Hydrogen chloride
C. Hydrogen peroxide
D. Sodium bicarbonate
E. Sodium hypochlorite
Ans is C
375. Patient with Lateral periodontal abscess ,what is the treatment of choice
A. Metronidazole
B. Amoxicillin
C. Tetracycline
D. Drainage of the pus and analgesic
Ans is D
BY DR.ABDULRAHMAN ALMUALM
MJDF MCQS WITH ANSWERS
377. Which of the following salivary gland tumours is most likely to occur
bilaterally?
a) pleomorphic adenoma
b)mucoepidermoid carcinoma
c)adenoid cystic carcinoma
d)acinic cell carcinoma
Ans warthin tumor
378. 206 Which sealant is more likely to cause tissue damage if extruded
– AH plus,
grossman's sealer,
tubliseal
Ans is C
379. A 35 year old male patient who admits to grinding his teeth at night has a
number of wedge-shaped cervical (Class V) lesions on his upper premolar teeth.
These are causing some sensitivity and are approximately 3mm deep. What is the
correct management option?
380. Which of the following is the ideal treatment for a degree II furcation
involvement of a
mandibular molar?
● Tunnel preparation
● Root resection
● Furcation plasty
● Extraction
● Guided Tissue Regeneration
Ans is E for Class III furcation --> tunnel preparation
For Grade I --> furcationplasty
381. A 7 year old boy has previously had all primary molars restored and a
pulpotomy on upper
BY DR.ABDULRAHMAN ALMUALM
MJDF MCQS WITH ANSWERS
right E. He has an early mixed dentition with lower lateral incisors erupting. There is
a midline
diastema of 2 mm. The upper right E has become symptomatic and
requires extraction.The most likely long term effect of the extraction on the occlusion
is:
● Early eruption of the second premolar.
● Loss of upper central line.
● No significant effect.
● Overeruption of the lower right teeth.
E. Potential crowding in the upper right quadrant
Ans is E
382. Six months ago you saw a child patient, then aged 9 years. His upper right
maxillary
canine was palpable in the labial sulcus but the upper left was not. The situation is
now
unchanged, so you have taken two periapicals of the non-palpable tooth. They both
show that
there is some resorption of the CI root but the permanent canine appears somewhat
mesially
angled and is more mesial on the more mesially positioned film.
What is your the best course of action?
● Keep a careful watch on it and take another x-ray in 6 months.
● Refer to an oral surgeon for early exposure of the permanent canine.
● Refer to an oral surgeon for early removal of the permanent canine before it
damages
the lateral incisor.
● Refer to an orthodontist for a treatment
Ans debatable Qs some answered E some c
384. A 9 year old boy has a small white discolouration on his maxillary central
incisor. The
lesion is most probably,
● Hypocalcification due to trauma of the primary predecessor
● Hypoplasia due to acute systemic infection when 6-12 months old
● Defect during the histo differentiation stage of development
● Defect during the morho differentiation stage of development
Ans is A
BY DR.ABDULRAHMAN ALMUALM
MJDF MCQS WITH ANSWERS
● The ridge height is lost more from the maxilla than from the mandible
● The maxillary ridge will get more bone lost from the palatal aspect than the buccal
● The mandibular arch is relatively narrower than the maxillary arch
● Compared with the pre-resorption state, the mandibular ridge will lose more bone
from the lingual aspect than the buccal one.
Some answered C some D
386. 178 In planning and construction of a cast metal partial denture the study cast
● facilitates the construction of custom trays
● minimizes the need for articulating
● provides only limited information about inter ridge distance,
Ans is A
387. A 50 years-old patient presents with pain from time to time on light cervical
abrasions.
What is your first management to help patient in preventing pain in the future?
A.Change diatary habits
B.Change brushing habits
C.GIC fillings ans is c and B
388. The most common cause of porosity in porcelain jacket crowns is,'
● Moisture contamination
● Excessive firing temperature
● Failure to anneal the platinum matrix
● Excessive condensation of the porcelain
● Inadequate condensation ofthe porcelain
Ans is e
389. Which of the following liquids is not suitable for prolonged immersion of cobalt
chrome
partial dentures:
A. Alkaline peroxidase
B. Sodium hypochlorite
C.Soap solutions
D.Water
Ans is b
391. The flexibility of the retentive clasp arm does not depend on:
● Length of the arm
● The cross section shape
● The material used
● Degree of taper
● The exerted force
Ans is e
BY DR.ABDULRAHMAN ALMUALM
MJDF MCQS WITH ANSWERS
392. Following calcium hydroxide pulpotomy, the dentist would expect dentine bridge
to form at,
● The exact level of amputation
● Level somewhere below the amputation
● Half way between amputation and apex
● At the apical region of the tooth
Ans is b
393. Where would you expect to find the mylohyoid muscle in relation to the
periphery of a full
lower denture:
A.Mandibular buccal in the midline
B.Mandibular lingual in the first premolar area
C.Mandibular lingual in the midline
D.Mandibular disto buccal area
Ans is B
394. In anxoius and psychologically stressed patients gingivitis is often more severe
because of
A.Stress causes histamine and serotonine release
B.stress causes catecholamine and corticosteroid release
c.Stressed people neglect their oral hygiene
Ans is b
395. What does not help in establishing the caries risk in children?
A.History of caries
B.Lactobacilius count
C.Dietary habits
D.Brushing habits
E.Genetic predisposition
Ans E
BY DR.ABDULRAHMAN ALMUALM
MJDF MCQS WITH ANSWERS
398. While removing the second primary molar of 9 years old child, the apical third of
the root
fracture and stay in the socket,
● You will just leave it and observe it
● You take surgically by a lingual flap
● You try to take out by using a root apex elevator
● You use a fine end forceps to take it out
Ans is A
399. 32-year-old man was treated for a fractured mandible and had post-operative
radiographs taken, which one of the following treatments has he received?
.401 In relation to resin-retained bridges the term retainer describes: The cement
attaching the wing to the tooth
A. The prosthetic tooth within the bridge
B. The tooth to which the bridge is secured
C. The wing portion of the bridge that is attached or cemented Jo the natural tooth
D. None of the above
Ans is c
BY DR.ABDULRAHMAN ALMUALM
MJDF MCQS WITH ANSWERS
403Assuming that the distance between the gingival margin on the lingual su face of
the lower anterior teeth and the base of the lingual suIcus is 7 mm which of the
following is the most appropriate major connector to prescribe in this case?
A. Continuous plate
B. Dental bar
C. Lingual bar
D. Lingual plate
E. Sublingual bar
Ans is C
.404 Which one of the following statements best describes statistical power?
a) This is when a true null hypothesis gets rejected
b) This is when a false null hypothesis gets accepted
c) Refers to the probability that the observed statistical difference has
occurred by chance
d) It shows that there is no difference in outcome between two groups after an
intervention in one of the groups
e) The probability of correctly rejecting a false null hypothesis
Ans is E
. 405 In the construction of a full veneer gold crown, future recession of gingival
tissue can be
prevented or at least minimised by,
● Extension of the crown 1 mm under the gingival crevice
● Reproduction of normal tooth incline in the gingival one third of the crown
● Slight over contouring of the tooth in the gingival one fifth of the crown
● Slight under contouring of the tooth in the gingival one fifth of the crow
Ans is B
.406 patient had a superficial parotidectomy 6 weeks ago. They report that an
area of their cheek on the same side becomes red and sweats at mealtimes.
Which nerve has been damaged?
a) Auriculotemporal branch of the trigeminal nerve
b) Buccal branch of the facial nerve
c) Mandibular branch of the trigeminal nerve
BY DR.ABDULRAHMAN ALMUALM
MJDF MCQS WITH ANSWERS
.407 13 years old boy comes to you with excessive hyperplasia of the gingiva as a
result of
Phenytoin what is your management,
● Stop the medication
● Force a strict oral hygiene and surgical removal of excess gingival tissues
● Debridement and conservative approach
Ans is B ?
.408 What is the main purpose of using corticosteroids in pulpal obturation material?
● For their antibiotic action
● For their antiinflammatory action
● To relief pulp pressure
Ans is B
.412 Which one of the following combinations produces the lowest stress within a
root when used as a post and core?
A. Cast metal post and core
B. Composite resin post and core
C. Composite resin core in combination with a glass fibre post
BY DR.ABDULRAHMAN ALMUALM
MJDF MCQS WITH ANSWERS
.413 When designing a partial denture, which one of the following steps comes first?.
A. Direct retention
B. Minor connectors
C. Support
D. Indirect retention
E. Major connectors
Ans is C
1. Support.
2. Direct retention.
3. Indirect retention.
4. Major connectors.
5. Minor connectors.
Ans is b
BY DR.ABDULRAHMAN ALMUALM
MJDF MCQS WITH ANSWERS
.418 The most common drug that causing ulcer resembling Squamous cell
carcinoma ulcer :
Ans is Nicorandil
.420 Pt are Complaining about the jaw pain two weeks after RCT, what is going be
your first management
• A) do RCT again
• B) reassure , sof diet and review in two weeks time
• C) soft splint
• D) remove the tooth
Ans is B
.421 How to know that non surgical periodontal therapy had effect?
• A) less than 15% bleeding on probing
• B) pockets getting bigger
• C) more than 15% bleeding on probing
• D)recession is controlled
Ans is A
422
BPE score is
324
3 2 2*
How to manage this case?
• A) refer to secondary care
• B) refer to GP
• C) refer to your colleague with special interest in perio
• D) refer to specialist in primary care
Ans is A
BPE 4 --> periodontist (secondary care)
BPE 3 --> GDP with special interest in perio
423BCC spread by
BY DR.ABDULRAHMAN ALMUALM
MJDF MCQS WITH ANSWERS
• A) locally
• B) lymphatically
• C) hematogenus
Ans is is A
.424 What is the most likely infection that can be caught by NSI in fully immunized
person? •
A)HepB
• B)HepC
C)HIV
Ans is B
.425 Patient had BMI=41. Which of the following is his ASA classification? •
A)ASAI
• B) ASA II
• C) ASA III
• D) ASA IV
Ans is C
BMI <30 --> ASA I (normal)
BMI 30-40 --> ASA II (mild systemic illness)
BMI >40 --> ASA III (severe systemic illness).
.426 Recent studies have shown that new drug therapy can avoid
osteoradionecrosis in the treatment of pt with bone metabolism disorder or cancer.
Which of the following does not cause MRONJ?
• A) alendronic acid
• B) calcitonin
• C) sunitamb
• D)denosumb
• E) zolendronic acid
Ans is B
.427 BPE score for the lower anterior teeth with pocket depth of 3mm with recession
of 3mm?
• A)0
• B)1
• C)2
• D)3
• E)4
Ans is A( recession part is not included )
.428 . Your associate has just found that he missed GDC renewal and not paid GDC
fee. What is your next action as his employer?
• A) suspend
• B) no action, it is associate’s problem
• C) inform the GDC
Ans is A suspend
BY DR.ABDULRAHMAN ALMUALM
MJDF MCQS WITH ANSWERS
.429 . A pt was referred to you for extraction of 5 by the ortho specialist in prepration
for ortho treatment. You mistakenly extracted the wrong tooth. What is the
consequence if you fail to report this incident?
• A) nothining if pt does not complain
• B) erasure from GDC
• C) criminal offence
• D) suspension
Ans is D
.430 Pt told the GP that she lost 4kg in last 4 months, have black stool, no
gastrointenstinal polyp, Lab results are; HB=80, low MCV, normal WBC, normal
palatal count.. What is your diagnosis?
• A) hypothyroidisam
• B ) B12 defficency
• C ) pernicious anemia
• D )chronic blood loss
Ans is D
Low MCV and MCHC with low Hb --> microcytic anaemia --> iron deficiency or
chronic blood loss
Dark stool is a sign of a GIT ulcer with bleeding.
.431 A83 year old pt with dementia comes to your practice with discharge pus from
parotide gland and does not have a family, also she does not have elected power of
attorney, who can give a consent for her?
• A ) her close relative
• B) her good friend
• C ) independent mental capacity advocate
• D) her GP
• E) her caregiver
Ans is C / best ans is do what is the best of interest of the patient
433. You are going to employ clinical dental technician, which of the following
procedure you expected from him to do independently?
• A)diagnose and treatment planning of perio disease
• B) complete denture without the consultation with dentist (correct ans)
• C) administrate LA prior the ridge augmentation
BY DR.ABDULRAHMAN ALMUALM
MJDF MCQS WITH ANSWERS
.435 A 11 year old girl with multiple caries, what would you advice to your
patient from following?
• A) use 5000ppm fluoride toothpaste
• B ) use 2800ppm F toothpaste
• C ) topical fluoride of 11600 ppm 2 or more times per year
Ans is B ,patient older than 15 ans is C
.436 4 years old girl comes with the mother, she has good hygiene and no caries,
living in area with fluoridated water, mother is asking about toothpaste and your
recommendation is:
• A) do not swallow toothpaste
• B ) use fluoride free toothpaste
• C) rinse with a water after brushing
• D) do not rinse after brushing, just spit all out and go to the bed
Ans is A (also D is right )
BY DR.ABDULRAHMAN ALMUALM
MJDF MCQS WITH ANSWERS
Ans is C
.439 Pregnant pt complaining about swelling of her gums. Upon examination you
have noticed that is a pregnancy epulis, which of the following is correct?
• A) no treatment required, lesion will resolve by itself
• B) the presence of epulis is independent if pt oral hygiene
• C)surgical treatment mandatory after delivery
Ans is A
.440 A 6 years old child comes to you with fever, poor appetite, feeling of being
unwell, sore throat. 4 days later fever starts, painful rash, blister develop at the hand,
feet and buttocks. What is the causative agent?
• A) Herpes simplex
• B) paramyxovirus
• C) Coxackie virus
• D) HPV 16
Ans is C
.441 Kenedy class II classification, which of the following design is most suitable for
optimal function?
• A) distal rest and I bar
• B) occlusal approaching clasp
• C ) dental bar
• D) mesial rest and I bar
Ans is D
.442 A student did a study and found that her data skewed. Which of the following
data will be relevant/valuable in this scenario?
a. Mean, median
b. Median, upper and lower
c. Median, IQR
d. Mean and IQR ans is C (Skewed data --> non-normally distributed data -->
Median,and IQD so it's C.)
443(An OPG was given showing fracture line on the parasymphysis area)
What is the next radiograph will you order to further understand your findings?
a. SMV
b. OCM
c. PA Jaw
d. La Ceph
Ans is C PA mandible / SMV IS USED FOR ZYGOMA FRACTURE
444 Patient had a BMI of 41. Which of the following is her ASA Classification?
a. ASA I
b. ASA II
c. ASA III
d. ASA IV
Ans is C
BY DR.ABDULRAHMAN ALMUALM
MJDF MCQS WITH ANSWERS
.446. A patient came to you for GIC filling few months ago and now attend to you for
Stainless Steel Crown. Which of the following NHS Band can you claim?
a. GDP 21.60
b. GDP 59.10
c. GDP244.30
d. GDP299.90
ans is B
.447 Patient was diagnosed with cancer and she told her family it metastasis through
nerve. Which of the following is your diagnosis?
a. Adenoid Cystic Carcinoma
b. Pleomorphic Adenoma
c. Whartin’s Tumour
d. Sialolithiasis
e. Mucoepidermoid Carcinoma
Ans is A
448 You have been called in by the owner of the practice regarding you lack of
punctuality to work. Which of the following is the appropriate response?
a. My lack of punctuality does not affect patient flow
b. My lack of punctuality does not affect nurse staff flow
c. My lack of punctuality will be compensated with me staying longer after hours
d. My lack of punctuality can be better improved if the schedule was adjusted
ans is d
449What is the referral time for suspicious cancer lesion and for secondary care?
a. 1 week
b. 2 week, 20 weeks
c. 2 week, 18 weeks
Ans is c
450. 13. Patient had his history of epilepsy and presents to your practice with a
painful dental abscess. She is currently on Carbamazepine. Which of the following
antibiotic is contraindicated?
a. Amoxicilin
b. Erythromycin
c. Clindamycin
d. Metrodinazone
e. Tetracycline
Ans is B
BY DR.ABDULRAHMAN ALMUALM
MJDF MCQS WITH ANSWERS
.451. Your practice owner have just extracted a patients LL7. You were informed that
the patient has return a few visits for treatment of Dry socket. Today you see the
patient and decided to take another xray. The x-ray taken have shown the following
(an xray with fractured mandible w submerged tooth at the border of mandible)
What are your findings?
a. Fractured mandible
b. Fractured mandible with retained LL7
c. Displaced LL7
Ans is B
BY DR.ABDULRAHMAN ALMUALM
MJDF MCQS WITH ANSWERS
.456 Patient told her Gp she lost 3kg 4month have black stool with no
gastrointestinal polyp.
Lab results was:
- Low HB
- Low MCV
- Normal WBC
- Normal Platelet Count
.457 Which of the following anesthetic agent do not require any anti-emetic?
a. Fentanyl
b. Ketamine
c. diamorphine
d. Pethidine
Ans is A
.459 A student is carrying out a study on the effective use of antibiotics. Which of the
following do you would be relevant in this context?
a. Guardianship of antibiotics
b. Antibiotics resistance
c. Prophylactic antibiotics
ans is A
.460 With regards to burning mouth syndrome, which of the following is correct?
a. It disturbs sleep
b. It aggravates during eating
c. The intensity is bilateral
ans is B
.461 A 8 years old boy came to your dental clinic with the complain of impacted 11.
You manage to diagnose reason of delayed eruption of 11 and it was due to the
presence of a supernumerary teeth. Which of the following will not affect the eruption
of permanent 11.
a. Type of supernumerary
b. Number of supernumerary
c. Stage of root formation
ans is A ( number will affect but type ?? Any supernumery can affect the eruption of
secondary tooth)
462. You manage to complete a 11 year old orthodontic case. How long should you
keep her record?
BY DR.ABDULRAHMAN ALMUALM
MJDF MCQS WITH ANSWERS
a. 11 years
b. 14 years
c. 5 years
Ans is B( either 14 years or when patient reach to 25 years )
463 In the new NICE guidelines, antibiotic prophylaxis is no longer needed during
dental treatment for patients with risk of infective endocarditis. It was needed
previously due to the risk of which bacteria?
a. Streptococcus mutans
b. Streptococcus viridans
c. Staphylococcus Aureous (ans is B)
.464 1. Periodontal damage to abutment teeth of partial denture with
distal extension can best be avoided by
A. Applying Stressbreakers
B. Employing bar clasps on all abutment teeth
C. Maintaining tissue support of the distal extension
D. Clasping at least two teeth for each edentulous area
E. Maintaining the clasp arms on all abutment teeth at the ideal degree of
Tension
Ans is C / yes stress breakers are important but maintaining tissue support can
reduce forces over the abutment greatly
BY DR.ABDULRAHMAN ALMUALM
MJDF MCQS WITH ANSWERS
.469 . You are going to employ clinical dental technician, which of the following
procedure you expected from him to do independently?
• A)diagnose and treatment planning of perio disease
• B) complete denture without the consultation with dentist
• C) administrate LA prior the ridge augmentation
Ans is B
Ans is A
BY DR.ABDULRAHMAN ALMUALM
MJDF MCQS WITH ANSWERS
.476 Antimicrobials should be prescribed for one of the following clinical scenarios:
a) After apicectomies
b) If there is a sinus present
c) After periodontal surgery
d) For surgical removal of mandibular third molars
e) After surgical removal of retained roots and teeth where there is a history of dry
sockets
ans is non of the above ..according to recent guidelines no antibiotic should
prescribed unless signs of systemic affection
BY DR.ABDULRAHMAN ALMUALM
MJDF MCQS WITH ANSWERS
A)Active periodontitis
B)smoking
C)warfarin
Ans is A
481)suitable time to extract first molar ?due to long term bad prognosis or
orthodontics
A)8_10 y
B)10_12 y
C)12_13 y
D)13_14y
Ans A
BY DR.ABDULRAHMAN ALMUALM
MJDF MCQS WITH ANSWERS
Ans is A
A)6 months.
B)12 months
C)18 months
Ans C
BY DR.ABDULRAHMAN ALMUALM
MJDF MCQS WITH ANSWERS
Ans b
495A patient collapses in the dental practice surgery waiting room. They have
been fitting for 5 minutes, what is your next course of action?
a) Buccal midazolam
b) Chest compressions
c) Intramuscular midazolam
d) Rectal diazepam
e) Two rescue bbreath
Ans is A
BY DR.ABDULRAHMAN ALMUALM
MJDF MCQS WITH ANSWERS
b) Atropine
c) Methionine
d) N-acetylcysteine
e) Naloxone
Ans is D
498Which one of the following is the greatest risk factor for periodontal
disease?
a) Plaque retentive factors
b) Down’s syndrome
c) Shortened dental arch
d) Smoking
e) Intravenous drug abuse
Ans is D
BY DR.ABDULRAHMAN ALMUALM
MJDF MCQS WITH ANSWERS
502Which one of the following interacts with warfarin to decrease the patient’s
INR?
a) Fluconazole
b) Vitamin K
c) Metronidazole
d) Erythromycin
e) Aspirin
Ans is B
.503 You are doing endo treatment for a diabetic patient. At which blood glucose
level you defer your treatment?
A. >7 mmol
B. >15MOL or <5mmol
C. <6mmol
D. >6 MOL
E. >20 MOL
Ans A
504IOPA showing nerve close to the third molar, What would be the next Xray you’ll
order?
A. Panoramic
B. Periapical
C. Bitewing
D. Occlusal
Ans B
BY DR.ABDULRAHMAN ALMUALM
MJDF MCQS WITH ANSWERS
e) Is an topical antibiotics
ans is A : periochip.is consist of gelatin within it chlorhexidine gluconate 2.5mg
508Which one of the following is associated with the greatest increase in risk
of periodontal disease?
a) Malocclusion
b) Pregnancy
c) Poor diet
d) Type 1 diabetes mellitus
Ans d
BY DR.ABDULRAHMAN ALMUALM
MJDF MCQS WITH ANSWERS
517year-25 old man attends your clinic who has not been seen by a dentist
for the last 10 years. He is complaining of mobile upper central incisors.
These have probing depths of 9 mm and are bleeding on probing. There is
bleeding on probing in all other sextants but no probing depths above 3mm.
Which one of the following is the most likely diagnosis?
a) Juvenile periodontitis
b) Refractory periodontitis
BY DR.ABDULRAHMAN ALMUALM
MJDF MCQS WITH ANSWERS
521Which one of the following is the correct pressure applied when using the
BPE?
a) 5g
b) 10-15g
c) 20-25g
d) 30g
e) 35g
Ans C
.522
●When performing a Basic Periodontal Examination (BPE) for a new patient,
you record the following scores:
3/1/3
3/1/3
● As this is a new patient, you then decide that radiographs are indicated.
Which are the most appropriate radiographs to prescribe for this patient?
a) Bitewings (horizontal or vertical)
BY DR.ABDULRAHMAN ALMUALM
MJDF MCQS WITH ANSWERS
523Which one of the following associated with dentifrices does not offer
advantages in controlling gingivitis?
a) Amine fluoride/stannous fluoride
b) Chlorhexidine
c) Fluoride
d) Stannous fluoride
e) Triclosan
Ans c
a) Diabetic patients who control their condition by diet alone are still likely to
develop advanced periodontal destruction
b) Poorly controlled diabetic patients experience a similar degree of periodontal
destruction to non-diabetic patients
c) Responsiveness to periodontal treatment is not correlated with overall diabetic
control
d) There is no difference in neutrophil function between diabetic and non-
diabetic patients
e) There is evidence to suggest the existence of a bidirectional association
between diabetes mellitus and periodontal disease (ie where glycaemic control
can affect periodontal destruction, and the level of periodontal inflammation can
affect glycaemic control)
ans E
525Which one of the following is classed as a code 2 on the Silness and Loe
plaque index?
a) Film of plaque visible only by removal on probe or by disclosing
b) Heavy accumulation of soft material filling the niche between the gingival
margin and the tooth surface; the interdental region is filled with debris
c) Moderate accumulation of plaque that can be seen with the naked eye
d) Soft debris covering more than one-third but not more than two-thirds of
the tooth surface e) Soft debris covering not more than one-third of the tooth
surface
Ans C
0 --> no plaque
1 --> thin film of plaque that cannot be visible by naked eyes and can be visible by
disclosing agents.
2 --> moderate accummulation.
3 --> heavy
BY DR.ABDULRAHMAN ALMUALM
MJDF MCQS WITH ANSWERS
BY DR.ABDULRAHMAN ALMUALM
MJDF MCQS WITH ANSWERS
531A 62-year-old diabetic man who was started on diclofenac for post-
operative pain develops abdominal pain and dark stools. On examination, he
has melaena. By what mechanism has this drug caused his symptoms?
a) Type-1 hypersensitivity reaction
b) Cytotoxicity
c) Induction of cyclo-oxygenase pathway
d) Inhibition of PGE2 and PGI2 action of
mucosae
e) Induction of thromboxane
Ans is D
BY DR.ABDULRAHMAN ALMUALM
MJDF MCQS WITH ANSWERS
a) Aspirin 300 mg
b) Aspirin 75 mg
c) Simvastatin 40 mg
d) Warfarin 5 mg
e) Dipyridamole 200 mg
Ans A ||MONA = Morphine, Oxygen ,nitroglycerin, Aspirin
□535Oral steroids are often used in the treatment of chronic illnesses such as
chronic obstructive airways disease and rheumatological conditions. Which of
the following concurrent illnesses can be exacerbated by steroids?
a) Asthma
b) Diabetes
c) Psoriasis.
d) Inflammatory bowel disease
e) Systemic lupus erythaematous
ans is B steroids increase glucose level
537diabetic patient on your operating list for dental extraction under local
anaesthesia complains of feeling sweaty and unwell. He has been mistakenly
instructed to fast overnight and took his evening insulin as usual. You ask the
nurse to check the blood sugar, it reads as 1.8 mmol. If the patient was to
become unconscious, what is the most appropriate immediate pharmacological
option?
a) Hypostop
b) Administer a sugary drink
c) 10 ml of 50% dextrose solution
d) 1 mg glucagon iv
e) 1 mg glucagon im
ANS E (case if he was unconscious what will be the treatment )
BY DR.ABDULRAHMAN ALMUALM
MJDF MCQS WITH ANSWERS
538In the assessment of a patient with severe anaphylactic shock, which part
of assessment is most crucially affected by the administration of intramuscular
adrenaline?
a) Airway
b) Breathing
c) Circulation.
d) Disability
e) Exposure
ans is A
541You are taking a medical history from a 55-year-old woman who tells you
she is on ramipril. What class of drug is ramipril?
a) B-blocker
b) Thiazide diuretic
c) Loop diuretic
d) Angiotensin II blocker
e) Angiotensin-converting enzyme (ACE) inhibitor
Ans E
BY DR.ABDULRAHMAN ALMUALM
MJDF MCQS WITH ANSWERS
b) Amoxicillin
c) Ibuprofen
d) Captopril
e) Paracetamol
Ans C
549cleido-cranial dysplasia is not associated with [in the head and neck]:
a) Supernumerary teeth
b) delayed closure of fontanels-hypodontia
c)prominent clavicles
d) posing of forehead
Ans C
BY DR.ABDULRAHMAN ALMUALM
MJDF MCQS WITH ANSWERS
Ans A
554The use of latex gloves does has the following effect when a polyvinyl
siloxaneimpression is taken
a. retards the set of the impression material
b. enhances the set of the impression material
c. results in porosities in the impression material
d. latex gloves stick to the polyvinyl siloxane impression material
ans A
BY DR.ABDULRAHMAN ALMUALM
MJDF MCQS WITH ANSWERS
556Afteryou had finished scrubbing, glove and other PPE, you accidentlytouch the
chair. What
would bethe appropriate management?
1. Remove glove, wash again and put glove on
2. Remove glove but no need to wash because didn’t touch the
patient yet
3. Remove all PPE, wash again and wear PPE again
4. Remove glove, alcohol gel and glove on
5. Carry out the procedure because it isn’t infectious
Ans is 1 / A
55821 yrs old female patient who give you the history of
autoimmune disease which turn out to be
Pernicious anemia. What you think would be primarily deficiency?
.1 Intrinsic factor
.2 Iron
.3 Vitamin B12
.4 Folate
.5 Vitamin D
Ans 1
Ans 1
BY DR.ABDULRAHMAN ALMUALM
MJDF MCQS WITH ANSWERS
.2Herpes stomatitis
.3Herpangina
.4Hand foot & mouth disease
Ans is 3
Ans is 2
Ans is 5
BY DR.ABDULRAHMAN ALMUALM
MJDF MCQS WITH ANSWERS
564. An autistic pt comes to your clinic for follow up dental treatment, on his previous
dental visit another doctor prescribed fissure sealant, mother is nervous and called
the clinic asking about the procedure for her autistic son. What would you provide?
• A) non invasive treatment
• B) prophylactic treatment
• C )filling
• D) familiarization visit
Ans D
565A 58 year old female patient complain of the pain in TMJ area, OPG shows flat
condylar procecus, pain is more intense in the morning, no muscle tenderness, what
would be diagnosis?
• A) osteoporosis
• B)reumathoid artritis
• C) ostheoartritis
• D )anteriror disc displacement
• E)posterior disc displacement
Ans c
566. Reason for failure of canine eruption without early loss of deciduous teeth?
• A) supernumary
• B) ectopic
• C) crypt
• D)supplementary
• E) primary failure of eruption
Ans B
BY DR.ABDULRAHMAN ALMUALM
MJDF MCQS WITH ANSWERS
570PA radiograph was given which was not that good but
acceptable. According to guideline which
one of the following best describe
this situation.
1. 10% cone cutting
2. 10% elongation
3. 20% elongation
4. 30% elongation
5.40% cone cutting
Ans 1
571Which of the following has highest five year survival rate for class V restoration?
• A. Amalgam
• B. Bonded amalgam
• C. Composite (flowable)
• D. Conventional GIC
• E. Resin modified GIC
And e
57321 yrs old male patient who give family history of X-linked
amelogenesis Imperfecta of his
father. What is the likely chance of him having this disease?
1.25%
2.75%
3.50%
4.0%
5.100%
Ans 4
574
Which of the following is NOT tax deductible expenses?
• A. Conference fee
• B. Critical illness cover
• C. GDC annual retention fee
• D. Indemnity/ insurance
• E. Surgery clothing
BY DR.ABDULRAHMAN ALMUALM
MJDF MCQS WITH ANSWERS
Ans B
577What is the minimum Factor VIII level required to avoid significant post extraction
haemorrhage in Haemophilic patient?
• A. 30%
• B. 50%
• C. 60%
• D. 70%
• E. 75%
Ans is A
578What is the value of gift that must be recorded and kept at dental practice?
• A. All gifts regardless of value
• B. No obligation to record
• C.>20GDP
• D.>50GDP
• E. >100 GDP
Ans is E
580Patient has generalized probing 5-8mm, bone loss 30%-80%. What is the stage
and grade of his periodontitis?
• Stage III Grade B
• Stage III Grade C
BY DR.ABDULRAHMAN ALMUALM
MJDF MCQS WITH ANSWERS
• Stage IV Grade A
• Stage IV Grade B
• Stage IV Grade C
Ans is E (stage lV grade c )
583
28 yrs old female patient comes with pain in lower jaw
and while you took x-ray, finds
fractured mandible. Then she tells that her partner hits her
when he drunk. She has capacity.
What would you do?
1. Report to police
2. Safeguarding to social service
3. Respect her confidentiality
4. Inform GP
5. Advise her to consult
Ans is 2
If the patient has the capacity --> adviser to consult safeguarding social services.
If the patient has no capacity (Minor, <16 years old) --> consult safeguarding social
services after taking consent from the patient's parents. If you suspect that the child's
life might in danger or sexual abuse then you have to contact safeguarding social
services immediately.
Ans A
BY DR.ABDULRAHMAN ALMUALM
MJDF MCQS WITH ANSWERS
Ans A
590) Maximum Percentage of NO come out from Machine during Sedation is:
a) 10%
b) 40%
c) 50%
d) 30%
e) 70%
ans is e
BY DR.ABDULRAHMAN ALMUALM
MJDF MCQS WITH ANSWERS
595. While you were trying to remove UR6, one of the roots of UR6 fractured. You
were trying to remove
the retained root, however it suddenly disappears and was displaced into the
maxillary sinus. What is
your first management?
a. Antibiotics and review
b. Refer to the oral surgeon department
c. Attempt removal
Ans B ??
596. (An X ray showing a UL8 being displaced into the maxillary sinus)
While you are performing a UL8 removal, the tooth was displaced and went into the
maxillary sinus. And
a Posterior anterior radiograph was taken. Which x ray would you take next?
a. CBCT
b. CT
c. Upper occlusal
d. Periapical
e. OPG
BY DR.ABDULRAHMAN ALMUALM
MJDF MCQS WITH ANSWERS
Ans D
BY DR.ABDULRAHMAN ALMUALM
MJDF MCQS WITH ANSWERS
BY DR.ABDULRAHMAN ALMUALM
MJDF MCQS WITH ANSWERS
BY DR.ABDULRAHMAN ALMUALM
MJDF MCQS WITH ANSWERS
601. Cervical lines and dark lines on central incisors are more likely from:
• A) childhood illness
• B) amelogenesis
• C)dentinogenesis
• D )flourosisi
Ans is A
603. Q10. Stainless steel crown placed on 7 yr old 2 months after MO GIC filling LLE
What NHS band is treatment
A.1
b.2
C. 3
d. urgent
ans B (no lab work needed )
a. 6
b. 12
C. 18
BY DR.ABDULRAHMAN ALMUALM
MJDF MCQS WITH ANSWERS
d. 24
e. 36 montHs
Ans B
605. Pt attends with non bacterial TsL he is taking medication -which is likely meds
a. cimetidine
b. omeprazole
C. aspirin
d. insulin
ans B
a. Articane
b. Lignocaine
C. Bupivocaine
d. Prilocaine
ans A
Diphtheria
HepatitisA
C. Hepatitis B
D. Measles
Ans is A (diphtheria)
609. A fifteen year old patient asks for bleaching of teeth. What is the first line of
management?
Discuss options with both patient and parent
Discuss option with both patient and parent, once you have obtained consent from
patient
Discuss only with patient
Discuss only with parent
Ans is B
BY DR.ABDULRAHMAN ALMUALM
MJDF MCQS WITH ANSWERS
613 Aortic root defect,eye problems , high palate suggest what condition
Ans Marfan syndrome
615. 8 year old, all first permanent molars carious, which toothpaste you prescribe
Ans is 1450 -1500
BY DR.ABDULRAHMAN ALMUALM
MJDF MCQS WITH ANSWERS
.1nasopalatine cyst
2 stafne bone cyst
3residual cyst
4dentigerous cyst
5odotnogenic Keratocyst tumor
6infected mandibular cyst
7simple bone cyst
8periodontal cyst
9preapical cyst
Ans is 5
A Record dental charting and oral tissue assessment carried out by other registrants
B Process dental radiographs
BY DR.ABDULRAHMAN ALMUALM
MJDF MCQS WITH ANSWERS
Ans F
618.Which of the following protocols must be included in the written practice
protocols'?
A Disposal of hazardous waste
B Disposal of sharps
C Annual leave entitlements
D Radiation protection
E Autoclaving
F A+B+D+E ( Ans is E)
619. In juvenile periodontitis, which bacterium is the recognised aetiological agent=
A Porphyromonas gingivalis
B Prevotella intermedia
C Actinobacillus actinomycetemcomitans
D Borrelia burgdorferi
E Fusobacterium intermedium
Ans C
.621 Six months ago you saw a child patient, then aged 9 years. His upper
right maxillary canine was palpable in the labial sulcus but the upper left was
not. The situation is now unchanged, so you have taken two periapicals of the
non-palpable tooth. They both show that there is some resorption of the CI root
but the permanent canine appears somewhat mesially angled and is more
mesial on the more mesially positioned film. What is your the best course of
action?
A● Keep a careful watch on it and take another x-ray in 6 months.
b● Refer to an oral surgeon for early exposure of the permanent canine.
C● Refer to an oral surgeon for early removal of the permanent canine before it
damages the lateral incisor.
D ● Refer to an orthodontist for a treatment plan.
E● Wait and watch' until the child is 11.
Ans is D
BY DR.ABDULRAHMAN ALMUALM
MJDF MCQS WITH ANSWERS
A)enamel pearl
B)root groove
C)overhang restoration
623A child of 5 years attends with pain from a grossly decayed lower right D
which has a discharging sinus. He is a hemophiliac. Which of the following is the
most appropriate treatment to relieve his pain?
● Antibiotics
● Extraction
● Fluoride application
● Non-vital pulpotomy
● Vital pulpotomy
Ans D
.624 48 year old woman complains of a sore area on the right buccal mucosa
adjacent to a restored tooth. The lesion has a lichenoid appearance and this is
confirmed histopathologically following a biopsy. Which of the following
restorative materials is most frequently associated with *lichenoid* changes.?
● Gold
● Amalgam
● Porcelain
● composite
● Glass ionomer cement
Ans is B (Amalgam)
.
.625 Which of the following about acrylic is true:
A) can be stained and designed
B) has higher strength compared to Co-Cr
C) has less hypersensitivity than Co-Cr
D) less surface area required for tissue cOverage
Ans is A
.626 Unilateral rush on one side of the face, pt is feeling well what is the most
common features you
will find in the mouth?
A) vesicular rash on the palate
B) ulcers on border side of the tongue
C) something on the cheek
Ans is B
BY DR.ABDULRAHMAN ALMUALM
MJDF MCQS WITH ANSWERS
.631 12 yr old with missing UR 1st permanent molar. What is the most right
diagnosis
(A.hypodontia
B macrodontia
C failure of primary eruption
D, early tooth loss)
Ans A
632 15year old with retained upper primary canine . contralateral permanent canine
is erupted 1.5 years ago. Reason is ?
(A.hypodontia,
b.macrodontia,
c.early tooth loss)
D.ectopic canine
Ans is D ( please note this Qs was adjusted, it was without D option ,I added it
because it’s more logical answer , hypodontia in permanent canine is very rare /but if
you faced the Qs without D option the most close answer will be A )
633. 16 year old boy with missing UR2 . What is your diagnosis? (
A.ectopic,
B. hypodontia,
c. primary failure in eruption,
D.deciduous tooth loss
Ans here is B
BY DR.ABDULRAHMAN ALMUALM
MJDF MCQS WITH ANSWERS
634. Appliance you will give to 12 yr old boy who complains his upper teeth are too
outward and children in school call him boy with no chin
A(fixed appliance,
B.functional appliance,
C.headgear)
Ans here B
.635 The professional conduct committee can suspend the dentist registration for a
period not exceeding 12 months, or erase his name?
True
False
Ans True
.636 Which one of the following statements regarding the platform switching concept
in Implent
dentistry is FALSE?
Ans A
.637 After prescribing a dose of post-operative antibiotics, you are called to the ward
to review the
patient who Is dyspnoeic with audible wheeze, has facial swelling and ls covered in a
red rash. Her
pulse is 120/min regular and blood pressure is 86/40 mmHg. Oxygen saturations on
151 of oxygen
are 100%. You quickly diagnose anaphylactic shock. Which is the most important
immediate
treatment?
BY DR.ABDULRAHMAN ALMUALM
MJDF MCQS WITH ANSWERS
.638 Which one of the following has the best 10-year survival rate, when replacinga
single upper centra
incisor?
.639 Which one of the following does not cause a unitateral lower motor neurone
palsy of the facial
nerve?
oGuillin-Barré syndrome
o Diabetes
o Ramsey Hunt Syndrome
oBell's palsy
o Acoustic neuroma
Ans is A
Ans is C?
.641 The official name of a drug is generally used inter-changeably with the:
(a) proprietary name
(b) chemical name
(c) brand name
(d) trade name
(e) generic name
Ans is E
BY DR.ABDULRAHMAN ALMUALM
MJDF MCQS WITH ANSWERS
645 The time until the patient notes the therapeutic ef-fect of an administered drug is
termed its:
(a) onset of action
(b) duration of action
(c) pharmacological action
(d) latency
(e) individual variation
Ans is D
latency
From the administration of drug to the onsent of action
.647 A patient exhibits very little effect following ad-ministration of a relatively large
dose of a pharmaco-logically active drug. This is best described as:
(a) idiosyncrasy
(b) high therapeutic index
BY DR.ABDULRAHMAN ALMUALM
MJDF MCQS WITH ANSWERS
(c) hyporeactlon
(d) hyperreaction
(e dependence
Ans C
.648 Administering double the usual dosage of a drug to a patient would most likely
double the: f
a) therapeutic effectiveness
b) mcidence of side effects
c) therapeutic index l.
d) potential for toxicity (
e duration of action
ans is D
.649 A drug that has an affinity for a receptor site and c-roduces the intrinsic activity
of the receptor cell or e1zyme system is known as: (a) a) an agonist
(b) a competitive antagonist
(c) a noncompetitive antagonist
(d) any of the above (
e) none of the above
Ans A
.652 Drugs are best absorbed when they are: Choose from (a). (b), (c), (d), and (e).
(a) 1 and 2.
BY DR.ABDULRAHMAN ALMUALM
MJDF MCQS WITH ANSWERS
(b) 2 and 3
(c) 2 and 4
(d) 3 and 4
(e) 3 and 5
Ans is E
# Drugs best absorbed when they are low molecular weight ,unionized, and lipid
soluble
.655 After a drug has been absorbed, an equilibrium is usually established at all of
the following possible sites of distribution except· is:
(a) nephrons
(b) hepatic microsomal enzymes
(c) urine
(d) plasma and ttssue fluids
(e) fat binding and protein binding sites
Ans is C
.656 A drug that becomes plasma-protein bound ts: (a) available for
biotransformation
(b) easily excreted
(c) more readily transported across cell membranes
(d) pharmacologically active
(e) pharmacologically inactive
Ans is E
.657 Trichloroacetic acid, a strong acid, has been used by dentists for chemical
cautery of hypertrophic tissue and of aphthous ulcers. Its mechanism of action is:
(a) thermodynamic action
(b) activatton of tissue enzymes
BY DR.ABDULRAHMAN ALMUALM
MJDF MCQS WITH ANSWERS
658 Full-denture patients with inflammation of the en-tire tissue-bearing area should
probably be treated with:
(a) neomycin
(b) polymyxin B
(c) amphotericin B
(d) ampicillin
(e) chloramphenicol
Ans is C
659 Permanent auditory nerve deafness is frequently associated with use of:
(a) the streptomycins
(b) chloramphenicol
(c) the penicillins
(d) the tetracyclines
(e) polymyxin B
Ans is A
662 A protective mechanism of the dental pulp to external irritation or caries is the
formation of
A. pulp stones.
B. tertiary dentin.
C. secondary cementum.
D. primary dentin.
BY DR.ABDULRAHMAN ALMUALM
MJDF MCQS WITH ANSWERS
Ans is B
Ans A
Behavioral management :
Remember Techniques for behaviour
management
Tell, show, do Self-explanatory, but use
language the child will understand.
Desensitization Used for child with pre-
existing fears or phobias. Involves helping
patient to relaxin dental environment, then constructing a hierarchy of fearful
stimuli for that patient. These areintroduced to the child gradually, with
progression on to the next stimulus only when the child isable to cope with
previous situation.
Modelling Useful for children with little previous dental experience who are
apprehensive. Encourage child to watch other children of similar age or
siblings receiving dental Rx happily.
BY DR.ABDULRAHMAN ALMUALM
MJDF MCQS WITH ANSWERS
Behaviour shaping The aim of this is to guide and modify the child's
responses, selectively re-inforcing appropriate behaviour, whilst
discouraging/ignoring inappropriate behaviour.
.668 A boy 14 years old came to ur clinic with avulsed tooth not accompanied
by parents, most appropriate in this scenario ?
1- carry out treatment
2- ask the boy to place tooth in storage media
3- contact parents
Ans is 1
Ans A
#to remember :
# Cephalometrics Most commonly used cephalometric points :
S = Sella: mid-point of sella turcica.
N = Nasion: most anterior point on fronto-nasal suture.
Or = Orbitale: most inferior anterior point on margin of orbit (take average of
twoimages).
Po = Porion: uppermost outermost point on bony external auditory meatus.
.670 The light emitted by the polymerization lamp has to be checked from
time to time. The meter used for this only measures light in the range of:
A. 100-199 nm
B. 200-299 nm
C. 300-399 nm
D. 400-499 nm
BY DR.ABDULRAHMAN ALMUALM
MJDF MCQS WITH ANSWERS
Ans D
Ans 2
673 Trauma to tooth with closed apex. In which one the tooth is more likely
to maintain vitality?
Concussion
Subluxation
Intrusive luxation
Avulsion Luxation
Ans is A
674 Which odontogenic pathology would contain clear straw coloured fluid?
Options are
1. Odontomes
2. Solitary bone cyst
3. Ameloblastoma
4. Dentigerous cyst
5. Keratocyst.
Ans is 2
675 Which of the following types of tooth movement can be achieved with a
removable appliance?
a) Bodily movement
b) Correction of rotation
c) Rapid maxillary expansion
d) Tipping
e) Torque
Ans is D
BY DR.ABDULRAHMAN ALMUALM
MJDF MCQS WITH ANSWERS
677 Which active component is commonly used to move a tooth mesio-distally within
the arch?
a) Palatal finger spring
b) T spring
c) Robert’s retractor
d) Z spring
e) Labial bow
Ans A
678Which appliance would you use to treat an 8-year-old child with an anterior
crossbite associated with an upper central incisor?
a) URA with anterior bite plane and a midline screw
b) URA with a midline screw and a T spring
c) URA with posterior bite planes and a midline screw
d) URA with an anterior bite plane and a T spring
e) URA with posterior bite planes and a T spring
Ans e
.679 Which of the following statements about rapid maxillary expansion is true?
a) It can be used in the mandible
b) It can only be carried out in adults
c) It can only be carried out in the deciduous dentition in It involves expansion of the
mandibular symphysis
d) It is a form of distraction osteogenesis
ans is D
680 Which of the following features are most commonly associated with a digit
sucking habit?
a) An increased overjet with a deep Overbite and no crossbites
b) A Unilateral Crossbites with a decreased overjet and a deep overbite
c) An increased overjet with a deep Overbite and a unilateral crossbite
d) A Unilateral Crossbites with a reduced Overjet and an incomplete bite
e) A unilateral crossbite with an increased overjet and an incomplete overbite
Ans is E
681 Which of the following cases is an ideal case for functional appliance therapy?
BY DR.ABDULRAHMAN ALMUALM
MJDF MCQS WITH ANSWERS
Ans A
683 Fixed appliances can achieve tooth movement in all three spatial planes.
What type of tooth movement is achieved with a second order bend?
a) Bucco-lingual movement of the root apex to correct the inclination of teeth
b) Vertical movement of the crown to correct discrepancies in crown height
c) Bucco-lingual movement of the crown to correct the inclination of teeth
d) Movement in the horizontal plane to account for differences in bucco-lingual
thickness of teeth
e) Movement in the vertical plane to correct the mesio-distal angulation of
teeth
Ans is E
684 An adult patient complains of her prominent upper front teeth and receding chin.
She says she did not get 'braces' when she was younger because the family had to
move area a lot. Her oral care and health is good, and she has a 9 mm overjet.
Where is the most suitable place to refer her? "
a) A GDP friend who has a special interest in orthodontics and has been trained to
use fixed appliances.
b) A specialist practitioner who uses a lot of functional appliances.
c) A private specialist practitioner, because she's too old to get NHS treatment now.
d) A hospital consultant, as it is likely she will need surgery now to correct her
problem
e) The nearest dental hospital, although it is 70 miles aaway
Ans D?
685 pt. presents with an increased overjet with 7 mm, an anterior open bite of 5 mm,
and a lower left second permanent molar partially erupted & impacted against the
first perm. Molar. Which IOTN category? a) None
b) Little need
c) Moderate need
d) Great need
e) Very great need
Ans c
BY DR.ABDULRAHMAN ALMUALM
MJDF MCQS WITH ANSWERS
Ans is A
. 690 Which sagittal relationship described below is not represented via a lateral
cephalogram?
a) Jaws to the skull base
b) Maxilla to mandible
c) Teeth to each jaw
d) Soft tissues to hard tissues
e) Teeth in the maxilla to the teeth in the mandible
D?
691 An orthodontic patient requires imaging for treatment planning, which radiograph
listed below is standardized and reproducible for all patients? a) Dental
Orthopantomogram (OPG)
b) Lateral cephalogram
c) Lower standard occIusaI
d) Upper standard occIusaI
e) Lateral skull radiograph
Ans A??
692 You are meeting a three year old patient for the first time. His mother explains
that he had a unilateral cleft lip and palate that was repaired in infancy. She has
heard that orthodontics will usually be required when he is older. She asks you what
is the commonest orthodontic problem that occurs with a repaired cleft palate. What
is the most appropriate answer?
a) A contracted maxillary arc
b) An anterior open bite
c) An increased overjet
d) An expanded arch
Ans A
BY DR.ABDULRAHMAN ALMUALM
MJDF MCQS WITH ANSWERS
696 What is the prevalence of non-syndromic cleft lip and palate in the UK?
a) 1 in 20
b) 1 in 300
c) 1 in 400
d) 1 in 700
e) 1 in 1000
697. Which one of the following is the correct incidence of cleft lip in the UK?
a) 1 in 200
b) 1 in 500
c) 1 in 900
d) 1 in 1800
e) 1 in 2500
Ans is B?? Its 1/750
698 Which one of the following is the classical extraction pattern for the serial
extraction technique?
a) Deciduous canines, first premolars then first deciduous molars
b) Deciduous canines, first permanent molars then first premolars
c) Deciduous canines, second deciduous molars then second premolars
d) Deciduous canines, second molars then second deciduous molars
e) Deciduous canines, first deciduous molars then first premolars
Ans E
699 nine year old boy presents with a class I occlusion with no crowding or overjet
with a grossly carious upper left 6 which is not suitable for restoration. The upper left
7 is very near to eruption. Upper right 6, lower right 6 and lower left 6 are sound and
fissure sealed. The oral hygiene is good. What is the most appropriate extraction
pattern for this patient?
BY DR.ABDULRAHMAN ALMUALM
MJDF MCQS WITH ANSWERS
700 n facial growth, at the age of 5 years, what percentage of growth has been
completed?
.701 Which of the following statements about the Leeway space is correct?
702 8-year-old patient attends with a painless lower right 1st permanent molar which
requires extraction. The patient has a Class I occlusion with no crowding. What
feature will determine the ideal time to extract the tooth to maximize space closure?
703 Early loss of which upper teeth is the most likely cause of impacted upper
second premolar teeth?
a) Both first and second deciduous molars
b) First permanent molars
c) Second deciduous molars
d) First deciduous molars
e) Both first and second permanent molars
Ans C
BY DR.ABDULRAHMAN ALMUALM
MJDF MCQS WITH ANSWERS
a) The mesiobuccal cusp of the upper first permanent molar occludes with the
mesiobuccal groove of the Iower first permanent molar
b) The distobuccal cusp of the upper first permanent molar occludes with the
mesiobuccal groove of the Iower first permanent molar
c) The mesiobuccal cusp of the upper first permanent molar occludes with the
distobuccal groove of the Iower first permanent molar
d) The mesiobuccal cusp of the upper first permanent molar occludes with the
distobuccal cusp of the Iower first permanent molar
e) The mesiobuccal grove of the upper first permanent molar occludes with the
distobuccal cusp of the Iower first permanent molar
Ans is B
706 year old boy has previously had all primary molars restored and a pulpotomy on
upper right E. He has an early mixed dentition with lower lateral incisors erupting.
There is a midline diastema of 2 mm. The upper right E has become symptomatic
and requires extraction. The most likely long term effect of the extraction on the
occlusion is:
a) Early eruption of the second premolar.
b) Loss of upper central line.
c) No significant effect.
d) Overeruption of the lower right teeth.
e) Potential crowding in the upper right quadrant
Ans e
BY DR.ABDULRAHMAN ALMUALM
MJDF MCQS WITH ANSWERS
Ans A
AnS
1 C
2 B
3 E
4 A
5 F
711 What teeth should a 9 year old child have a given quadrant?
a) 12CDE6
b) 1BCDE6
c) 12CDE
d) ABCDE6
e) 1234E6
Ans A
BY DR.ABDULRAHMAN ALMUALM
MJDF MCQS WITH ANSWERS
e) 16 y
Ans D
AnsC
a) in All children under the age of 5 years should use toothpaste with a fluoride
concentration of 500 ppm
b) Fluoride supplements are recommended for all children
c) It exerts its ant cariogenic effects mainly pre eruptively
d) It is more effective in preventing pit and fissure caries than in preventing caries in
smooth surfaces
e) The certain lethal dosage is 32—-64 mg/kg
ans is E
BY DR.ABDULRAHMAN ALMUALM
MJDF MCQS WITH ANSWERS
718 Which one of the following relates to saliva and dental caries?
a) The buffering power of saliva depends mainly on its bicarbonate content and is
increased at high rates of flow
b) The buffering power of saliva is independent of the bicarbonate content
c) The buffering power of saliva depends on mainly its bicarbonate content and is
decreased at low rates of flow
d) The buffering power of saliva depends on its IgA content and is increased at high
rates of flow
e) The buffering powers of saliva depends on its IgA content and is decreased at
high rates of flow
Ans A
719 Which one of the following is the most common cause of dental pain and loss of
teeth, especially in the younger population?
a) Fracture of a crown/cusp
b) Thermal or chemical irritation
c) Dental caries
d) Traumatic pulpal exposure
e) ‘Cracked tooth syndrome’
Ans C?
720 Which one of the following is not a factor in the etiology of dental caries?
a) Diet
b) Oral flora
c) Age
d) Time
e) Susceptible surface
Ans C
721 Which one of the following inter-dental aids is most effective at removing inter-
dental plaque?
a) Dental floss
b) Electrical toothbrushes
c) Inter—dental brushes
d) Triangular toothpicks
e) Dental tape
ans C (pas)
722 18 months of age, what is fluoride supplement for child living in 0.25 ppm
fluoridated water?
a) 0.25mg
b) 0.5 mg
c) 1 mg
Ans A
BY DR.ABDULRAHMAN ALMUALM
MJDF MCQS WITH ANSWERS
723 There has been much debate regarding the effectiveness of fluoride in water for
preventing tooth decay. A systematic review of the evidence drew conclusions as to
the reductions in decay which can be expected. Fluoridation of public water supplies:
724 Epidemiological studies have shown that dental decay is normally greater
amongst northerners and those in socially deprived circumstances. Which of the
following would be most effective in reducing caries in a high risk population?
726 3 year old with fluoride concentration of 0.3 ppm fluoridated water, what is the
fluoride supplement dose given
a) 0.25 mg
b) 0.5mg
c) 1mg
Ans B?
728 Child accidentally consumed 5mg/kg fluoride what the first line treatment?
a) Give child salty drink
b) Give child sugary drink
c) Give child milk
ans C
BY DR.ABDULRAHMAN ALMUALM
MJDF MCQS WITH ANSWERS
729 Mother of a 3 years old child came to your clinic asking for advice on what do to
after brushing her child’s teeth , you would tell her :
a) Spit out after brushing and rinse
b) Spit out after brushing and don’t rinse
c) Spit out and wait for 10 minutes then rinse
d) Spit out and wait 30 minutes then rinse
Ans B
730Patient needs a minimal preparation metal crown for his LL6 the best cement is
a) GIC
b) zinc phosphates
c)RMGIC
d) zinc polycarboxylate cement
e) resin cement
ans is E
732What is the type of impression used for recording of upp dentur pa well fit stock
tray
for a patient having a hypersensitive gag reflex
a) Alginate
b Zo/E
c) Addition silicon
d) Polyether
e) Polysulphide
Ans is C
733What is the best type of restoratiom. storing buccal root caries in the lower
mandibular molar in patient:
a) Glass ionom
b) Amalgam
c) Composite
d) porcelain veneer crown
Ans A
734. 5)patient with compelete denture complaining difficulty in F &V sounds most
probably:
a) Lack on retention
b )palatal valut too high
c.high behind incisors
BY DR.ABDULRAHMAN ALMUALM
MJDF MCQS WITH ANSWERS
736The liquid which is present in the adhesive dentin systems to wet the dentin is:
a) Water
b) Acetone
c) Polymers liquid
d) Ketones
Ans B
737During the try-in stage of complete Denture you need to do slight grinding of
back teeth
Which teeth cusps should be reduced
a) Reduce teeth randomly
b) Upper buccal and lower lingual
c)Upper palatal and Jower buccal
d) Upper palatal and lower lingual
e) Upper buccal and lower buccal
Ans B
738. the obturation technique which gives the best hermetic seal for gutta-percha in
endodontitreatment
a)warm lateral compaction
b)warm verticalcompaction
c)thermoplasticised gutta-percha technique
d) coldlateral coropaction
e)chlorpercha technique
Ans is C
739The use of latex gloves does has the following effect when a polyvinyl siloxane
impression is taken
a) Retard the set of the impression material
b) Enhance the set of the impression material
c)Result in porosities in the impression material
d) Has no effect on impression
e) Latex gloves stick to the polyvinyl siloxane impression material
Ans A
BY DR.ABDULRAHMAN ALMUALM
MJDF MCQS WITH ANSWERS
a) 1 month
b) 3 months
c) 6 months
d) 1 year
e) Only if become symptomatic
Ans is D
743. First line treatment for patient with NCTL in palatal surfaces of upper front teeth:
a) Direct Composite palatal veneers
b) Full coverage crowns
c)Gold veneers
d) Indirect composite palatal veneers
e) Indirect porcelain veneers
ans A
Ans is D
BY DR.ABDULRAHMAN ALMUALM
MJDF MCQS WITH ANSWERS
Ans D
748Child pt who has severely broken down of all four permanent molars, they
have poor prognosis so you have decided to remove them. What would you
check to determine the right time for extraction? •
BY DR.ABDULRAHMAN ALMUALM
MJDF MCQS WITH ANSWERS
Ans D
757.Severe case of Down Sy require surgical removal of one tooth, who can
give a consent ?
Before 16 > parents
After 16 > no one only patient according to his mental.capacity to understand retain
and communicate or (best interest of the patient )
Ans is A
BY DR.ABDULRAHMAN ALMUALM
MJDF MCQS WITH ANSWERS
759Pt are compaling about the jaw pain two weeks after RCT, what is gonna
be your first management •
A) do RCT again •
B) reassure , sof diet and review in two weeks time •
C) soft splint •.
D) remove the tooth
Ans is B
Ans us B
BY DR.ABDULRAHMAN ALMUALM
MJDF MCQS WITH ANSWERS
766. Pt comes to your practice and on OPG you have noticed fracture of
angle of mandible, then she said she was hit by her husband. Pt has
capacity, how to manage this situation/ •
A) report to the police •
B) suggested her to report case to the police •
C) contact local safeguarding services •
D) ignore and start treating injuries
Ans C
767. What is the most likely infection that can be caught by NSI in fully
immunized person? •
A) Hep B •
B) Hep C •
C) HIV
Ans is B
768. Patient had BMI=41. Which of the following is his ASA classification? •
A) ASA I •
B) ASA II •.
C) ASA III •
D) ASA IV
Ans C
769. A double blind RCT was carried out to study xxxx at University. In this
case who is not informed of which group were given new drugs and which
group is given placebo? •
A) nurse only •
B) dentist only
C) participants and dentist •
D) participants only
Ans C
BY DR.ABDULRAHMAN ALMUALM
MJDF MCQS WITH ANSWERS
771. PICTURE OF LEAF FIBROMA and question what is the most likely
diagnosis? ( did not
rememeber was it leaf or
fibroepithelial fibroma)
Ans B
773. BPE score for the lower anterior teeth with pocket depth of 3mm with
recession of 3mm? •
A) 0
• B) 1 •.
C) 2 •
D) 3 •
E) 4
Ans is A
BY DR.ABDULRAHMAN ALMUALM
MJDF MCQS WITH ANSWERS
774Your associate has just found that he missed GDC renewal and not paid
GDC fee. What is your next action as hid employer? •
A) suspend •
B) no action, it is associate’s problem •
C) inform the GDC
Ans is A ?
775. . A 83 year old pt with dementia comes to your practice with discharge
pus from parotide gland and does not have a family, also she does not have
elected power of attorney, who can give a consent for her? •
A ) her close relative •..
B) her good friend •
C ) independent mental capacity advocate •
D) her GP •
E) her caregiver
Ans is C
BY DR.ABDULRAHMAN ALMUALM
MJDF MCQS WITH ANSWERS
D) 150
Ans B
780You are going to employ clinical dental technician, which of the following
procedure you expected from him to do independently? •
781. Calculation for sensitivity, you will be given the number in form of true
positive =40, true negative=10, false positive=5, false negative=20 ect and
need to remember formula to get a result!
782. Pt has an anaphylactic attack during the dental procedure, what type of
hypersensitivity it is? • A) type I
BY DR.ABDULRAHMAN ALMUALM
MJDF MCQS WITH ANSWERS
• B) type II •..
C) type III •
D) type IV
Ans A
783. You have given one dose of Prilocaine with felipressin to the pt, but he
is complaining of pain. If we know that maximum dose is 300mg how many
of 2.2 ml ampules we can give to the same patient safety? •
A ) 2.5 •
B )5 •
C )7 •
D)11
Ans B
784. 11 year old girl with multiple caries, what would you advice to your
patient from following? •
A) use 5000ppm fluoride toothpaste •
B ) use 2800ppm F toothpaste •
C ) topical fluoride of 11600 ppm 2 or more times per year
785. 11 year old girl with multiple caries, what would you advice to your
patient from following? •
A) use 5000ppm fluoride toothpaste •
B ) use 2800ppm F toothpaste •
C ) topical fluoride of 11600 ppm 2 or more times per year
Ans B
786. years old girl comes with the mother, she has good hygiene and no
caries, living in area with fluoridated water, mother is asking about toothpaste
and your recommendation is: •
A) do not swallow toothpaste •
B ) use fluoride free toothpaste •
C) rinse with a water after brushing •
D) do not rinse after brushing, just spit all
out and go to the bed
Ans is D
Ans is A
BY DR.ABDULRAHMAN ALMUALM
MJDF MCQS WITH ANSWERS
791. male pt comes for regular dental check up, but OPG shows OKC and
he said that he is previously was diagnosed with BCC. What condition it is? •
A) Gardener syndrome •
B) Gorlin-Gotz Sy •
C) Apert sy •
D)Frey’s Syndrome •
E) Marfan Syndrome
Ans B
792. A 6 years old child comes to you with fever, poor appetite, feeling of
being unwell, sore throat. 4 days later fever starts, painful rash, blister develop
at the hand, feet and buttocks. What is the causative agent? •
A) Herpes simplex •
B) paramyxovirus •
C) Coxackie virus •.
D) HPV 16
Ans C
BY DR.ABDULRAHMAN ALMUALM
MJDF MCQS WITH ANSWERS
795. An autistic pt comes to your clinic for follow up dental treatment, on his
previous dental visit another doctor prescribed fissure sealant, mother is nervous
and called the clinic asking about the procedure for her autistic son. What
would you provide? •
796. A 80 year old pt with swelling, does not have capacity nor relative to give
a consent. What would be an appropriate management? •
Ans B
797. 58 year old female patient complain of the pain in TMJ area, OPG
shows flat condylar procecus, pain is more intense in the morning, no muscle
tenderness, what would be diagnosis? • .
A) osteoporosis •
B)reumathoid artritis •
C) ostheoartritis •
D )anteriror disc displacement •
E)posterior disc displacement
798. Reason for failure of canine eruption without early loss of deciduous
teeth? •
BY DR.ABDULRAHMAN ALMUALM
MJDF MCQS WITH ANSWERS
A) supernumary •
B) ectopic •
C) crypt •
D)supplementary •
E) primary failure of eruption
Ans is C
Ans is A
801. Picture of lower central incisors, fractured on the gum level, there is no
pathological changes in the bone but pt is on biposphonates for 5 years.. •
A)remove the teeth •
B)leave as it is •
C) provide an overdenture on the state how it is now •
D) provide an overdenture after you did RCT
Ans is D *
BY DR.ABDULRAHMAN ALMUALM
MJDF MCQS WITH ANSWERS
803. You plan to do double blinded Randomised controlled trial. Once you
have finished design the trial, what do you do next? •
A. Advertise in dental research journal for participants •
B. Allow patient to choose the treatment they wish to receive •
C. Ensure potential patient understand the method of treatment allocation •
D. Inform interested patient and get their consent •
E. Seek approval from appropriate research ethics committee
Ans is E
Ans is B
Ans is D
806. Which is best described suspected internal resorption for maxillary central
incisor that had trauma? •
A. Asymmetric, central lesion with discontinued canal • B. Asymmetric, peripheral
lesion with intact canal •
C. Round, central lesion with discontinued canal •
D. Round, central lesion with intact canal •
E. Round, peripheral lesion with discontinued canal
Ans C
BY DR.ABDULRAHMAN ALMUALM
MJDF MCQS WITH ANSWERS
Ans is E ?
808. What is the value of gift that must be recorded and kept at dental
practice? •
A. All gifts regardless of value •
B. No obligation to record •
C. > 20 GDP •
D. > 50 GDP •
E. >100 GDP
Ans is E
809. Which of the following acts by reversibly competing with vitamin K for
binding site with an enzyme essential for formation of clotting factors II, VII, IX,
X by liver? •
A. Aspirin •
B. Clopidogrel •
C. Dabigatran •
D. Heparin
• E. Wafarin
Ans is E
810.One more question about ALL but they gave figure of lymphocites,
leukocites, hemoglobin ect.. Make sure you know normal blood count range
figures….(ALL=ACUTE lymphoid leukaemia)
BY DR.ABDULRAHMAN ALMUALM
MJDF MCQS WITH ANSWERS
BY DR.ABDULRAHMAN ALMUALM
MJDF MCQS WITH ANSWERS
811. comes with the career and they have an verbal argue and you have
noticed that caeer were aggressive towards client, what would you do; •
A) ignore •
B) report to the police •
C) ask them politely to book next appointment •
D) report to local safeguarding services •
E) report to care home
Ans is D??
812. Placing canula for iv sedation, patient say pinky finger pain, what structure
injured??
#!
813. To ensure high quality of health care that is provided in line with standards the
NHS recommend:
a) clinical governance
b) clinical effectiveness
c) clinical audit
d) weekly peer review
Ans is C
814. framework through which UK National Health Service (NHS) organisations and
their staff are Accountable for continuously improving the quality of patient care
a) clinical governance
b) clinical effectiveness
c) clinical audit
d) weekly peer review
ans is A
815. Patient with history of removal of multiple intestinal polyps has multiple
supernumerary teeth is going to do a panorama so what do you expect:
a) Multiple cysts
b) Multiple osteoma
c) Multiple abscess
d) Multiple bony defects
e) Multiple exposed teeth
ans is B
816. Patient is presented with wear facets due to bruxism and you want to make an
appliance for him. how would he pay?
a) 21.6 pounds
b) 59.10 pounds.
c) 256.5 pounds
d) No claim
ans is B??
BY DR.ABDULRAHMAN ALMUALM
MJDF MCQS WITH ANSWERS
c) 256.5 pounds
d) No claim
819. To ensure high quality service, the general dental practice should audit:
a) Weekly checks for emergency medical equipment and drugs
b) Daily checks for emergency equipment and drugs
c) Monthly checks for emergency equipment and drugs
d) Check the equipment annually and the drugs in its expire date
e) Follow the manufacturer instructions of every single items
Ans is A
820. 55 years old female patient with history of hip fracture and osteoporosis taking
oral bisphosphonates from about 2 years, no other medications. She want to extract
a tooth:
a) Extract with no alteration but advice to keep her mouth clean
b) Stop the drug 3 days before extraction after consultation with GP
c) Consider 3 months drug holiday after consultation with GP
d) Never to extract, RCT and crown amputation
e) Extract under antibiotic cover
ans is A??
822. Pregnant and lactating mother with swelling in the mandibular region &
submandibular lymphadenopathy she is allergic to penicillin, what type of antibiotic
should you prescribe
a) Clindamycin
b) Co-amoxiclav
c) Erythromycin
d) Metronidazole
e) Metronidazole and co-amoxclav
ans C
BY DR.ABDULRAHMAN ALMUALM
MJDF MCQS WITH ANSWERS
821. Regular attending patient who is epileptic and usually have fits in the clinic what
is the medication that must be present?
a) Phenytoin
b) Midazolam
c) Diazepam
d) Carbamazepine
e) Sodium valproate
ans B
822. What is the next step after calling for help (999) in BLS
: a) 30 chest compressions
b) 2 rescue breaths
c) 5 rescue breaths
d) 20 chest compressions
e) Shout for anybody help
Ans is A
823. Patient with candidiasis and taking warfarin so the antifungal of choice is:
a) Nystatin
b) Fluconazole
c) Amoxycilin
d) Metronidazole
e) Miconazole
Ans A
Ans is C
BY DR.ABDULRAHMAN ALMUALM
MJDF MCQS WITH ANSWERS
e) 50 mg once daily
Ans is E
827. Patient with high arched palate, aortic aneurism and tall, slim legs
828. After avulsion trauma for 10 years old child, the survival and prognosis of
periodontal tissue is dependent mostly on: a) Stage of root development
b) Contamination of root surface
C.Extra-alveolar dry time
d) Immunization of the child
e) Open or close apex
Ans is C
829. patient asks for his records what is the best action:
a) pay fees and give records in maximum 20 days
b) pay fees and give records in maximum 40 days
c) no fees and give records in maximum 20 days
d) no fees and give records in maximum 40 days
ans is B
830. The required CPD points for dentist to be completed every 5 years:
a) 50 CPD
b) 75 CPD
c) 100 CPD
d) 150 CPD
e) 200 CPD
Ans is B
831. The minimum CPD points a dentist can provide to the GDC for renewal over 2
years: a) 10 CPD
b) 20 CPD
c) 50 CPD
d) 5 CPD
e) zero
832. Patient with liver disease and dental abscess, antibiotic of choice is:
a. clindamycin
b. doxycycline
c. ceftriaxone
d. erythromycin
e. ciprofloxacin
Ans is C
BY DR.ABDULRAHMAN ALMUALM
MJDF MCQS WITH ANSWERS
833. Which of the following drugs interacts with warfarin and increases the patients
INR?
a) Fluconazole
b) Cephalosporin
c) Tetracycline
d) Nifipedine
e) Conaz
ans A
834. Patient faint on the dental chair after finishing long dental procedure the most
likely diagnosis is
a) Vasovagal attack
b) Epilepsy
c) Cardiac arrest
d) Diabetic coma
e) Anaphylactic shock
ans A
838. In line with the mental capacity act, a person is regarded as being unable to
give consent or make decisions if he is unable to do [4 steps to assess the mental
capacity]
BY DR.ABDULRAHMAN ALMUALM
MJDF MCQS WITH ANSWERS
839. cleido-cranial dysplasia is not associated with [in the head and neck]:
a) Supernumerary teeth
B) delayed closure of fontanels-hypodontia
c) prominent clavicles
d) posing of forehead
e) dentigerous cyst formation
ans is C
840. according to the mental capacity act, advance decisions can be made by
people who lack mental capacity who are:
a) above 16
b) above 18
c) below 16
d) below 18
e) doesn’t matter
ans B?
841. During clinical audit which of the following may lead to direct dismissal without
warning
a) No enough informed consent
b) No enough confidentiality
c) No enough staff available
d) incorrect hiring of staff member
e) Wrong extraction
ans B
842. Which of the following is considered “never events” that may lead to series
incidents which may lead to closing the practice permanently?
a) Extraction of wrong tooth [wrong side surgery]
b) Causing temporary blindness
c) Causing perforation
d) Causing temporary paralysis
Ans A
Ans B
844. According to the recent recommendations for infection control the floor of the
practice should be :
a) Wipe-able
BY DR.ABDULRAHMAN ALMUALM
MJDF MCQS WITH ANSWERS
b) Shiny
c) Wooden
d) At 90 degrees to the walls
e) Impervious
Ans is E
845. After doing pulpectomy, you performed a readymade stainlesteal crown and
space maintainer So under which band you should treat this child:
a) Band1
b) Band 2
c) Band 3
d) Band 4
e) No claim
ans C
845. Your assistant called you Friday morning and said that she just realized that her
GDC license has expired and she forget to pay for renewal. What should be the most
appropriate action?
a) Immediately inform the GDC
b) Tell her to work under supervision until she pay
c) Tell her that it is her responsibility
d) Pay for her
ans C??
846. 70 years old incapacitated patient with dementia suffering from parotid gland
infection and pus oozing who needs facial surgery. He is unable to consent and lost
power of attorney so according to mental capacity who will give consent:
a) Patient young carer
b) 1st degree relative
c) Solicitor
d) Independent mental incapacity advocate
e) Patient’s friend
Ans C
847. According to the recent infection control recommendations, the partially charged
local anesthetic cartridge should be disposed via: a) Clinical wastes container
b) Sharp hazardous wastes container
c) Amalgam container
d) Fixer container
BY DR.ABDULRAHMAN ALMUALM
MJDF MCQS WITH ANSWERS
848. Oral steroids are commonly used for treatment of chronic disease like COPD,
which of the following conditions may be exacerbated by the use of steroids
a) Asthma
b) Psoriasis
c) Diabetes
d) Inflammatory bowel disease
e) Systemic lupus
ans C
849. During decontamination stage; the temperature of the used water shouldn’t
exceed
a) 25°
b) 35°
c) 45°
d) 55 °
e) 65°
Ans C
851. Who of the following should be registered with GDC other than dentist?
a) Practice manager
b) Laboratory technician
c) Laboratory technician assistant
d) RPA
e) Dental receptionist
ans B
BY DR.ABDULRAHMAN ALMUALM
MJDF MCQS WITH ANSWERS
c) Three days
d) seven days
e) Five days
Ans D
Ans B
855. Which of the following fire extinguishers should be present in the dental clinic to
control electrical fire:
a) Carbon dioxide powder
B) Wet chemical agents
C) Dry powder
D) Water supply
E) Foam
Ans A
a) Formalin
b) Water
c) Sodium hypochlorite 10% diluted
d) Distilled water
e) Glutaraldehyde
Ans C
857. Extracted teeth with amalgam restoration for educational purpose are best
stored in:
a) Formalin
b) Water
c) Sodium hypochlorite 10% diluted
d) Distilled water
e) Glutaraldehyde
Ans is A
858. Patient has TIA [transient ischemic heart disease], you have planned extraction
and the patient is on aspirin, what is the best way to proceed
a) Delay the extraction
b) Refer to GP
c) Refer to oral surgeon
d) Go ahead with extraction but with local measures
BY DR.ABDULRAHMAN ALMUALM
MJDF MCQS WITH ANSWERS
859. First thing to do if patient faint on the dental chair during treatment on supine
position :
a) Give Adrenalin .05 ml of 1:1000 intra-muscular
b) Call the ambulance
c) Shout for help
d) Check airway, pulse rate and Start ABCD approach
e) Open airway
ans D
861. What will be the best action for a patient with prolonged epileptic seizures :
a) Give him o2
b) Give him midazolam IV
c) Give him midazolam IM
d) Give him midazolam Bucally
e) Give him diazepam
ans D
862. 28 years old female patient complaining of pain in lower jaw, you took xray to
find a fractured mandible. She then admitted that her partner hit her while he was
drunk, she has good capacity
a) report to police
b) safeguard her to social service
c) refer to community service
d) refer to hospital
e) respect her confidentiality
#! Either e or B ..????
863. Which of the following lap investigation is important for a patient taking aspirin
a) PT
b) PPT
c) Bleeding time
d) Clotting time
e) INR
ANS C
864. You realized that the patient is oversedated during IV sedation with midazolam
a) Flumazenil
BY DR.ABDULRAHMAN ALMUALM
MJDF MCQS WITH ANSWERS
b) O2
c) Diazepam
d) Glycril nitrates
e) Adrenalin IM
ans A
865. Patient entered to automated external ventricular defibrillator, what is your next
step
a) Administer adrenaline
b) Dc shock 150/200
c) Assess the rhythm
d) Don’t continue or do
anything
e) Start CPR immediately
Ans C
BY DR.ABDULRAHMAN ALMUALM
MJDF MCQS WITH ANSWERS
868. 8 years old child has extracted his permanent 1st molar, how long should the
dentist keep the records
a) 11 years
b) 25 years
c) 8 years
d) 17 years
e) 50 years
Ans is D
869.
For a patient with ulcer, the most common area in which the ulcer may chan
ge to malignancy is:
A)Upper eye lid
B)Lower eye lip
C)Lower lip
Ans C
A)Obstructive parotitis
B)Sialolith
C)Sialadenitis
Ans A
A)bolus 1 mg midazolm
B)bolus 2 mg midazolam
C)no midazolam
Ans B
BY DR.ABDULRAHMAN ALMUALM
MJDF MCQS WITH ANSWERS
A)Effective dose.
B)Absorbed dose.
Ans B
Ans C
878.patient 27 years
old good oral hygiene low caries risk don't need any ttt recall is every ?
A)6 months.
B)12 months
C)18 months
Ans C
BY DR.ABDULRAHMAN ALMUALM
MJDF MCQS WITH ANSWERS
881
if u know that the risk of cancer from panoramic radiograph is 24 then the ri
sk from CBCT is how much times multiple than panorama?
A)0.5_1 times
B)2_42 times C)200_300
#!#8
882. percentage of smoker develop cancer more than non smoker?
A)34
B)7-10
C)14-17
Ans B
Ans A??
885
during u doing surgical extraction of lower 8 there was massive bleeding ,, ex
traction was difficult ,..pt came to u with numbness in lower lip & tongue ner
ve damge after surgical extraction what u do?
A)reassurance and review 6 months later
B)refetr to OMFs
C)prescribe dexamethadone & review later
Ans B
BY DR.ABDULRAHMAN ALMUALM
MJDF MCQS WITH ANSWERS
B)3 year)
C)5 years
Ans B???
891.
lignocaine stored out of refrigator by how months u will reduce the shelf life
of the drug?
A)6 months
B)1 year
C)3 years
Ans A
892.
Patient with long term dietary erosion, most probable clinical sign would be :
893.
calcification of seconed permenant molar to ensure maximum closure after extr
action of first permenant molar :
A)early dentin calcification at bifurcation of lower molar
B)early calcification of dentin at coronal part of root
C) mid root calcification
Ans A
895. child 5 years , pale and weak came with spontaneous gum bleeding?
A. ALL
B. AML
BY DR.ABDULRAHMAN ALMUALM
MJDF MCQS WITH ANSWERS
C. Vit k deficiency
D. Von willebrand disease
E. All of the above
Ans A
896. During clinic audit, what can caused dismission before warning?
A)breach in confidentiality
B)Uncorrect hiring of staff
C)Not enough staff
Ans is A
897. child had 5mg/kg flouride its potential lethal dose whats the first line treatment?
Give child salty drink
Give child sugary drink
Give child injection i cant remember the name of the drug
Ans is A ??
(Antidotes: < 5mg F/kg body weight= large volumes of milk.
>5mg F/kg = refer to hospital for gastric lavage.
If any delay = IV Calcium Gluconate + An Emetic )
.898 A Dentist did pulpotomy of primary molar and then after few moths he did
stainless steel crown. What band he gets to claim.
Band 1
Band 2
Band 3
Band 4
No Claim
Ans B
.899 After doing mesio-angular impaction of lower third molar, which suture
material is best used to close mucoperiosteal flap A.3-0 Silk cutting B.needle
3-0 Vicryl C.cutting needle
Catgut
Ans B
.900 Patient has TIA (Transient ischemic attack three months ago, you have
planned extraction and patient is on aspirin. What is the best way to proceed?
A.Delay the extraction for three months
B. Go ahead with extraction, following appropriate local measures
C.Refer to Oral surgery
DStop aspirin
Ans B
901Patient has an over jet of above 9mm which UDA band it will be
Band 1
Band 2
Band 3
Band 4
Band 5
BY DR.ABDULRAHMAN ALMUALM
MJDF MCQS WITH ANSWERS
Ans C
902Patient has an over jet of above 9mm which UDA band it will be
Band 1
Band 2
Band 3
Band 4
Band 5
.906 Patient had pain in the chest after prolonged dental session, with an
extraction procedure, what is the best medicine to give
GTN (nitroglycerin)
ASPIRIN
Ans A
BY DR.ABDULRAHMAN ALMUALM
MJDF MCQS WITH ANSWERS
908Patient is waiting in waiting room and collapse and faints. Patient is cold,
clammy, but pulse is good. What do you give?
IM Glucagon
Oral Glucose
GTN
Hydrocortisone
Diazepam
BY DR.ABDULRAHMAN ALMUALM
MJDF MCQS WITH ANSWERS
A Fox-bite-plane
B OCcluding the upper and lower denture
Ans A
BY DR.ABDULRAHMAN ALMUALM
MJDF MCQS WITH ANSWERS
BY DR.ABDULRAHMAN ALMUALM
MJDF MCQS WITH ANSWERS
916Burning mouth syndrome which part of the oral cavity is more painful
#Tongue
918GTN in Angina
a.Relaxes heart muscle
b.Decreases blood Pressure
C. Increase positive ionotropic
charge
D. Dilation of arterioles and
venules
Ans D
BY DR.ABDULRAHMAN ALMUALM
MJDF MCQS WITH ANSWERS
A.Orbital fracture
b.Zygomatic fracture
c.Lefort-l
D. Lefort-ll
Ans D
.922 Patient had trauma with minimal mobility of front teeth, no symptoms or
pain or anything, what is the best approach
.923 Patient had trauma 8 days ago, upper central incisor palatal luxation mild,
not interfering with occlusion but tooth non vital. What do you do?
,Ferric sulphate
Formocresol
Ans A
925Patient had trauma on 14 (vital pulp), only thin buccal cusp remaining, best
treatment option
MOD-Onlay
BY DR.ABDULRAHMAN ALMUALM
MJDF MCQS WITH ANSWERS
inlay
92618 months of age, what is fluoride supplement for child living in 0.25 ppm
fluoridated water?
0.25mg
0.5 mg
1 mg
Ans A
9293-year old with fluoride concentration of 0.3 ppm fluoridated water, what is
the fluoride supplement dose given
0.25 mg
0.5mg
Ans B
930Duraphat in high caries risk group children is applied every
2 months
4monttls ~.
6 months
8 months
One-yearly
Ans B ?
BY DR.ABDULRAHMAN ALMUALM
MJDF MCQS WITH ANSWERS
933Cervical line and dark lines on Central incisors, molars and incisal edges,
most possible reason could be
Ameologensis
Dentinogenesis
Childhood illness
Hypoplasia
Ans C
934An edentulous patient presents with ulcer in retro molar pad area first line
of management would be
935Possible infection that would occur from needle stick injury in a person
who is appropriately or properly immunized
a-Hepatitis B
b-Hepatitis C
c-Hepatitis A
d-HIV
Ans B
30% chances for HBV incase of non immunised person exposed to infected blood,
3% HCV and 0.3% HIV
936Got a call from mother of a 3-year old child, who had trauma and avulsed
the front tooth. What instruction you will give to
937A patient was diagnosed with salivary gland malignancy which has infiltrated
through the
nerves quickly:
a) Muco-epedrmoid carcinoma
b) Adencystic carcinoma
c) Pleomorphic adenoma
d) Warthon’s tumour
Ans B
BY DR.ABDULRAHMAN ALMUALM
MJDF MCQS WITH ANSWERS
938A mother has a newly born baby with cleft palate, what to tell her about the
condition?
a) It affects 1:140 of population
b) It is due to genetic factors
c) It is inherited condition
d) It is more common in males than females
e) Multifactorial due to genetic and environmental causes
Ans E
93913 year old patient with non-crowded arch and good occlusion except for
palatally
displaced upper lateral incisor:
a) Leave it until puberty
b) Fixed appliance orthodontic therapy
c) Removable orthodontic appliance
d) Correct with crowns
e) Surgical intervention for repositioning
ans C
94013-year-old patient with non-crowded arch and good occlusion except for
palatally
displaced upper lateral incisor in crossbite, which component will you use in your
orthodontic appliance?
a) Z spring
b) T spring
c) Z spring with posterior bite-plan [posterior capping]
d) Howley appliance with labial bow
e) Expansion screw
ans C
941For a patient, only lower first premolar didn’t have BOP, there are 4 teeth in that
sextant. What is the BPE score for this sextant :
a) BPE 0
b) BPE 1
c) BPE 2
d) BPE 3
e) BPE 4
Ans B
943Patient presented with hemorrhagic erythematous ulcers and crusted lips, the
diagnosis may be?
BY DR.ABDULRAHMAN ALMUALM
MJDF MCQS WITH ANSWERS
a) Erythema multiform
b) Pemphigus vulgaris
c) Mucous membrane pemphigoid
d) Systemic lupus erythematosus
e) Liver cirrhosis
Ans A
BY DR.ABDULRAHMAN ALMUALM
MJDF MCQS WITH ANSWERS
949the feature best supporting inhalation sedation over the oral sedation is
a) can be titrated according to patient response
b) can be titrated according to painful stimuli
c) can be titrated according to patient age
d) doesn’t require anesthesia
e) can be reversed in case of over-sedation
Ans A
950
what best describes sedation?
a) patient conscious, protective reflexes retained, can respond to verbal
b) patient unconscious, protective reflexes lost, cannot respond to verbal
c) patient conscious, protective reflexes lost, can respond to verbal
d) patient conscious, protective reflexes lost, cannot respond to verbal
e) patient conscious, protective reflexes retained, cannot respond to verba
ans A
.952 The 1st colored band in the nabors probe used for measuring furcation:
a) 3-6 mm
b) 3.5-5.5 mm
c) 2-4 mm
d) 4-6 mm
e) 8-11 mm
Ans B??
954A 35 years old patient complains of swollen gums, present for several years this
is:
a) Cyclosporine associated enlargement
b) Chronic gingivitis
c) Vitamin c deficiency
d) Atenolol therapy
e) Pregnancy gingivitis
Ans B
BY DR.ABDULRAHMAN ALMUALM
MJDF MCQS WITH ANSWERS
.956
The treatment of choice for a patient with BPE 3 should be:
a) OHI and scaling
b) OHI, scaling and root planning
c) Flap surgery
d) OHI, scaling and restore faulty restorations
e) Refer to specialist
Ans B
957A child of 8 years old come suffering from impaction of upper 1st molar and the
upper
E is still in place so the correct action is:
a) Extract E
b) OHI and review for self-correct
c) Refer to orthodontic for disimpaction
d) Extract E and remove 6 as well
e) Put crown on E and review
Ans B???
958A 3-year-old boy avulsed 61, and mother re-implanted it in its socket and
clot is holding the primary tooth. IOPA shows permanent tooth present, what
would you do
A. Leave it and review
B.Splinting for one week
C. Remove and space maintenance
Ans C
.959 Patient has painful symptoms at the time of eating and also has ear pain,
what x-ray would you consider. (The option of occlusal x-rays was not given in
the choices)
OPG
Oblique Lateral
Lateral chepalograph Postero-anterior view
Ans B
960A child has grossly broken down upper and lower molars, carious tooth,
which x-ray would you consider
IOPA
BY DR.ABDULRAHMAN ALMUALM
MJDF MCQS WITH ANSWERS
Bitewings_ .
Vertical Bitewings
DPT
Ans is D
963When preparing the Apical Zone, the use of the files sequentially from apex
to backwards (lower the size of instrument) - what is the best distance to
achieve good apical area preparation
0.5mm
1.0mm
l.5mm
2.5mm
3.5mm
Ans is B
964Best material to use in the root caries and buccal caries in elderly patient.
Hybrid Composite
Amalgam
Silicate cement
Zinc phosphate
Ans is C
965A diabetic patient complains of pain on a root canal treated tooth. Root
canal is done 2 years ago. On examination the restoration on the tooth seems
leaking. Reasons for pain and failure
A.No proper obturation
b.Medical condition
C.Lack of coronal seal Inadequate
D.biomechanical preparation
Ans C
Straight canals
BY DR.ABDULRAHMAN ALMUALM
MJDF MCQS WITH ANSWERS
968A child patient is brought to the dental clinic, parents complain of child having
trauma and has fallen down, hurting his teeth. You suspect NAI's. Which is the most
appropriate finding which would give suspension of NAI'S?
Red eye
Neck bruise
Child agitated and distressed
Injuries to shin and knee areas
Ans B
969What is the bpe score if gingival recession of 2mm and band completely
disappears
1
2
3
4
4* due loss of attachment >7
Ans E
Peri-orbital edema
Loss of visual acuity
Loss of sensation or paresthesia over cheek
Ans is C
BY DR.ABDULRAHMAN ALMUALM
MJDF MCQS WITH ANSWERS
Diameter
Size
Luting cement
Type of post
Ans C
973RCT treated tooth, post placed, which is the best option to restore the
tooth. Nayyar technique used, good tooth structure 50%
Gold crown
Gold inlay
PJC
Gold Onlay
Ans A
Lingual
Glossopharyngeal
Hypoglossal
Chordatympani
Ans is C
Patients requiring minor dental procedures (including extractions) who have an INR
below 4 may continue warfarin without dose adjustment’. INR should be checked
within 72 hours of the procedure starting. If multiple extractions are required, then a
single extraction should be carried out first. Subsequent extractions of two to three
teeth at a time may be carried out if recovery is uneventful.
BY DR.ABDULRAHMAN ALMUALM
MJDF MCQS WITH ANSWERS
977Secondary impressions for edentulous patient close fit tray with hyper gap
reflex. Which material you use for taking an impression
A Alginate
B. Silicone rubber based material
C. Plaster of Paris
D. Zinc oxide eugenol
Ans D???
978Palatally impacted upper maxillary canines. How would you know? Whether
they are impacted palatally or buccally and which of the following x-rays are
best option
A. Paralleling technique
B. Bisecting technique Bitewings
C. OPG
D Taking two x-rays, with paralleling technique
Ans D
980A patient came after two days of extraction, complains he had pain for two
days, examination showed localized swelling, no lymphadenopathy. What is the
first line of treatment?
Possible dry socket
Oversedation
Asthma
Ans A
Give diazepam
Oxygen
Intranasal or buccal midolazam
BY DR.ABDULRAHMAN ALMUALM
MJDF MCQS WITH ANSWERS
Ans is C
983Sudden unilateral onset of facial swelling and pain over parotid area,
which is not yet fluctuant:
Bacterial saliadenitis
Salivary duct infection
Saliorrhea .
Viral siliadenitis
Ans D
985A fifteen year old patient asks for bleaching of teeth. What is the first line
of management?
Discuss options with both patient and parent
Discuss option with both patient and parent, once you have obtained consent
from patient
Discuss only with patient
Discuss only with parent
Ans is B ?
986child avulsed permanent tooth came with her aunt, what will you do?
Do not do anything Do not do treatment as aunt is not legal guardian of the
child
Do not do treatment as child's mother is not there to give consent
Re-implant and splint it with aunt's consent
Do the best interest for the patient
Ans is E
987Best ways to reduce radiation for patient who is going to have IOPA X-rays
Lead apron
Parallel
Rectangular collimation
D-Film
Ans is C
988
46 and 44 present 45 missing, 44 minimally tilted, 46 heavily filled both are
tipped 46mesial and 44 distal>Fixed movable
989What is the best cement used to cement minimally done inlay restoration
(MOD)
GIC
BY DR.ABDULRAHMAN ALMUALM
MJDF MCQS WITH ANSWERS
ZNO
Zinc phosphate
Zinc poly carboxylate
Resin cement
Ans e
992 Patient had trauma 8 days ago, upper central incisor palatal luxation mild,
not interfering with occlusion but tooth non vital ,What do you do?
Pulp extirpation +CaoH2 dressing
Reposition
Reposition and splinting
Ans A
993Deciduous tooth had intrusion 61, what is most likely to occur for
permanent tooth
Impaction
Uneruption
Hypoplasia
Dilaceration
Ans D
The traumatically induced type is caused by intrusion of the 1° incisor, resulting in
displacement of the developing 2° incisor tooth germ. The effects depend upon the
developmental stage at the time of injury
995Which test is the most reliable test to indicate the presence of active hepatitis
H bs(surface }antigen
Hbc(core)antigen
Hbe antigen
Antibody to HBs antigen
BY DR.ABDULRAHMAN ALMUALM
MJDF MCQS WITH ANSWERS
Ans C
997Medical condition of pt who reserved a seat in a dinner meating for his dead wife
Mania .
Anxiety
Depression
Schizphrenia
Ans D
998Lady in 40s got TMJ pain for about few months, she got divorced recently what is
the first line treatment
BY DR.ABDULRAHMAN ALMUALM
MJDF MCQS WITH ANSWERS
1004Which of the following may affect the results of electric pulp testing?
A. Emotional factors.
B. Pain threshold.
C. Analgesics.
D. Recent trauma.
E. All of the above.
Ans E.
1007Which is the best cantilever bridge design for missing maxillary canine?
Abutment on
A.Both premolars
B.Lateral and central incisor
C.Lateral incisor
D.First premolar
Ans B
BY DR.ABDULRAHMAN ALMUALM
MJDF MCQS WITH ANSWERS
Ans A
1013What is the reason for a tooth to develop pulpitis several years after
setting of a full veneer gold crown?
A. Bacterial microleakage
BY DR.ABDULRAHMAN ALMUALM
MJDF MCQS WITH ANSWERS
1015In construction of full dentures, what does the term "too low vertical
dimension" refer to?
A.A situation in which there is too much interocclusal space between upper and
lower artificial teeth when the mandible is in rest position.
1019What kind of root fracture in a tooth has the best prognosis? A fracture at
the
A.Apical third
B.Coronal third
C.Middle third
D.Vertical fracture
Ans A
Lactobacillus species are also acidophilic and have been implicated in fissure
caries
1021Child with sore throat and feeling unwell,he got macular rash on his
cheek?
A.measles
b.chicken pox
c.herpes simplex
d. Infectious mononucleosis (glandular fever)
Ans A
BY DR.ABDULRAHMAN ALMUALM
MJDF MCQS WITH ANSWERS
1022Pt worried of getting cancer what the advice that should u give? Regular
check up
5 fruit/veg a day
1023Pt got recently metallic taste sensation after taking oral medication?
A.burning mouth syndrome
b.oral dysthesia
c.oral dysgeusia
ans C
1026A patient in your dental chair shows chest pain, weak pulse and dysponea,
what is your initial management,
● Administer nitro-glycerine and keep the patient up seated
● Put the patient in supine position
● Wait until the symptoms go away
Ans A
1027Developer was contaminated with other chemical and was not mixed
properly. What is the effect on the X-ray film?
● Too dark film
● Light film
● Foggy
And C
BY DR.ABDULRAHMAN ALMUALM
MJDF MCQS WITH ANSWERS
1030Some hours after the extraction of a lower molar the patient complains of
prolonged post operation bleeding and pain, how would manage this,
● Prescribe analgesics and ask the patient to follow a strict oral hygiene
● Administer 5% Marcaine Local Anaesthetic, prescribe analgesics and pack
the socket with alvogyl
● Administer 5% Marcaine Local Anaesthetic, suture the socket and prescribe
analgesics
● Suture and give pressure packs
Ans C
1032 A patient with no positive history came along for scaling. The moment
you pick up your anterior scaler you punch your finger, what should you do?
● Complete the procedure as if nothing has happened
● Check dentist's blood for Hepatitis B antibody HBsAb
● Check dentist's blood for Hepatitis B antigen HBsAg
● Check patient's blood for Hepatitis B antibody HBsAb and HIV antigen HIVAg
● Check patient's blood for Hepatitis B antigen HBsAg and HIV antibody HIVAb
● Dentist should go and take a HBsAb vaccine
Ans A
1033A patient on the dental chair has cardiac arrest. What is INCORRECT,
● Observing the vital signs and check that the air way is clear is at high
importance
● Expired air has 15% 02 only, and cardiac compressions achieve 30-40% of
cardiac output
● Intermittent positive pressure at the rate of 40/min will reduce the chances
of cerebral hypoxia
● Intermittent positive pressure is better than mouth to mouth when it has
been given at the same rate.
● You check the pulse and respiration of the patient before starting any
resuscitation
Ans C
Ans D
BY DR.ABDULRAHMAN ALMUALM
MJDF MCQS WITH ANSWERS
Pulpectomy
Ans a
1036Pt with labially impacted upper canine, how does u know clinically?
Swelling on the labial area
Swelling on the palatal side
Ans is A
BY DR.ABDULRAHMAN ALMUALM
MJDF MCQS WITH ANSWERS
BY DR.ABDULRAHMAN ALMUALM
MJDF MCQS WITH ANSWERS
C Bupivcaine
D Benzocaine
E Articaine
Ans is B
A2
B3
C4
D5
Ans is C
Ans A
.1052 what ur advice to childs mum whos her child s teeth knocked out?
a.Reimplant it
b.keep it in the milk and come as soon as possible correct
c.keep it in the water
if permenent tooth // ans is B ??
1053What is the most correct flap design in apecictomy?
Apical re position
Apex is broader than the base
Base is broader than the apex
Apex and base are equal
Ans is C
BY DR.ABDULRAHMAN ALMUALM
MJDF MCQS WITH ANSWERS
.1056 What is the purpose of making a record of protrusive relation and what
function does it serve after it is made?
A.To register the condylar path and to adjust the inclination of the incisal
guidance.
B.To aid in determining the freeway space and to adjust the inclination of the
incisal guidance.
C.To register the condylar path and to adjust the condylar guides of the
articulator so that they are equivalent to the condylar paths of the patient.
D. To aid in establishing the occlusal vertical dimension and to adjust the
condylar guides of the articulator so that they are equivalent to the condylar
paths of the patient.
Ans C
1057A 50 years-old patient presents with pain from time to time on light
cervical abrasions. What is your first management to help patient in preventing
pain in the future?
A.Change diatary habits
B.Change brushing habits
C.GIC fillings
Ans B
1058planning and construction of a cast metal partial denture the study cast
● facilitates the construction of custom trays
● minimizes the need for articulating
● provides only limited information about inter ridge distance, which is best
assessed clinically
● can be used as a working cast when duplicating facilities are not available
Ans A
BY DR.ABDULRAHMAN ALMUALM
MJDF MCQS WITH ANSWERS
● Compared with the pre-resorption state, the mandibular ridge will lose more
bone from the lingual aspect than the buccal one.
Ans is D?
1064 How does fluoridation work in theory? A. Fluoride ions are integrated by
changing Hydoxylapatite to Fluorapatite
1065A major difference between light cured and chemical cured composite is
that during setting or in function the light cured materials tend to:
● Seal the margins better and completely
● Exhibit less wear on time
● Undergo greater colour change
● Shrink more rapidly
● Posses greater fracture toughness
Ans D
Ans B
BY DR.ABDULRAHMAN ALMUALM
MJDF MCQS WITH ANSWERS
Ans E
1069 The minimal labial tooth reduction for satisfactory aesthetics with
porcelain fused to metal crown is,
A.1mm
B.The full thickness of enamel
C.1.5 mm
D.2.5mm
E. One third of the dentine thickness
Ans C
1071 When a removable partial denture is terminally seated the retentive clasps
tips should:
A. Apply retentive force into the body of the teeth
b.Exert no force
c.Be invisible
d.Resist torque through the long axis of the teeth
Ans is B
BY DR.ABDULRAHMAN ALMUALM
MJDF MCQS WITH ANSWERS
X Vagus. Has a motor input to the palatal, pharyngeal, and laryngeal muscles.
Impaired gag reflex, hoarseness, and deviation of the soft palate to the unaffected
side are seen if damaged. The vagus has a huge parasympathetic output to the
viscera of the thorax and abdomen
1075 Which of the following liquids is not suitable for prolonged immersion of
cobalt chrome partial dentures:
A. Alkaline peroxidase
B. Sodium hypochlorite
C.Soap solutions
D.Water
Ans is B
BY DR.ABDULRAHMAN ALMUALM
MJDF MCQS WITH ANSWERS
BY DR.ABDULRAHMAN ALMUALM
MJDF MCQS WITH ANSWERS
A.2/3 of tooth
B.the tooth length
C.1.5 times that of the crown
D.Is root length
E.The length of the crown
Ans is E
109740 years old patient presented to your clinic complaining of salty taste and signs
of
unilateral inflammation affecting the check near to the upper 1st molar, what will be
the
1
st examination to do to reach diagnosis
a) Stimulated saliva sialometry
b) Un-stimulated saliva sialometry
c) Sialograph
d) MRI
Ans A
BY DR.ABDULRAHMAN ALMUALM
MJDF MCQS WITH ANSWERS
109930 years old patient came to your clinic for treatment under IV midazolam
sedation so
how can you achieve optimum sedation?
a) Bolus injection of 1 mg and 1 mg to be titrated
b) Bolus injection of 2 mg and 1 mg to be titrated
c) Bolus injection of 3 mg and 1 mg to be titrated
d) Bolus injection of 1 mg and 2 mg to be titrated
e) Bolus injection of 1 mg and 3 mg to be titrated
ans B?
Ans A
1104patient has round chronic lesion on the dorsum surface of the tongue
a) geographic tongue
b) migratory glossitis
c) median rhomboid glossitis
d) lichen planus
BY DR.ABDULRAHMAN ALMUALM
MJDF MCQS WITH ANSWERS
e) psoriasis
Ans C
110510 years old boy, developed small greyish white lesions surrounded by red
halos in the
soft palate, and small vesicles are presented at the tonsillar pillar as well. He is
feverish
and reporting pain with swallowing. What is the most likely?
a) Herpangina
b) Herpes simplex
c) Post herpetic neuralgia
d) Glosso-pharyngeal neuralgia
ans A
BY DR.ABDULRAHMAN ALMUALM
MJDF MCQS WITH ANSWERS
C. Microfibrillar collagen.
D. Topical bovine thrombin.
E. All of the above
Ans E
1113A 8-year old child has an 8mm central diastema. The etiology could include
1. frenum.
2. cyst.
3. mesiodens.
4. normal development.
A. (1) (2) (3)
B. (1) and (3)
C. (2) and (4)
D. (4) only
E. All of the above.
Ans A
1114While you finish a class I cavity, the enamel is sound but you notice a
thin brown line ln the dentine and on the dentino-enamel junction, what is your
response,
● You leave it and complete the final restoration
● You extend your preparation and clean it
● You apply a cover of varnish
Ans A
1115Dental caries ofthe proximal surfaces usually starts at,
● Somewhere between the ridge and the contact area
● Just gingival to contact areas
● Just above the gingival margin
● At the contact point
BY DR.ABDULRAHMAN ALMUALM
MJDF MCQS WITH ANSWERS
Ans B
1117The difference between normal stone and the die stone is,
● In the size and shape of the particles
● The mixing
Ans A
1118The advantage of the silicone in soft relining material over hard plastic
acrylic materials is,
● Capability to flow
● Prevents the colonization of Candida albicans
● Resilient in long run
● Better bond strength
Ans C
1120The flexibility of the retentive clasp arm does not depend on:
● Length of the arm
● The cross section shape
● The material used
● Degree of taper
● The exerted force
Ans E
BY DR.ABDULRAHMAN ALMUALM
MJDF MCQS WITH ANSWERS
● Reproduction of normal tooth incline in the gingival one third of the crown
● Slight over contouring of the tooth in the gingival one fifth of the crown
● Slight under contouring of the tooth in the gingival one fifth of the crown
Ans is B
1126Two central incisors on a radiograph are showing with what looks like eye
drop radiolucency. You decided to start endodontic treatment on these teeth but
when you tried to open access to the root canal you find clearly closed orifices
with what look like secondary dentine. What is your initial management?
● Leave as it is and start a permanent restoration.
● Start systemic antibiotic
● Try to ream and file canals
Ans C
1127After the initial development stage and in the absence of pathology, the
size of the pulp chamber has been reduced by, ● Deposition of primary dentine
● Deposition of secondary dentine
● Reparative dentine
● Pulp fibrosis
● Deposition of reparative dentine
Ans B
BY DR.ABDULRAHMAN ALMUALM
MJDF MCQS WITH ANSWERS
Ans C
1129 The light emitted by the polymerization lamp has to be checked from time
to time. The meter used for this only measures light in the range of:
● 100-199 nm
● 200-299 nm
● 300-399 nm
● 400-499 nm
Ans is D
1131Where would you expect to find the mylohyoid muscle in relation to the
periphery of a full lower denture:
A.Mandibular buccal in the midline
B.Mandibular lingual in the first premolar area
C.Mandibular lingual in the midline
D.Mandibular disto buccal area
Ans B
BY DR.ABDULRAHMAN ALMUALM
MJDF MCQS WITH ANSWERS
1137A suddenly swollen upper lip that lasts for 48 hours or more is most likely
A.Haemangioma
B.Agioneurotic oedema
C.Mucocele
D.Cyst
Ans A
.1139 What is the best way for a permanent decline in caries of a population?
A.Change diatary habits
B.Topic and water fluoridation
C.Awareness of dental health matters
D.Better tooth brushing
Ans is B
BY DR.ABDULRAHMAN ALMUALM
MJDF MCQS WITH ANSWERS
B.erupt lingually
C.ankylose
Ans B
1147patient presents with pain in the upper left segment. On inspection you
find a localized alveolar abscess distal 27. What will be you management? -
A.Drainage
B.Extraction of tooth ||
Ans is A
1148What does not help in establishing the caries risk in children?
A.History of caries
B.Lactobacilius count
C.Dietary habits
D.Brushing habits
E.Genetic predisposition
Ans is E?
BY DR.ABDULRAHMAN ALMUALM
MJDF MCQS WITH ANSWERS
Ans A
1151When no radiation shield is available, the operator should stand out of the
primary x-ray beam at a distance from the patient's head of at LEAST:
A.0.5 metres
B.1 metre
C.1.5 metres
D.2 metres
E. 3 metres
Ans is D
Exposure switches should be positioned so that the operator can remain outside the
controlled area and at least 2m from the X-ray tube and patient the controlled
area>1.5 m in any direction from the patient and tubehead and anywhere in the line
of the main beam until it is attenuated by a solid wall.
1155What are the points that determine the facial line in cephalometric points
● Nasion, pronasale, pogonion.
● Sella, nasion, pogonion
BY DR.ABDULRAHMAN ALMUALM
MJDF MCQS WITH ANSWERS
Ans A
1156A 10 year old boy presents with small greyish white lesion surrounded by a
red halos on the soft palate and tonsillar pillars, small vesicles are found. He
has fever and pain in the ear and won't eat. The MOST probable diagnosis is?
A.Herpangina
B.Measles
C.Primary herpetic stomatitis
Ans is A
Herpangina Febrile illness with sore throat due to ulcers on soft palate and
throat. Usually lasts about 3–5 days. Rx: soft diet.
1158What is the effect of office dental prophylaxis of regular six month intervals
on children's oral health?
● Reduced caries incidence by approximately 30%
● Provide a long term improvement in oral hygiene
● Provide a short term improvement in oral hygiene
● Prevent gingivitis
● Reduce the need for patient cooperation
Ans is C?? /
1159What is the most frequent cause of pain which occurs several days after
obturation?
● Entrapped Bacteria in the periapical region
● Underfilling the root canal system
● Overfilled root canal
Ans A
The following factors need to be considered should pain occur following sealing of
the root canal system. High restoration Overfilling Underfilling Root fracture Once
obturation of the root canal space has been completed, restoration of the rest of the
tooth can be carried out. The occlusion must be checked for interferences, to avoid
an apical periodontitis, or worse, a fractured tooth. Root fillings that are apparently
overfilled do not as a rule cause more than mild discomfort after completion. The
BY DR.ABDULRAHMAN ALMUALM
MJDF MCQS WITH ANSWERS
most likely cause of pain following obturation of the root canal space is the presence
of infected material in the periapical region. The significance of an underfilled root
canal is whether the canal has been properly cleaned and prepared in the first
instance, and infected debris is still present in the canal. Post-endodontic pain in
these circumstances may well be due to inadequate debridement of the canal.
Removal of an overextended root filling is rarely completely successful and the
options left are as follows: Prescription of analgesics and, if the pain is more severe
and infection is present, antibiotics. An attempt at removal of the root filling and
repreparation of the root canal. Periradicular surgery and apicectomy
1160diabetic patient with moist skin, moist mouth and weak pulse; what would
you do:
● Give glucose orally
● Administer 02
● Administer adrenaline
● Inject insulin
Ans A
1161patient has developed a sever chest pain and difficulties in breathing while
in the dental chair. Your initial response is:
● Administer glycerine trinitrate and monitor patient in upright position
● Patient has an acute episode of angina as demonstrated by curve in ECG
● No treatment is required until confirmed as MI by ECG
● Patient has myocardial infarction as confirmed by ECG
Ans is A
BY DR.ABDULRAHMAN ALMUALM
MJDF MCQS WITH ANSWERS
Ans is C
1165A physician refers a nine year old boy to you to confirm diagnosis. The
boy has a fever of 40°C and coughing. When you focus your light into his
eyes he turns away. Intra-orally there are white spots surrounded by red
margins. The disease and lesions are:
● Measles and the spots are Koplik's spots
● AHGS vesicles
● Rubella and the spots are Fordyce's spots
Ans A
BY DR.ABDULRAHMAN ALMUALM
MJDF MCQS WITH ANSWERS
Ans is B
1172 A patient comes with a firm, painless swelling of lower lobe of parotid
which has grown progressively for the past year. He complains of paresthesia
for the past 2 weeks. This is most likely to be:
● Pleomorphic adenoma
● Carcinoma of the parotid
● Lymphoma of parotid
Ans B
BY DR.ABDULRAHMAN ALMUALM
MJDF MCQS WITH ANSWERS
● Dismiss the patient with instructions for warm saline rinses and re-
examination
● Repeat the biopsy
Ans D
Ans is C
1179 What effect has placing a sealant over pits and fissures on the
progression of caries?
l.Decreased new caries
2.lncreased new caries
3.Progression of exististing caries
4 No effect on existing caries
Ans is 1
BY DR.ABDULRAHMAN ALMUALM
MJDF MCQS WITH ANSWERS
.1184 58 year old male has been treated with radiation for carcinoma of tongue.
The patient complains of pain associated with poor dentition. The dental
management would be:
A.Immediate extraction of any poor teeth under local anaesthetic with antibiotic
coverage
B.Segmental dental clearance and closure to eliminate problems
C.No dental treatment may be due to neuronic of neoplasms
D. Clearance of poor dentition followed by hyperbaric oxygen treatment plus a
primary closure of wounds under antibiotic coverage E. No extraction as
radionecrosis is an important sequelae
Ans is D ?
1186 in a radiograph the roots of the upper teeth are too short because of:
● Inadequate horizontal angulation
● Too high vertical angulation
● Too small vertical angulation
Ans is B
BY DR.ABDULRAHMAN ALMUALM
MJDF MCQS WITH ANSWERS
Ans A
ANS C /
1191 Normal prothrombin time and elevated partial thromboblastin time is seen
in
A.Factor VIII deficiancy (Haemophilia)
B.Thrombocytic pupura
C.Leukemia
D.Von Willebrand disease
ANS A
1193 What is the best reason to promote tooth brushing to the public?
A.Less fissure caries
B.Less gingivitis
C.Gingival massage
Ans is B
BY DR.ABDULRAHMAN ALMUALM
MJDF MCQS WITH ANSWERS
B.lt can be easily transported from one part of the oral cavity to another
C.lt has the ability to restructure carbohydrates
Ans C
1198 A patient in your dental chair suddenly becomes agitated with shallow
breathing, full pulse and a blood pressure of 150/80. You would
A.Give oxygen
B.Give insulin
C.Give glucose
D.Place patient in supine position
Ans C
1201 A patient reports that his post crown has fallen out. This crown had been
present for many years. You note that there appears to be a hairline vertical
fracture of the root. The tooth is symptomless. What is the most sensible
approach to treatment? A.Replace the post crown using a resin-reinforced glass
ionomer material
BY DR.ABDULRAHMAN ALMUALM
MJDF MCQS WITH ANSWERS
1202A patient says that he does not like the appearance of his previously root
filled upper central incisor tooth. His dentition is otherwise well maintained and
his periodontal health is good. The tooth appears to be darker than the adjacent
teeth. What is the most appropriate approach to treatment?
A.Provision of a post crown
B.Provision of an all ceramic crown
C.Provision of a metal bonded to ceramic crown
D Non vital bleaching
E. Provision of a porcelain veneer
ANS IS D
120325 year old male attends for the first time complaining of sensitivity of a
number of teeth. On examination, the occlusal surfaces of all the teeth are
worn with obvious wear facets on the canines and premolars. Posterior
amalgam restorations are proud of the surrounding tooth. What would be the
first stage management?
● Take impressions for study models
● Prescribe fluoride mouth rinse
● Replace the amalgam restorations
● Dietary analysis
Ans is D
120435 year old male patient who admits to grinding his teeth at night has a
number of wedge-shaped cervical (Class V) lesions on his upper premolar teeth.
These are causing sensitivity and are approximately 3mm deep. What is the
correct management option? ● Provide tooth brushing instruction and fluoride
● Restore the lesions with compomer
● Restore the lesions with micro-filled composite
● Restore the lesions with a hybrid composite
● Restore the lesions with conventional glass-ionomer
Ans is C
1205patient attends with pain of four days duration in a carious upper molar
tooth. The pain is constant and is not relieved by paracetemol. Sleep has been
disturbed by the pain. The tooth is tender to percussion and gives a positive
response to Ethyl Chloride. What is the most likely diagnosis?
● Pericoronitis
● Apical periodontitis
● Marginal periodontitis
● Reversible pulpitis
Ans is B?
BY DR.ABDULRAHMAN ALMUALM
MJDF MCQS WITH ANSWERS
1206A 14 year old patient attends with a decayed and hypoplastic LL7. A
radiograph shows the presence of an unerupted LL8 and the LL6 is sound.
What would be the most appropriate long-term treatment for this tooth?
● Amalgam restoration ● Antibiotics ● Extraction
Ans is C
120730 yr-old patient attends complaining of pain from the lower left quadrant.
Clinical examination reveals an extensively restored dentition with generally good
oral hygiene. There is no significant periodontal pocketing other:.than an
isolated defect in the region of the furcation of lower left first molar which is
non-mobile. The gingival tissue in thiS area appears erythematous and slightly
hyperplastic with a purulent exudate on probing. From the list below, which is
the most appropriate next step?
● Obtain a radiograph
● Biopsy the gingival tissue
● Remove the restoration
● Vitality testing
● Prescribe antibiotics
Ans is D
Vitality testing. We want to reach a diagnosis as to whether the lesion is endodontic
or periodontal. The radiograph may show a radioloucency in the furcal area in both
cases (Primary endo lesions can cause furcal or periapical radiolucency. Furcation
involvement also causes furcal radiolucency) but will still not give us a definite
diagnosis. We initially need to know whether the tooth is vital or not, then carry on
with radiographic investigation.
1208A 40 yr old patient had root-canal treatment to his upper first molar. This
was performed 6 months ago using contemporary techniques under rubber dam
and was crowned after completion of treatment. He attends complaining of
continued discomfort from this tooth. Radiographic examination shows each of
the three roots to be obturated with a well-condensed filling to the full working
length though there is no evidence of in-fill of the periapical lesion when
compared to the pre-op view. What is the most likely cause of the continued
problem?
● Extra-radicular infection
● Contamination of canal(s) with E.faecalis
● Uninstrumented canal >MB 2
● VerticaL root fracture
● Perio-endo problem
Ans is C
120921 year old female presents for the first time to your practice. She is very
upset with the appearance of her upper left central incisor. On examination you
find healthy oral hard and soft tissues and excellent oral hygiene. On close
examination you can see that the upper left central incisor is slightly greyer than
the upper right central incisor and has a composite restoration placed palatally.
What is the most appropriate form of treatment given the information you have?
● Bleaching with carbamide peroxide in custom formed trays of upper and
lower arches
● A bonded crown
BY DR.ABDULRAHMAN ALMUALM
MJDF MCQS WITH ANSWERS
● A composite veneer
● A porcelain veneer
● E. Non-vital bleaching with carbamide peroxide
Ans E
The patient has anterior disc displacement with reduction. that's y he has midway
click .. when the condyle tries to translate.. an anterior repositioner splint will put the
condyle in a downword and anterior position allowing the disc to get back to original
position and hea A.Stabilization Splint: mainly used for the correction of the
occlsuion or creating a perfect occlusion for patient with TMD reflected from occlusal
interference, till the muscles return to their normal tension and lengh, then restorative
or selective grinding work will b done B. Localized occusal splints: used for clenching
and bruxism, focusing the occlusion on less number of teeth, making their
propioseptive response sensetive for biting, so the brain avoid excessive biting and
then decrease the Bruxism
C.Bite Raiser: used for the raisong of the bite to relive muscle stress mainly
D. Soft Bite Guard: first line of treatment in symptomatic bruxer, act to decrease
muscle spasm and decrease the habbit. E. Anterior repositioning splint: it is done as
we say [to catch up the disc] as the patient doesnt experience clicking when he
opens from Edge to Edge, thats means that first stage of translation is executed in
this edge to edge movement where the condyle and the disc are aligned as in
normal position. keeping this realtion for a long time allows the retrodiscal tissues
[where pain is moderate ] to heal on this level and the pterigoid spasm will b
decreased. THAT`S THE ANSWER TO THE CASE BEST REGARDS EBTISAM
Combination muscle and disc disorders are identified by joints that click or pop, and
muscle symptoms are also present. These disorders tend to be more chronic in
nature (unless there has been an acute exacerbation), and are associated with more
damage. Stabilization splints are the treatment of choice, as they provide long-term
BY DR.ABDULRAHMAN ALMUALM
MJDF MCQS WITH ANSWERS
wear that is usually needed. They also cover the entire dental arch, ensuring that the
covered teeth do not move. They must be worn continually for 24 hours (except
when eating) for as long as required to eliminate muscle, disc, ligament, and tooth
symptoms. Three to 6 months of wear is often required. These disorders may be
reversible if detected relatively early and treated appropriately.
.1212 A patient presents with a history of a post-crown having fallen out. The
post-crown was originally placed fifteen years ago and had been successful up
until four months ago since when it has come out and been recemented four
times. At recementation there was no evidence of any caries. The patient had
been a regular attender and not needed any restorative treatment for the last
eight years. Which of the following is the most likely cause for the failure of
this crown?
● The post was to narrow
● The post was to short.
● The root canal treatment was failing.
● A vertical root fracture was present.
● There were excessive occlusal loads on the tooth
Ans D
1213A 23 year old patient attends complaining of pain in an upper right molar
and is keen to keep the tooth. The pain is typically sharp in nature, is triggered
by cold and persists after removal of the cold stimulus. The tooth is not tender
to percussion; a radiograph of the upper right first molar shows a large
radiolucency extending to the pulp horn but no peri-radicular changes. What
treatment is most likely needed in this case?
● Oral hygiene instruction and fluoride application
● Excavation of caries and placement of a permanent restoration
● Root -canaI treatment?
● Indirect pulp cap and restoration?
● Direct pulp cap and restoration
Ans C
1214A patient attends your surgery complaining of severe pain, swelling and
mobility associated with a lower first molar tooth in which there is a broken
filling. A periapical radiograph indicates
that the tooth has not been root filled and there is loss of apical lamina dura
associated with the distal root and at the bifurcation. The periodontal bone
support is good. There is no significant pocketing. What is the likely diagnosis?
● Chronic periapical periodontitis
● An acute periodontal-endodontic periodontitis
● A chronic periodontal-endodontic periodontitis
● An acute periapical periodontitis
● Pararadicular periodontitis
Ans is D??
BY DR.ABDULRAHMAN ALMUALM
MJDF MCQS WITH ANSWERS
Acrylic
Soft tissue
Amalgam
Composite
Ans is B
1216 An upper incisor in a 16 year old patient has suffered trauma and the
coronal tissue has been lost. The tooth has been endodontically treated. How is
the tooth best restored?
● With a fibre post, direct core and crown.
● With a direct core and crown.
● With composite
● With an indirect post-core from a pre-fabricated pattern and a crown
● With a custom indirect post-core and crown
Ans E
1217 50 year old male patient has a Class III jaw relationship with an anterior
open bite. It is planned to restore his lower right second molar, which has
suffered tooth wear and fracture, with an indirect restoration. This tooth has
approximately 2mm of coronal height. What would be the most suitable
approach to restore this tooth?
● Provide an adhesively retained gold onlay
● Provide a conventional full crown
● Increase the vertical dimension and provide a full crown
● Surgically crown lengthen and provide a gold crown
● Provide an adhesively retained ceramic onlay
Ans A
1218 patient complains of a lower incisor which has been mobile for several
months. The radiograph indicates a normal level of bony support although the
periodontal space has widened. The apical bone appears normal. The tooth is
tender to pressure. Which of the following tests and or examinations would be
most likely to provide a diagnosis?
● Masticatory muscle palpation
● Electric pulp test
● Occlusal examination
● Ethyl chloride test
● Hot gutta percha application
Ans is C
BY DR.ABDULRAHMAN ALMUALM
MJDF MCQS WITH ANSWERS
1219A 23 year old male presents to your surgery. He lost his upper lateral
incisors some 10 years ago in a swimming pool accident. Since then he has
been wearing a 'spoon' denture which he now feels in aesthetically unacceptable.
He has sought an opinion on dental implants but has been told that he would
need bone grafting for this to be successful and he is not prepared to undergo
this. His dentition is excellent with no restorations and a Class I occlusion. He
wants some advice on what the best treatment might be. Which option would
you put first on your list of possibilities?
● Two fixed - fixed resin bonded bridges using the central and canine teeth
● Two cantilever resin bonded bridges from canines.
● Two conventional fixed - fixed bridges from the canine
● Conventional cantilever bridges from the canines
● Cobalt chrome partial denture
Ans is B
1221 55 year old female patient is missing her upper right second premolar
and upper right first molar and also is missing the upper left second molar. The
upper right second molar is functional and has an amalgam restoration (MOD
and buccal wall) that requires replacing. The patient has no functional or
aesthetic concerns. What would be the treatment of choice in this situation? ●
Provide an upper removable partial denture
● Replace the amalgam in the upper right 7 only >bcs patient has no funct and
esth concerns
● Provide a full coverage crown in the upper right 7
● Provide a fixed bridge in the upper right quadrant
● Provide a full coverage crown in the upper right 7 with guide planes and
occlusal rests
Ans A
1222 Endodontic treatment has failed on an upper first molar; the patient is
keen to retain the tooth. There are persistent symptoms from the tooth.
Radiographically there is evidence of periapical radiolucency although the three
canals are obturated with good length and compaction. What is the best course
of action?
● Extract the tooth
● Re-treatment with an iodine solution as irrigant
● Re-treatment with hypochlorite solution as irrigant
● Re-treat the tooth, looking particularly for additional canals
● apicectomy
BY DR.ABDULRAHMAN ALMUALM
MJDF MCQS WITH ANSWERS
Ans is D
1223 You suspect that there is occlusal caries in the lower right first permanent
molar of a 10 year old child. You wish to confirm your suspicions. Which
diagnostic test is most commonly used in this situation?
● Bitewing radiography
● Electro-conductive caries monitors
● Fibro-optic transillumination
● Panoramic radiography
● Visual examination of a dried tooth.
Ans E
1224 You notice that a 20 year old patient has marked tooth surface loss
associated with the labial and palatal aspects and incisal edges of the upper
anterior teeth. They are sensitive to hot and cold. The remainder of the dentition
is mainly unaffected What is the likely diagnosis?
● Attrition
● Active erosion >sensitivity
● Passive erosion
● Abfractions
● Abrasion
Ans is B
1225 The best radiograph for investigating the maxillary sinus is,
● Periapical radiograph
● Panoramic view
● Lateral cephaloghraph
● Occipitomental view
● Reverse Towne's view
Ans is D /waters view 30°
1226 1 In the preparation of Premolar class I cavity what is the best way of
getting retention,
● Slightly done undercut of the mesial and distal walls
● Slightly done undercut ofthe buccal and lingual walls
● The convergence of the cavity walls
Ans is C
BY DR.ABDULRAHMAN ALMUALM
MJDF MCQS WITH ANSWERS
1228 After the placement of a class I amalgam the patient comes back to you
complaining of pain on masticating and biting; what is the first thing you would look
at,
● Occlusal height
● Contacts areas
Ans A
1232 The ideal length of core in the fabrication of crown and core of
endodontically treated tooth is,
● 1.5 of crown length > core not post
● The length of the crown
● 2/3 tooth/root length
● D. 1/2, root length
Ans A
1233 If aesthetic is not a concern what is the first thing to do to treat soreness
under dentures,
● Take the denture off for a week
● Rinse the denture in nystatin
● Apply tissue conditioner
Ans A
BY DR.ABDULRAHMAN ALMUALM
MJDF MCQS WITH ANSWERS
1234 While you finishing a class I cavity, the enamel is sound but you noticed
in the dentine and on the Dento-enamel junction a brown line, what is your
response,
A. You leave it and complete the final restoration
B.You extend you preparation and clean it
C.You apply a cover of varnish
Ans A
1235 Dental caries of the proximal surfaces are usually start at,
● Somewhere between the ridge and the contact area
● Just gingival to contact areas
● Just about the gingival margin
Ans is B
1239 Which of the following local anaesthetic is indicated in case of the need
to long acting one after a surgical operation, /~
● Mepivacaine
● Bupivacaine (Marcaine)
Ans is B
BY DR.ABDULRAHMAN ALMUALM
MJDF MCQS WITH ANSWERS
1242 In preparing a very small proximal amalgam cavity on a molar tooth what
would consider,
● Extend the cavity to the gingival margin
● Extend the cavity beyond the contacts areas
● Achieve at least 2mm in dentine
● Extend cavity just beyond dento enamel junction
Ans is B
1245 The different between normal stone and the dye stone is,
● In the particles size
● The amount of water
Ans A
1246 A patient comes to you with medium pain of tooth filled with Composite
resin as a result of cold or hot drinks, what your initial management will be
, ● Remove the restorative material and start an Endontic treatment
● Remove the restorative material and place a sedative temporary material
● Place a coat of bonding material on the old composite
Ans C?
1247 Throbbing pain increases with heat and cold stimuli, the MOST probable
diagnosis is,
● Cyst
● Occlusal trauma
● Advanced pulpitis
Ans C
BY DR.ABDULRAHMAN ALMUALM
MJDF MCQS WITH ANSWERS
Ans A
2. IADT is saying
Milk is better then saliva (which is not recommended for children since they could
swallow the tooth )
or HSSb if it's there..
Based on those you have the right to select what you think is the most correct
1252 In the construction of partial denture the surveyor is not used to,
● Contour the wax as part of the fabrication of the working cast
● Locate the guide planes
● Determine the location of indirect retainers
● Identify any undesirable undercuts
Ans Is C
Surveyor can -outline desirable and undesirable undercuts -establish guide planes
-contour wax -block out undercuts -establish path of insertion -check parallelism
BY DR.ABDULRAHMAN ALMUALM
MJDF MCQS WITH ANSWERS
1255 Occlusal cavity with extension of the buccal fissure is classified as,
● Class II
● Class III
● Class I
Ans C
1256 Which of the following does not affect the elasticity of retentive clasp?
● Length of the arm
● The cross section shape
● The material used
● The undercut area
Ans is D
1257 A partial denture that seats on the master cast but fails to seat correctly
in the mouth is a result of,
● Contraction of the metal framework during casting
● Insufficient expansion of the investment material
● Distortion of impression
● Failure to block out unwanted undercuts
Ans is C
1258 Which of the following muscles may affect the borders of mandibular
complete denture?
● Mentalis
● Lateral pterygoid
. ●. Orbicularis oris
● Levator oris
● Temporalis
Ans A
1260
Which of the following statements is incorrect regarding Smoker's Keratosis?
● Typically affects the hard palate
● Minor mucous glands are swollen with red orifices
● There is a little regression if smoking is stopped
Ans is C
1270 13 years old boy comes to you with excessive hyperplasia of the gingiva
as a result of Phenytoin what is your management,
BY DR.ABDULRAHMAN ALMUALM
MJDF MCQS WITH ANSWERS
1271 White man 56 years old comes to you with a brown spot on his gingiva
and another one on his oral mucosa, when taking the history he mentioned a
weight and memory lost. He as well complains of headaches. What is your
most probable diagnosis,
● Addison's disease
● Hyperthyroidism
Ans A
1272While removing the second primary molar of 9 years old child, the apical
third of the root fracture and stay in the socket,
● You will just leave it and observe it
● You take surgically by a lingual flap
● You try to take out by using a root apex elevator
● You use a fine end forceps to take it out
Ans is A
1274 With view to Nitrous Oxide what is the major pharmacological problem?
● Contraindicated in pregnancy
● Contra indicated in cardiac dysrhythmias > reletive contraindaction
Ans A
1277 patient in your dental chair shows chest pain, weak pulse and dysponea,
what is your initial management,
● Give a nitro-glycerine tablet and keep the patient up seated'
● Put the patient in supine position
BY DR.ABDULRAHMAN ALMUALM
MJDF MCQS WITH ANSWERS
1279 Developer contaminated with other chemical and was not mixed pro
perlv. What is the effect on the X-ray film?
● Too dark film -
● Light film
● Foggy
Ans is C !!
1280 Which part of the cranium is considered as the most stable area,
● Frankfort plane
● Occlusal plane
● Anterior cranial plane
● Anterior nasal to gnathion.
Ans is C
1290 the most common way of oral carcinomas to other tissues is/are
● Lymphatic
● Invasion and blood
● Lymphatic and invasion
● Blood and lymphatic
Ans C
BY DR.ABDULRAHMAN ALMUALM
MJDF MCQS WITH ANSWERS
1293 The removable partial denture requires relining what is would be the most
appropriate action,
● take a new impression by asking the patient to occlude on it
● Provide equal space between denture and gingival tissues.
● Make sure the framework and retainers are seated in place before taking
impression
Ans is C
1295 Symptoms free patient comes to you after four weeks of an endodontic
treatment and you find on radiograph the canal is over filled with what it seems
to be a cone of Gutta Percha lmm beyond the apex with a radiolucent small
area. What is your initial management?,
● Start apiectomy through a flap and surgery
● Obturate the root canal
● Ask for a recall and observe in three months time
● Seal the pulp chamber and keep it under observation
Ans C
1296 After obturation and on X-ray you notice the obturation materials are lmm
beyond apex. What is your first management?
● Refill the canal
● Pull the GP cone about lmm out and take a new X-ray
● Leave it as it
Ans C
1297 Four years kid shows at your clinic with open bite as a result of thumb
sucking, you notice a delayed speech ability what would be your first
management,
● Refer to a speech therapist
● Apply a removable habit inhibitor denture
● Apply a removable habit inhibitor denture and educate the parents about it
so the kid will not be taking it off so often
Ans is A
1298 After the initial development stage and in the absence of pathology, the
size of the pulp chamber has been reduced by,
● Deposition of primary dentine
● Deposition of secondary dentine
● Reparative dentine
● Pulp fibrosis
Ans A
BY DR.ABDULRAHMAN ALMUALM
MJDF MCQS WITH ANSWERS
1303year old boy has a small white discolouration on his maxillary central
incisor. The lesion is most probably,
● Hypocalcification due to trauma of the primary predecessor
● Hypoplasia due to acute systemic infection when 6-12 months old
● Defect during the histo differentiation stage of development
● Defect during the morho differentiation stage of development
Ans A
1305Which of the following is not a part of the fully formed enamel organ,
● Outer enamel epithelium
● Inner enamel epithelium
Ans is B
BY DR.ABDULRAHMAN ALMUALM
MJDF MCQS WITH ANSWERS
1306.18 years old female her weight is 52Kg and she is 163cm tall. On dental
examination erosion of teeth on the most of her lingual surfaces is clearly
showing. Dietary history revealed a daily rate of 5000 to 7000 Kcal/day. What is
most probable would be her case?
● Alcoholism
● Drug abuse
● Bulimia
● Excessive smoking
● Diabetic mellitus type I
Ans is C
1310The principle muscle responsible for the opening of the mouth is,
● Mylohyoid
● Anterior temporal
● Posterior temporal
● I. Anterior belly of digastric
Ans D
BY DR.ABDULRAHMAN ALMUALM
MJDF MCQS WITH ANSWERS
● Macrodontia
● An increased susceptibility to periodontal disease
● Congenitally missing teeth
Ans C
131613 year old girl presents with an unerupted permanent canine and a
retained primary canine. You cannot palpate the unerupted canine in the buccal
sulcus and you are uncertain as to whether it is displaced palatally or in the
line of the arch. Which single radiographic view would be most helpful in
locating the unerupted tooth?
A. Bitewing
B.DPT
● Lateral oblique
● Single periapical
● Upper anterior occlusal
Ans E
BY DR.ABDULRAHMAN ALMUALM
MJDF MCQS WITH ANSWERS
131730 year old man with unknown allergy to latex goes into anaphylactic
shock whilst being treated in the dental surgery. Which drug and route of
administration is of most benefit in this situation?
● Hydrocortisone - orally
● Chlorphenamine - intramuscularly
● Chlorphenamine - orally
● Epinephrine - intravenously
● Epinephrine – intramuscularly
Ans E
131925 year old patient attends your surgery complaining of a swelling at the
angle of the mandible. A radiograph shows a uni-locular radiolucency associated
with the crown of an unerupted wisdom tooth. Which of the following is the most
likely diagnosis? ● Radicular cyst
● Dentigerous cyst
● Lateral periodontal cyst
● Ameloblastoma
● Odontogenic keratocyst
Ans is B
1320A 70 year old female, who suffers with persistent looseness of her lower
complete denture, is considered for implants. She will require radiological
evaluation of the potential implant sites. Which would be the most appropriate
radiological investigation at this stage?
● MRI scan of the mandible
● CT scan of the mandible > first choice is cbct then ct
● Periapical radiographs of the mandibular anterior region
● True lower anterior occlusal view
● Panoramic radiograph
Ans is B
BY DR.ABDULRAHMAN ALMUALM
MJDF MCQS WITH ANSWERS
1321Six months ago you saw a child patient, then aged 9 years. His upper
right maxillary canine was palpable in the labial sulcus but the upper left was
not. The situation is now unchanged, so you have taken two periapicals of the
non-palpable tooth. They both show that there is some resorption of the CI root
but the permanent canine appears somewhat mesially angled and is more
mesial on the more mesially positioned film. What is your the best course of
action?
● Keep a careful watch on it and take another x-ray in 6 months.
● Refer to an oral surgeon for early exposure of the permanent canine.
● Refer to an oral surgeon for early removal of the permanent canine before it
damages the lateral incisor.
● Refer to an orthodontist for a treatment plan.
● Wait and watch' until the child is 11. .
Ans D
132343year old patient is missing on the upper right the first premolar and
molar. He has good oral hygiene and requests a fixed replacement for these
teeth. The other teeth on the same side are all moderately restored with MOD
amalgam restorations and are vital, except the canine, which has a very large
restoration and is root-filled. He has group function. Radiographs show a large
sinus cavity and no peri-apical pathology. What would be the restoration of
choice for replacement of the missing teeth?
● Implant supported crowns
● A conventional fixed bridge using the 7 and 5 as abutments
● Two conventional cantilevered bridges, using the 7 and 3 as abutments
● A resin-bonded bridge, using the 7 and 5 as abutments
● A conventional fixed-moveable bridge using the 7 and 5 as abutments
Ans A.
BY DR.ABDULRAHMAN ALMUALM
MJDF MCQS WITH ANSWERS
132558 year old male presents at your surgery complaining of a sharp pain of
no more than 30 minutes duration arising from his upper left molar region. The
pain is brought on by cold stimuli but persists after-the stimulus is removed. It
does not seem to occur spontaneously. He has tried taking paracetemol and
this does temporarily stop the pain from recurring. The upper left 6' reacted to
a lower current on electronic pulp testing than the upper right 6, upper left ~
or the lower left molars. What is the most likely cause of the patient's pain? -
● Acute/reversible pulpitis
● Dentine sensitivity
● Chronic/irreversible pulpitis
● Periapical periodontitis
● Trigeminal neuralgia
1326 .A 58 year old male presents at your surgery complaining of a sharp pain
of no more than 30 minutes duration arising from his upper left molar region.
The pain is brought on by cold stimuli but persists after-the stimulus is
removed. It does not seem to occur spontaneously. He has tried taking
paracetemol and this does temporarily stop the pain from recurring. The upper
left 6' reacted to a lower current on electronic pulp testing than the upper right
6, upper left ~ or the lower left molars. What is the most likely cause of the
patient's pain? -
● Acute/reversible pulpitis
● Dentine sensitivity
● Chronic/irreversible pulpitis
● Periapical periodontitis
● Trigeminal neuralgia
Ans C
1327.60 year old patient attends your surgery complaining of a sore mouth. He
has Type II diabetes well controlled by diet and metformin. On examination
white patches which cannot be removed are present on his buccal mucosa:
What is the most likely diagnosis?
● Frictional keratosis
● Leukoplakia
● Lichen Planus
● White sponge naevus
● Candidosis
1328An 80 year old patient presents with an ulcer in the floor of the mouth.
This has been present for several months and has not responded to
conventional treatment. An incisional biopsy is taken. Which of the following
histological changes in the epithelium confirm a diagnosis of squamous cell
carcinoma?
● Hyperkeratosis
● Acanthosis
BY DR.ABDULRAHMAN ALMUALM
MJDF MCQS WITH ANSWERS
● Dysplasia
● Invasion
● Discontinuous epithelium
Ans D
1329 . A 35 year old patient complains of swollen gums. This has been present
for several years. What is the commonest cause of this complaint?
● Cyclosporin therapy
● Vitamin C deficiency
● Chronic Gingivitis
● Atenolol therapy
● E. Pregnancy
Ans C
1330A 45 year old patient attends the dental clinic complaining of a clicking
jaw. Examination reveals a reproducible click of the right TMJ when opening
wide. Upon asking the patient to open wide, close with incisors edge-to-edge
and then open and close to this position, the click is absent. From the options
below, which one is the most likely diagnosis?
● Myofascial pain
● Disc displacement with reduction
● Disc displacement without reduction
● TMJ osteoarthritis
● Arthralgia
Ans B
1332An adult patient attends your practice complaining of pain and swelling
associated with a previously restored upper first premolar tooth. The pain has
been present for a number of days and is no longer responding to analgesics.
His dentition is otherwise well maintained and his periodontal health is good.
What is the most appropriate approach to treatment?
● Antibiotics and analgesics.
● Extract the tooth
● Carry out a pulpotomy. Temporary dressing.
● Carry out a pulpectomy. Temporary dressing
● E. Establish open drainage
Ans D
BY DR.ABDULRAHMAN ALMUALM
MJDF MCQS WITH ANSWERS
1333patient says that he does not like the appearance of his previously root
filled upper central incisor tooth. His dentition is otherwise well maintained and
his periodontal health is good. The tooth appears to be darker than the adjacent
teeth. What is the most appropriate approach to treatment?
● Provision of a post crown
● Provision of an all ceramic crown
● Provision of a metal bonded to ceramic crown
● Carry out a non vital bleaching procedure
● Provision of a porcelain veneer
Ans D
133420 year old patient attends your surgery for the first time. You suspect
that he may have proximal caries as he has a frequent sugar intake. Which of
the following is the most accurate method of diagnosing proximal caries in a
lower molar tooth in this young adult? A. Clinical history
● Periapical radiograph
● Bitewing radiograph X
● Digital image
● Electronic resistance measurements
Ans B
133525 year old male attends for the first time complaining of sensitivity of a
number of teeth. On examination, the occlusal surfaces of all the teeth are
worn with obvious wear facets on the canines and premolars. Posterior
amalgam restorations are proud of the surrounding tooth. What would be the
first stage management?
● Take impressions for study models
● Prescribe fluoride mouth rinse
● Replace the amalgam restorations
● Dietary analysis
● Placement of resin sealant to sensitive teeth
Ans is D //☆ some drs answered E
133635 year old male patient who admits to grinding his teeth at night has a
number of wedge- shaped cervical (Class V) lesions on his upper premolar
teeth. These are causing some sensitivity and are approximately 3mm deep.
What is the correct management option?
● Provide tooth brushing instruction and fluoride
● Restore the lesions with compomer
● Restore the lesions with micro-filled composite
● Restore the lesions with a hybrid composite
E. Restore the lesions with conventional glass-ionomer
Ans C /* some answered B
1337patient attends with pain of four days duration in a carious upper molar
tooth. The pain is constant and is not relieved by paracetemol. Sleep has been
disturbed by the pain. The tooth is tender to percussion and gives a positive
response to Ethyl Chloride. What is the most likely diagnosis? .,
● Pericoronitis
BY DR.ABDULRAHMAN ALMUALM
MJDF MCQS WITH ANSWERS
● Apical periodontitis
● Marginal periodontitis
● Reversible pulpitis
● Irreversible pulpitis
Ans B?
1338You are trying in a partial chrome denture framework which fails to seat
properlv. It fits the master cast. What is the most likely cause of this problem?
● Insufficient expansion of the investment material
● Distortion of the impression
● Contraction of the metal framework during casting
● Failure to block out unwanted undercuts
● Complex denture design
Ans B
1340You are interested in finding out what the risk indicators are for a rare
form of oral cancer and decide to undertake a study to examine this. What
type of study would be the most appropriate for addressing this issue?
● Cohort
● Prevalence study
● Clinical trial
● Case-control study
Ans D
BY DR.ABDULRAHMAN ALMUALM
MJDF MCQS WITH ANSWERS
Ans B
1342You take a panoramic radiograph out of a patient's records but you find
that the film has a low density and poor contrast. Which of the following errors
could lead to low density and poor contrast?
● Too long a development time > poor contrast dark film or high density
● Developer temperature too high >poor contrast dark film
● Developer temperature too low> poor contrast pale or low density film
● Inadequate fixation > poor contrast
● poor storage > poor contrast
Ans C
BY DR.ABDULRAHMAN ALMUALM
MJDF MCQS WITH ANSWERS
134514 year old patient attends with a decayed and extensive hypoplastic LL7:
He is a very irregular attender with poor oral health habits. A radiograph shows
the presence of an unerupted LL8 and the LL6 is sound. What would be the
most appropriate long-term treatment for this tooth?
● Amalgam restoration
● Antibiotics
● Extraction
● Root canal therapy
● Sedative dressing
Ans C
13467 year old boy has previously had all primary molars restored and a
pulpotomy on upper right E. He has an early mixed dentition with lower lateral
incisors erupting. There is a midline diastema of 2 mm. The upper right E has
become symptomatic and requires extraction.The most likely long term effect of
the extraction on the occlusion is:
● Early eruption of the second premolar.
● Loss of upper central line.
● No significant effect.
● Overeruption of the lower right teeth.
●potential crowding of the right upper quadrant
.1347 Which of the following is the ideal treatment for a degree II furcation
involvement of a mandibular molar?
● Tunnel preparation
● Root resection
● Furcation plasty
● Extraction
● Guided Tissue Regeneration
Ans is E
1348You examine a patlent and find BPE code 4 in all sextants. Radiographs
show generalised horizontal bone loss with a minimum of 50% of bone support
remaining on all teeth. Which of the following is the most important factor when
considering the prognosis for the teeth?
● Age of the patient
● The Oral Hygiene Status
● Bleeding on Probing Score
● Mobility
● Gingival Recession
1349A mother is concerned that her child's adult upper front teeth have not
erupted and asks your advice. What is the usual age in years for the upper
permanent central incisors to erupt?
● 5 years.
● 6 years.
● 7 years.
● 8 years.
● 9 years.
Ans is C
BY DR.ABDULRAHMAN ALMUALM
MJDF MCQS WITH ANSWERS
1350You decide to refer an eight year old child to the oral surgery department
in your local hospital for extractions under a general anaesthetic. What key
reason for asking for a general anaesthetic would you put in the referral letter?
● Parents request GA.
● Failed to complete treatment under inhalation sedation.
● Child would not accept local analgesia
● Parents think hospital more convenient.
● Not one of my regular patients.
Ans B
1351 An eight-year-old boy presents with pain of three days duration that has
kept him awake. On examination you see a grossly carious lower left 6 and
some associated buccal swelling. Which of the following is the most appropriate
to give immediate relief of his pain?
● Extract the LL6.
● Gently excavate the. caries and obtain drainage.
● Give antibiotics.
● Incise any swelling
● Refer for-general anaesthetic
Ans is B
135210 year old girl tripped and-knocked out her upper incisor 10 minutes ago.
She is holding it in her hand. What is your most effective immediate treatment?
● Accept the tooth is lost and do nothing.
● Refer to specialist.
● Re-plant immediately.
● Root fill, clean and re-plant.
● Sterilise root in alcohol and re-implant.
Ans C
1353You are treating a patient who has a long history of recurrent episodes of
oral candidosis. His mouth has recently become sore once again and you want
to prescribe some antifungals for him. You check his medical history and find
that he suffers from atrial fibrillation and is taking warfarin. Which of the
following drugs would be most appropriate to treat this patient's candidosis? ●
●Metronidazole
● Nystatin
● Fluconazole
● Miconazole
● Amoxycillin
Ans is B
BY DR.ABDULRAHMAN ALMUALM
MJDF MCQS WITH ANSWERS
1357 53 year old male patient presents with an asymptomatic white patch on
the ventral surface of his tongue. He has smoked 35 cigarettes a day since he
was 17. An incisional biopsy of the white patch is likely to show which
histological feature:
● Basal cell liquefaction
● Hyperkeratosis
● Saw tooth rete ridges
● Acantholysis
E. Basal cell hypoplasia
Ans B
135846 year old female presents withi a slowly enlarging painless firm swelling
in the hard palate to the left of the midline. The most likely diagnosis is:
● A dental abscess
● Torus palatinus
● Osteoma
● Pleomorphic adenoma
● Canalicular adenoma
BY DR.ABDULRAHMAN ALMUALM
MJDF MCQS WITH ANSWERS
Ans D
135924 year old man presents with a loculated cystic radiolucency in the lower
third molar area approximately 3cm in diameter. nThere is no tooth associated
with the cyst and there is no bucco-lingual expansion of the mandible. There
is no resorption of the roots of the overlying second molar tooth. How would this
be treated?
● Marsupialisation
● Enucleation and ethyl chloride
● Block resection
● Enucleation and Carnoy's solution >keratocyst
● Enucleation and formalin solution
Ans D
1360year old female presents for the first time to your practice. She is very
upset with the appearance of her upper left central incisor. On examination you
find healthy oral hard and soft tissues and excellent oral hygiene. On close
examination you can see that the upper left central incisor is slightly greyer than
the upper right central incisor and has a composite restoration placed palatally.
What is the most appropriate form of treatment given the information you
have? ● Bleaching with carbamide peroxide in custom formed trays of upper
and lower arches
● A bonded crown
● A composite veneer
● A porcelain veneer
● Non-vital bleaching with carbamide peroxide
1363n dental epidemiology, indices are used to measure the oral health of a
population. The DMF index is commonly used to measure the prevalence and
severity of dental caries. What is the main limitation of this approach?
● It does not allow statistical analysis.
BY DR.ABDULRAHMAN ALMUALM
MJDF MCQS WITH ANSWERS
1364The biological process by which the architecture and function of the lost
tissue is completely restored". In Periodontology this is a definition of which of
the following
● New Attachment
● Regeneration
● Repair
● Primary Healing
● Reattachment
Ans B
1366An obese 40 year old who takes metformin is seen at the end of a
morning clinic. After administration of local analgesia in the supine position the
patient complains of feeling unwell. She is pale and sweating and is confused.
No pulse was detectable. She was given oxygen and maintained in the supine
with no improvement in condition. What would be the most appropriate drug to
administer next?
● Glucagon
● Glucose
● Adrenaline
● Hydrocortisone
● Chlorphenamine
Ans B
136712 year old boy is brought in by his mother who is concerned about the
appearance of his central and lateral maxillary incisors. She says they look like
they might be decayed. They are a new family to the practice having recently
moved to the area from Birmingham. On examination you notice a generalised
whitening of the incisors which seems to worse at the incisal edges. After you
BY DR.ABDULRAHMAN ALMUALM
MJDF MCQS WITH ANSWERS
have dried the teeth you notice that the whiteness appears to be linear running
from mesial to distal. There is a similar pattern on the lower incisors. What is
the most likely explanation for the presentation of the boy's incisors?
● Tetracycline staining
● Early caries (white spots)
● Fluorosis
● Demarcated Enamel Opacities
E. Previous treatment with composites
Ans C
1369A child of 5 years attends with pain from a grossly decayed lower right D
which has a discharging sinus. He is a hemophiliac. Which of the following is the
most appropriate treatment to relieve his pain?
● Antibiotics
● Extraction
● Fluoride application
● Non-vital pulpotomy
● Vital pulpotomy
Ans is D (its pulpectomy #!)
1370You are designing a partial denture for a patient with several missing teeth
in the maxilla. The reason for surveying the model prior to designing the
denture is to:
● Measure and mark out hard and soft tissue undercuts on the casts
● Relate the intended position of the inter-papillary plane of the patient to the
casts
● Establish the position of the post dam
● Relate the maxillary and mandibular casts
● Aid setting up the prosthetic teeth prior to trial insertion
Ans A
BY DR.ABDULRAHMAN ALMUALM
MJDF MCQS WITH ANSWERS
1371The parotid gland is one of the major salivary glands that supply the oral
cavity. Where is the orifice of the duct of the parotid gland located?
● At the hamular notch
● In proximity to the incisive papilla
● On the buccal mucosa near the maxillary second molar
● Slightly posterior to the mandibular central incisors
● Distal to the maxillary third molars on the palatal side
Ans C
1372Any patient receiving treatment under IV sedation must have their blood
pressure recorded as part of their assessment. What is the maximum blood
pressure that is generally regarded as being compatible with safe sedation in
general dental practice?
A.160/95
B.140/95
● 160/90
● 170/100
● E.120/80
Ans D
Ans is E
BY DR.ABDULRAHMAN ALMUALM
MJDF MCQS WITH ANSWERS
13743 year old child attends, showing evidence of bottle caries and also decay
affecting primary molars. The child's mother has heard that some sugars are
damaging her child's teeth. She is not sure which foods to avoid. From the
following, which are you most likely to suggest she avoids?
● Bread
● Cheese
● Ground almonds
● Pure fruit juices
● Whole fruits and vegetables
Ans D
1375An adult patient complains of her prominent upper front teeth and
receding chin. She says she did not get 'braces' when she was younger
because the family had to move area a lot. Her oral care and health is good,
and she has a 9 mm overjet. Where is the most suitable place to refer her? "
● A GDP friend who has a special interest in orthodontics and has been
trained to use fixed appliances.
● A specialist practitioner who uses a lot of functional appliances
● A private specialist practitioner, because she's too old to get NHS treatment
now.
● A hospital consultant, as it is likely she will need surgery now to correct her
problem.
● The nearest dental hospital, although it is 70 miles away.
Ans D
1376You are meeting a three year old patient for the first time. His mother
explains that he had a unilateral cleft lip and palate that was repaired in
infancy. She has heard that orthodontics will usually be required when he is
older. She asks you what is the commonest orthodontic problem that occurs
with a repaired cleft palate. What.:.is the most appropriate answer?
● A contracted maxillary arch -
● An anterior open bite.
● An elongated maxillary arch.
● An expanded maxillary arch
. ● An increased overjet
Ans is A
BY DR.ABDULRAHMAN ALMUALM
MJDF MCQS WITH ANSWERS
1378Gingivitis and periodontitis are associated with the loss of normal tissue.
Which of the following is the principal cause of the tissue loss
● Bacterial enzymes such as collagenase and hyaluronidase
● Release of lysosomal enzymes and oxygen free radicals by PMNs and
macrophages
● Cytotoxic metabolic by products such as ammonia, hydrogen sulphide and
toxic amides
● Complement activation
● Release of bacterial endotoxins and exotoxins.
Ans D
138048 year old woman complains of a sore area on the right buccal mucosa
adjacent to a restored tooth. The lesion has a lichenoid appearance and this is
confirmed histopathologically following a biopsy. Which of the following
restorative materials is most frequently associated with lichenoid changes.?
● Gold
● Amalgam
● Porcelain
● composite
● Glass ionomer cement
Ans is B
138255 year old female presents at your surgery complaining of a sharp pain
of no more than a few minutes duration arising from her lower incisors. The
pain only occurs when she eats or drinks cold or sweet foods and only lasts for
BY DR.ABDULRAHMAN ALMUALM
MJDF MCQS WITH ANSWERS
Ans is B
1384A 50 year old male patient has a Class III jaw relationship with an anterior
open bite. It is planned to restore his lower right second molar, which has
suffered tooth wear and fracture, with an indirect restoration. This tooth has
approximately 2mm of coronal height. What would be the most suitable
approach to restore this tooth?
● Provide an adhesively retained gold onlay
● Provide a conventional full crown
● Increase the vertical dimension and provide a full crown
● Surgically crown lengthen and provide a gold crown
● Provide an adhesively retained ceramic onlay
Ans is A
BY DR.ABDULRAHMAN ALMUALM
MJDF MCQS WITH ANSWERS
1388patient complains of a lower incisor which has been mobile for several
months. The radiograph indicates a normal level of bony support although the
periodontal space has widened. The apical bone appears normal. The tooth is
tender to pressure. Which of the following tests and or examinations would be
most likely to provide a diagnosis?
● Masticatory muscle palpation
● Electric pulp test
● Occlusal examination >occlusal trauma
● Ethyl chloride test
● Hot gutta percha application
Ans is C
1389nine year old boy presents with a class I occlusion with no crowding or
overjet with a grossly carious upper left 6 which is not suitable for restoration.
The upper left 7 is very near to eruption. Upper right 6, lower right 6 and lower
left 6 are sound and fissure sealed. The oral hygiene is good. What is the most
appropriate extraction pattern for this patient?
● Extract all four 1st permanent molars
● Extract upper left 6 and lower left 6
● Extract upper left 6 and lower right 6
● Extract upper right 6 and upper left 6
● E. Extract upper left 6 only
BY DR.ABDULRAHMAN ALMUALM
MJDF MCQS WITH ANSWERS
Ans E
1390 new patient attends your practice. At the initial examination, you carry out
a Basic Periodontal Examination (BPE). What does a BPE score of 1 signify?
● Periodontal health
● Probing depths greater than 3.5mm
● Presence of overhanging restorations
● Presence of supragingival calculus
● Presence of bleeding on probing
Ans is E
1392All the films which have been manually processed by an unsupervised new
trainee dental nurse in your practice are extremely dark. Which of the following
errors could lead to a dark film?
● Insufficient exposure
● Films have been left in the developer for too long a period of time ● Films
have been in the fixer for too long a period of time
● The concentration of the developer is too dilute
● The concentration of the fixer is too dilute
Ans is B
1393your colleague is having problem as every panoramic film that he produce
shows extremely wide anterior teeth which are blurred. The film also invariably
has the images of the condyles cut off from the sides of the film. The most
likely cause of this?
● The patient is incorrectly positioned too far forward relative to the image layer
● The patient is incorrectly positioned with the chin too low
● The patient is incorrectly positioned too far back relative to the image layer
● D. The patient is in a slumped position in the machine
Ans C
When patient too far back- anterior wide teeth plus lateral side of condyle cut off.
When patient chin slumped down -condyle HEADS are cut off
BY DR.ABDULRAHMAN ALMUALM
MJDF MCQS WITH ANSWERS
1396A patient presents with a chief complaint of “severe pain in my right ear” which
began when eating, three hours ago. An examination reveals tenderness over the
right preauricular region, maximum interincisal opening of 21mm with deflection to
the right, right lateral excursion of 9mm and left lateral excursion of 2mm. The most
likely diagnosis is
BY DR.ABDULRAHMAN ALMUALM
MJDF MCQS WITH ANSWERS
d) 4.4 ml
e) 5.5 ml
ANs D
BY DR.ABDULRAHMAN ALMUALM
MJDF MCQS WITH ANSWERS
d) 150 mg
e) 90 mg
1406A patient with BPE 4 due to poor filling; the best action is to:
a) Refer him to specialist
b) Treat him in the clinic
c) Refer him to a college who has special interests in periodontal treatment
d) Give him antibiotics
e) BPE4 necessitate extraction
140716 years old female patient presented with 4.5 mm open bite, 4 mm reverse
overjet
and impacted upper canine, his IOTN is:
a) 4e
b) 4b
c) 4a
d) 5i
e) 4t
1408The radiograph of choice for a new adult patient with BPE 4* in all sextants and
proximal decay in lower 1st molars is:
a) Full mouth vertical bitewings
b) Panorama with selected periapicals
c) Panorama with full mouth periapicals
d) Panorama with full mouth periapicals and selected bitewings
e) Full mouth periapicals
BY DR.ABDULRAHMAN ALMUALM
MJDF MCQS WITH ANSWERS
1411the term symblephron describes unusual sign of the eye and it is mostly
associated with
:
a) Behcet’s syndrome
b) MAGIC syndrome
c) Mucous membrane pemphegoid
d) Marfan syndrome
e) Pemphigus vulgaris
1412Patient presented with flaccid bulla affecting wide area of his back and his
mouth, he
also complaints of eye manifestations:
a) Pemphigus vulgaris
b) Mucous membrane pemphegoid
c) Bullous pemphigoid
d) Cicatricle pemphegoid
e) Behcet syndrome
1413For a patient with ulcer, the most common area in which the ulcer may change
to
malignancy is
a) Upper eye lid
b) Lower eye lid
c) Upper lip
d) Lower lip
e) Nose
1414Which of the following is mostly related to the strength of the the local anaesthia
effect
a) lipid solubility
b) weight of the patient
c) volume of LA that has been given
d) PKa must be less than 7
e) adrenalin content
1416female patient was very angry and has a very painful tooth necessitate
extraction; she
insist on extraction under general anesthesia
a) extract it under general anesthesia
BY DR.ABDULRAHMAN ALMUALM
MJDF MCQS WITH ANSWERS
1417Sudden acute unilateral swelling of the parotid gland which is not fluctuant:
a) Sialosis
b) Mumps
c) Acute sialedinitis
d) Pleomorphic adenoma
e) Salivary stone
14189 years old child presented with lymphadenopathy and petechae on the junction
between hard and soft palate. most likely causative virus is:
a) EBV
b) Herpes zoster
c) Paramyxovirus
d) Coxacki A
e) HSV
1422elderly man with ulcer on the lateral surface of the tongue, the patient is heavy
smoker and heavy drinker; clinically he has loose lower partial denture; the 1st action
:
a) reline the denture
b) refer to the oral surgery clinic
BY DR.ABDULRAHMAN ALMUALM
MJDF MCQS WITH ANSWERS
c) ease the denture and review the patient after one week
d) take a biopsy to exclude malignancy
e) reassure him and give antifungal cream
1426The escort's most important role for patient receiving IV sedation will be :
a) Drive the patient home at end of the operation
b) Stay 24 hour with him for observation
c) Prevent him from smoking or drinking alcohol
d) Talk to do doctors and take a written cons
1428Got a call from a mother of 3 years old child who had trauma and avulsed his
front
tooth, what instruction will you give to the mother?
a) Attend as soon as possible with the tooth
b) Place the tooth in milk and attend as soon as possible
c) Place the tooth in salty water and attend as soon as possible
d) Put the tooth back in its position and attend as soon as possible
e) Come at the end of the day
BY DR.ABDULRAHMAN ALMUALM
MJDF MCQS WITH ANSWERS
1431The antibiotic of choice for patient with aggressive periodontitis who is allergic
to
penicillin should be:
a) Metronidazole 200 mg for 7 days
b) Doxycycline 100 mg for 21 days
c) Doxycycline 100 mg for 7 days
d) Clindamycin 300 mg for 7 days
e) Doxycycline 100 mg + metronidazole 200 mg for 7 days
BY DR.ABDULRAHMAN ALMUALM
MJDF MCQS WITH ANSWERS
a. Malignant lymphoma
b. Pleomorphic adenoma
c. Lymphangioma
d. Sialosis
e. mumps
1435patient with unilateral parotid gland swelling and weak facial nerve, mostly has
a) Bell’s palsy
b) Pleomorphic adenoma
c) Mucoepedirmoid carcinoma
d) Bacterial sialdenitis
e) Mumps
BY DR.ABDULRAHMAN ALMUALM
MJDF MCQS WITH ANSWERS
.1441 The most common cause of failure of the IDN “Inferior Dental Nerve”
block is
A. Injecting too low
B. Injecting too high
1445A 45 year old patient awoke with swollen face, puffiness around the
eyes, and oedema of the upper lip with redness and dryness. When he went
to bed he had the swelling, pain or dental complaints. Examination shows
several deep silicate restorations in the anterior teeth but examination is
negative for caries, thermal tests, percussion, palpation, pain, and periapical
area of rarefaction. The patient’s temperature is normal. The day before he
had a series of gastrointestinal xrays at the local hospital and was given a
clean bill of health. The condition is
A. Acute periapical abscess
B. Angioneurotic oedema
C. Infectious mononucleosis
D. Acute maxillary sinusitis
E. Acute apical periodontitis
1446Internal resorption is
A. Radiolucency over unaltered canal
B. Usually in a response to trauma
C. Radiopacity over unaltered canal
BY DR.ABDULRAHMAN ALMUALM
MJDF MCQS WITH ANSWERS
1452The nerve supply of the pulp is composed of which type of nerve fibres
A. Afferent & sympathetic
1453In which direction does the palatal root of the upper first molar usually
curve towards
A. Facial / buccal/
B. Lingual
C. Mesial
D. Distal
1455Which of the following would be ONE possible indication for indirect pulp
capping
BY DR.ABDULRAHMAN ALMUALM
MJDF MCQS WITH ANSWERS
1456Following trauma to tooth, the next day there was no response to pulp
tests you should
A. Review again later
B. Start endodontic treatment
C. Extraction of tooth
BY DR.ABDULRAHMAN ALMUALM
MJDF MCQS WITH ANSWERS
146265 year old woman arrived for dental therapy. The answered
questionnaire shows that she is suffering from severe cirrhosis.
The problem that can be anticipated in the routine dental therapy is
A. Extreme susceptibility to pain
B. Tendency towards prolonged haemorrhage
C. Recurring oral infection
D. Increased tendency to syncope
E. Difficulty in achieving adequate local anaesthesia
1464Patient received heavy blow to the right body of the mandible sustaining
a fracture there. You should suspect a second fracture is most likely to be
present in
A. Symphysis region
B. Left body of the mandible
C. Left sub-condylar region
D. Right sub-condylar region
E. sub-condylar region
BY DR.ABDULRAHMAN ALMUALM
MJDF MCQS WITH ANSWERS
BY DR.ABDULRAHMAN ALMUALM
MJDF MCQS WITH ANSWERS
1474In regards to HIV infection, which of the following is the earliest finding
A. Kaposi sarcoma on the palate
B. Reduced haemoglobin
C. Infection with pneumocystic carinii
D. Reduction in white cells count
E. B cell lymphoma
BY DR.ABDULRAHMAN ALMUALM
MJDF MCQS WITH ANSWERS
BY DR.ABDULRAHMAN ALMUALM
MJDF MCQS WITH ANSWERS
1485A patient who has been taking quantities of aspirin might show increased
post operative bleeding because aspirin inhibits** A. Synthesis of thromboxane
A2 and prevents platelet aggregation
B. Synthesis of prostacyclin and prevents platelet aggregation
C. Synthesis of prostaglandin and prevents production of blood platelets
D. Thrombin and prevents formation of the fibrin network
E. Increase the absorption of vitamin K and prevents synthesis of blood
clotting factors
1486A patient who recently had a calculus removed from the kidney
presented with radiolucent area in the left maxilla with clinical evidence of
swelling. The disease that you would immediately suggest is
A. Diabetes
B. Thyrotoxicosis
C. Hyperparathyroidism
D. Osteoporosis
E. Adrenal insufficiency
1488The patient whom you are about to treat, states that he has Von
Willbrand’s disease. Which one of the following preoperative hematological
analysis may reflect this disease
A. Bleeding time and factor VIII level
B. Bleeding time and factor IX level
C. Bleeding time and factor X level
D. Platelet count
E. Thromboplastin generation time
BY DR.ABDULRAHMAN ALMUALM
MJDF MCQS WITH ANSWERS
1494You wish to purchase a dental X ray machine and have the choice
between 60kVp and 70kVp machines. With single change from 60kVp to
70kVp what would the approximate affects on exposure time
A. No effect
B. Half the time
C. Double
D. Quarter
E. Triple the time
BY DR.ABDULRAHMAN ALMUALM
MJDF MCQS WITH ANSWERS
1495You wish to purchase a dental X ray machine and have the choice
between 60kVp and 70kVp machines. With single change from 60kVp to
70kVp what would the approximate affects on exposure time
A. No effect
B. Half the time
C. Double
D. Quarter
E. Triple the time
14988 years old child who has sustained a fracture of maxillary permanent
central incisor in which 2mm of the pulp is exposed; presents for treatment
three hours after injury. Which of the following should be considered
A. Remove the surface 1-2 mm of pulp tissue and place calcium hydroxide
B. Place calcium hydroxide directly on the exposed pulp
C. Pulpotomy using formocresol
D. Pulpectomy and immediate root filling
E. Pulpectomy and apexification
1499When primary molars are prepared for stainless steel crowns should the
depth for reduction of the proximal surface be similar to the depth of the
buccal and lingual surfaces
A. Yes; reduction of all wall is similar for best retention
B. No, proximal reduction is greater to allow the crown to pass the contact
area
C. No, the buccal surfaces has the greatest reduction to remove the cervical
bulge
D. Yes, all undercuts are uniformly removed so that the steel crown can be
seated
E. No, because of lateral constriction, the lingual surface needs greatest
reduction
BY DR.ABDULRAHMAN ALMUALM
MJDF MCQS WITH ANSWERS
1500Which primary teeth are LEAST affected with the nursing bottle syndrome
A. Maxillary molars
B. Maxillary and mandibular canines
C. Mandibular incisors
D. Maxillary incisors
E. Mandibular molars
1503A health 6 year old child presents with carious maxillary second primary
molar with a necrotic pulp. Which treatment would be preferred
A. Extraction
B. Indirect pulp treatment
C. Pulpotomy
D. Pulpectomy
E. Antibiotic coverage
BY DR.ABDULRAHMAN ALMUALM
MJDF MCQS WITH ANSWERS
1510Probe pressure at the sulcus of pocket should not be more than enough
to
A. Feel the top of the crestal bone
B. Balance the pressure between fulcrum and grasp
C. Define the location of the apical and the calculus deposit
D. Feel the coronal end of the attached tissues
BY DR.ABDULRAHMAN ALMUALM
MJDF MCQS WITH ANSWERS
1512Of all the factors that increase the resistance of teeth to dental caries
THE MOST EFFECTIVE is
A. The general nutrition of a child during the period of tooth formation
B. The intake of fluoride during the period of enamel mineralization and
maturation
C. Periodic topical fluoride application by dental health care following tooth
eruption
D. Sufficient intake of calcium and Vitamin D during the period of enamel
mineralization and maturation
1514 76. When describing a removable partial denture, the minor connector refers to**
BY DR.ABDULRAHMAN ALMUALM
MJDF MCQS WITH ANSWERS
1515. The means by which one part of a partial denture framework opposes the action
of the retainer in faction is**
A. Tripoding
B. Reciprocation
C. Stress breaking
D. Indirect retention
1516. In removable partial denture, the principle of an indirect retainer is that
1517. Distortion or change in shape of a cast partial denture clasp during its clinical
use probably indicates that the
1518. Which of the following is true regarding preparation of custom tray for
elastomeric impression
BY DR.ABDULRAHMAN ALMUALM
MJDF MCQS WITH ANSWERS
1519. When a removable partial denture is terminally seated ; the retentive clasps tips
should
1520. Why do you construct a lower removable partial denture with lingual bar
A. It is used when the space between raised floor, mouth and gingival margin is
minimal
B. Plaque accumulation is less than lingual plate
C. Should be make thicker when short
1522. Which of following restoration material its strength is not effected by pins
A. Amalgam
B. Composite resin
BY DR.ABDULRAHMAN ALMUALM
MJDF MCQS WITH ANSWERS
A. Hyperparathyroidism
B. Von Willebrand’s syndrome
A. It is a soft film composed mainly of food debris and can not be rinsed off teeth
B. It is a soft film composed mainly of food debris and can be rinsed off teeth
C. It is a soft film composed mainly of none calcified bacteria and can not be rinsed
off the teeth
D. It is a soft film composed mainly of dextran and can not be rinsed off the teeth
E. It is a soft film composed mainly of dextran and can be rinsed off teeth.
1527 The gingivae of child is diagnosed on the basis of all of these except of
BY DR.ABDULRAHMAN ALMUALM
MJDF MCQS WITH ANSWERS
1529.In regards to the glass of quartz particles of filling restorative resin; the microfill
resins tend to have
BY DR.ABDULRAHMAN ALMUALM
MJDF MCQS WITH ANSWERS
A. The maximum stress under tension that can be induced without failure
B. The maximum elongation under tension that can be measured before failure
C. The minimum stress required to induce permanent deformation of a structure
D. Minimum stress in structure
E. Maximum strain that can be measured.
1533.Denture resin are usually available as powder and liquid that are mixed to form
a plastic dough; the powder is referred to as**
A. Initiator
B. Polymer
C. Inhibitor
D. Monomer
E. Dimer
1534. Which one of the following is the major disadvantage of stone dies used for
crown fabrication
BY DR.ABDULRAHMAN ALMUALM
MJDF MCQS WITH ANSWERS
A. Acid-Base reaction
B. Addition polymerisation reaction
C. Growth of glass crystals
D. Slip plane locking
E. Solvent evaporation
1536.The articular surface of the normal temporomandibular joint are lined with
1538.Which one of the following types of pain is most likely to be associated with
cranio mandibular disorders
BY DR.ABDULRAHMAN ALMUALM
MJDF MCQS WITH ANSWERS
A. Mechanical equivalent of horizontal and vertical overlap of upper and lower incisors
B. Mechanical equivalent at the compensating curve
C. Same as condylar guidance
D. Estimated by the equation: Incisal guidance = 1/8 of condylar guidance
1540.When immature permanent molars that have been treated with Ledermix pulp
capping, the most probable pathology is
1541.Child with rampant caries taking medicine with high quantity of sugar; the best
way to help preventing caries is
1542. How many ppm “ Part Per Million” of fluoride are present in water supply in case
of temperate climate**
A. 1 ppm
B. 2 ppm
C. 8 ppm
BY DR.ABDULRAHMAN ALMUALM
MJDF MCQS WITH ANSWERS
D. 1.2 ppm
A. Deciduous teeth have a higher pulp horns and larger pulp chambers
B. Deciduous teeth have flatter contact areas
C. Deciduous teeth have thinner enamel surface
D. All of the above
1544. The most resistant filling materials to fill class IV cavities are
1547. The method you will use to fill root canal of maxillary lateral incisor is
BY DR.ABDULRAHMAN ALMUALM
MJDF MCQS WITH ANSWERS
B. Laterally condensed
C. Laterally above condensed
A. Teeth
B. Receptors in periodontal membrane
C. Proprioceptors
D. Neuromuscular receptors
E. TMJ
F. All of the above
A. Rotation
B. Lingually
C. Labially
1550.Why the method of extracting lower 8’s by directing the extraction lingually is
used**
A. 5 ml
BY DR.ABDULRAHMAN ALMUALM
MJDF MCQS WITH ANSWERS
B. 10 ml
C. 50 ml
D. 100 ml
A. Resin-metal
B. Resin enamel
C. Resin layer
Ans A
1553What is the function of gypsum-binder in the investment**
1554. Where is the retentive position on tooth according to the survey line
A. Will require relining more often than a denture supported with teeth
1559.What are the most common errors when constructing partial denture
A. Improper survey
B. Bad positioning of the occlusal rests
BY DR.ABDULRAHMAN ALMUALM
MJDF MCQS WITH ANSWERS
C. Incorrect design
Ans B
A. Buccal pockets
B. Lingual pockets
C. Mesial pockets
D. Distal pockets
E. Sinuous
Ans D (also C is correct (
1561.. What factor do you consider the most important when storing the occlusal part
of a tooth
A. Occlusal anatomy
B. Function
Ans A
1562. All dental plaque**
A. Produce acid
B. Produce caries
C. Produce chelation
D. Not necessarily produce acid
Ans A
1563. Treatment of gangrenous tooth
A. Pulp capping
B. Root canal therapy
C. Pulpotomy
Ans B
BY DR.ABDULRAHMAN ALMUALM
MJDF MCQS WITH ANSWERS
A. Bone resorption
A. Diazepam
B. Carbamazepine (Tegretol)
C. Ergotamine
D. Phenytoin
A. Lingual nerve
B. Long buccal nerve
BY DR.ABDULRAHMAN ALMUALM
MJDF MCQS WITH ANSWERS
1568.in an X ray; the mesio buccal root of upper first molars is elongated which is the
result of
A. May be submucous
B. More common in males than females
C. Predispose to speech defects, orthodontics problem and hearing loss
D. Patients are more likely to have cardiovascular defect than the general population.
BY DR.ABDULRAHMAN ALMUALM
MJDF MCQS WITH ANSWERS
1573.Labially displaced anterior tooth is restored with a gold core porcelain jacket
crown so that it is in line with the arch; the crown will appears
A. Short
B. Long
C. Narrow
D. Wide
BY DR.ABDULRAHMAN ALMUALM
MJDF MCQS WITH ANSWERS
C. 3 to 4 days
D. 3 to 4 weeks
E. Not at all
1577.Caries which is close to the pulp chamber; on x rays you find dent in dent; the
right treatment is
A. Chelation
B. Dental caries
C. Acids
A. Friction locked
B. Too expensive
C. Not all sizes available
D. May cause tooth cracking
BY DR.ABDULRAHMAN ALMUALM
MJDF MCQS WITH ANSWERS
A. I
B. II
C. IV
D. III
E. V
1582. How much space do you need to cap a weakened cusp with amalgam
A. 1mm
B. 1.5mm
C. 2mm
D. 2.5mm
1583. Upper premolar with MO cavity; what is important about the application of the
matrix band: “the question has shown too as
….What is complicated by”
BY DR.ABDULRAHMAN ALMUALM
MJDF MCQS WITH ANSWERS
A. Agranulocytosis
B. Candida Albicans
A. Pain
B. Bleeding
C. Paraesthesia
1588. Why Class IV gold can not be used in cavity as a filling material
A. Hard 18%
B. Type IV 75%
1590.In regards to Partial dentures, how do you establish reliable vertical dimension
BY DR.ABDULRAHMAN ALMUALM
MJDF MCQS WITH ANSWERS
1592.. If amalgam gets contaminated with moisture, the most uncommon result is**
A. Blister formation
B. Post operative pain
C. Secondary caries
D. Lower compressive strength
A. Loss of contacts
B. Slight tilting
C. Pocket formation
D. TMJ problem
E. All of the above
A. Leukoplakia
B. Erythema migrans /Geographic tongue/
BY DR.ABDULRAHMAN ALMUALM
MJDF MCQS WITH ANSWERS
A. Lingual Nerve
B. Long buccal nerve
1597. Patient complains of itching and vesicles on the upper labium (Vermillion region)
every year, your diagnosis would be
A. Herpes simplex
B. Recurrent ulceration aphthae
C. Impetigo
A. Amphotencin
B. Tetracycline lozenges
C. Mycostatin
A. Cotton wool
BY DR.ABDULRAHMAN ALMUALM
MJDF MCQS WITH ANSWERS
B. Ground glass
C. Orange peel
D. Beaten copped
1601 The most serious complications which may occur from abscess of max canine
is
A. Cellulitis
B. Cavernous sinus thrombosis
C. Lacrimal duct stenosis
D. Damage to infra orbital nerves
A. Radiographs
B. Electric pulp test
C. Biopsy
D. Thermal
A. Pericoronitis
B. Cysts
C. Abscesses
BY DR.ABDULRAHMAN ALMUALM
MJDF MCQS WITH ANSWERS
A. Penicillin
B. Tetracycline
C. Streptomycin
1608. A patient with long standing rheumatoid arthritis and a history of steroid therapy,
until a week ago, he presents for multiple extractions. The dentist should consult the
patient’s physician because
1609. A patient whose hands fell warm and moist is MOST likely to be suffering from**
BY DR.ABDULRAHMAN ALMUALM
MJDF MCQS WITH ANSWERS
A. Anxiety
B. Congestive cardiac failure
C. Thyrotoxicosis
1611. A 12 year old girl complains of sore mouth, she has painful cervical
lymphadenitis and a temperature of 39°c, oral examination shows numerous yellow
grey lesions. What is the MOST LIKELY diagnosis
A. Measles
B. Erythema multiform
C. Herpetic gingivostomatitis
D. Stevens-Johnson syndrome
1613. To reduce the side effects risk of local anaesthetic injections; you should follow
all of the following EXCEPT
BY DR.ABDULRAHMAN ALMUALM
MJDF MCQS WITH ANSWERS
1614. The most potent viricidal properties: “another format of the same answer:
Indicate which of the following has viricidal properties”
A. Sodium hypochlorite
B. Chlorhexidine
C. Glutaraldehyde
D. Alcohol 70%
E. Quaternary ammonium
1615. Antibiotics should be used routinely to prevent infection arising from oral surgery
in patients suffering from all the following EXCEPT
A. Agranulocytosis
B. Sever uncontrolled diabetes
C. Aplastic anaemia
D. Mumps
E. Leukaemia
A. 12 times a minute
B. 24 times a minute
C. 50 times a minute
D. 80 times a minute
1617. Nitrous Oxide (N2O) is not used alone as a general anaesthetic agent because
of**
BY DR.ABDULRAHMAN ALMUALM
MJDF MCQS WITH ANSWERS
A. X-Ray
B. Periodontal probe / Calibrated probe/
C. Periodontal marker
D. Bitewing radiograph
E. Sharp explorer
F. Study cast
1619. The final material you use for endodontically treated deciduous molars is**
A. Amalgam
B. GIC
C. Composite resin
D. Wrought base metal crown
A. Mast cells
B. Polymorphonuclear leukocytes
C. Eosinophils
D. Epithelial cells
BY DR.ABDULRAHMAN ALMUALM
MJDF MCQS WITH ANSWERS
A. Actinomycosis
B. Candidosis
C. Viral infection
D. Keratocyte
A. Avoid Osteomyelitis
B. Control pain
A. Sjogren’s syndrome
B. Emotional reaction
C. Antidepressants drugs
D. Submandibular sialolith
A. Mucocele
B. Mumps
BY DR.ABDULRAHMAN ALMUALM
MJDF MCQS WITH ANSWERS
C. Sjogren’s syndrome
A. Heart disease
B. Liver damage
C. Diabetes
1628. Which is the most important local factor in the aetiology of periodontal disease
A. Occlusal trauma
B. Calculus
C. Brushing habits
D. Coarse food
1629. Which of the following does state BEST the morphology of periodontal ligament
fibres
A. Elastic
B. Striated
C. Non striated
D. Levity
E. Wavy
1630. Which of the following is LEAST to cause toxicity from local anaesthetic injection
BY DR.ABDULRAHMAN ALMUALM
MJDF MCQS WITH ANSWERS
1631. If a child’s teeth do not form; this would MOSTLY affects the growth of**
A. Alveolar bone
B. Whole face
C. Mandible
D. Maxilla
1633. After the age of 6 years, the greatest increase in the size of the mandible occurs
A. At the symphysis
B. Between canines
C. Distal to the first molar
A. Open bite
B. Retrusion of maxillary central incisors
C. Reduced Overjet
BY DR.ABDULRAHMAN ALMUALM
MJDF MCQS WITH ANSWERS
D. Increased overbite
A. 2.2ml
B. 22ml
A. Ice cream
B. Canned juice
C. Cough syrups
D. Breakfast cereal
E. Sweet potato
1638. The amount of fluoride required to reduce caries according to age and level of
fluoride in drinking water. Which of the following figures is incorrect**
A. 1 year old child requires no fluoride when the fluoride in drinking water is 0.3PPM
B. 3 years old child requires no fluoride when the fluoride in drinking water is 0.7PPM
C. 6 years old child requires 1mg of fluoride when drinking water containing 0.5mg
BY DR.ABDULRAHMAN ALMUALM
MJDF MCQS WITH ANSWERS
1639. The major etiological factor responsible for Class II division 2 malocclusion in
Angel’s classification is**
A. Thumb sucking
B. Growth discrepancy
C. Tongue thrust habit
D. Tooth to jaw size discrepancy
E. Skeletal cause (discrepancy)
1640. Ankylotic primary second molar in the mandible is not always a good space
maintainer because of
1641. Preschool child has an intruded upper incisor; what would your treatment be**
A. X-ray
B. Put it back in place and splint
C. Control bleeding and check after a month
D. Make the patient comfortable without disturbing the tooth.
1642. An upper deciduous molar has a caries exposure and on X ray the
corresponding 2nd permanent premolar is absent. What treatment would you do to
the deciduous tooth
A. Pulpotomy
B. Endodontic treatment
C. Pulp capping
BY DR.ABDULRAHMAN ALMUALM
MJDF MCQS WITH ANSWERS
1643. Where is the MOST probable place of bone resorption after a deciduous molar
has a pulpal gangrene
A. Interradicular septum
B. The periapical area
1644. How many pulp horns are presented in a typical mandibular deciduous second
molar
A. 2
B. 3
C. 4
D. 5
A. Crevicular epithelium
B. Palatal epithelium
C. Alveolar mucosa
D. Free gingiva
E. Attached gingiva
A. Calculus
B. Plaque
C. Caries
D. Restorative material
BY DR.ABDULRAHMAN ALMUALM
MJDF MCQS WITH ANSWERS
1648. The advantage of using dental floss over rubber point interdentally
1649. After prophylactic treatment, you decide to change the flora to a non-acidogenic
by changing the diet. How long does it take to achieve this change
A. Few weeks
B. Several months or longer
A. Mannitol
B. Saccharin
C. Xylitol
1651. 6 year old child who had a history of primary herpes simplex has got a recurrent
infection. What is the likely cause
A. Herpes labialis
1652. A newly placed restoration interferes with occlusion. What will be the periodontal
response
BY DR.ABDULRAHMAN ALMUALM
MJDF MCQS WITH ANSWERS
1653. In class II restoration, all of the following considered to occur as probable causes
of periodontal problems except
A. Flat ridge
B. Faulty or not proper contour
C. Not properly polished restoration
D. Cervical wall is too deeply apical
E. Overextension of lining in cavity
1654. Angular type of bone resorption can be seen more often in**
A. Occlusal traumatism
B. Food particles retention
C. Periodontosis
D. All of the above
1655. What is the most important function of periodontal ligament
A. Narrow
B. Wide
1657. Which radiographic method would you use in assessing periodontal conditions
and lesions
A. Bitewing
B. Periapical
BY DR.ABDULRAHMAN ALMUALM
MJDF MCQS WITH ANSWERS
C. Occlusal
D. Panoramic
A. Acquired pellicle
B. Interlocking to the crystals of the tooth
C. Penetrated into enamel and dentine
D. Mechanical interlocking
E. All of the above
BY DR.ABDULRAHMAN ALMUALM
MJDF MCQS WITH ANSWERS
A. 0.25 to 0.5mm
B. 1mm
A. Interdental papillae
B. The free gingival ridge
C. The attached gingiva
D. The marginal gingiva
1665. Which is the MOST local factor in the aetiology of periodontal disease
A. Occlusal trauma
B. Calculus
C. Brushing habits
D. Coarse food
1666. Incisive foramen when are superimposed over apex of root on radiograph may
be mistaken to be
A. Cyst
B. Cementoma
C. Odontoma
1667. Which of the following factors can affect the shape and size of the pulp canal
BY DR.ABDULRAHMAN ALMUALM
MJDF MCQS WITH ANSWERS
1669. What is the MOST important role of saliva in preventing dental caries
A. Buffering action
1670. A patient comes with a lactobacillus of more than 100000. what is your advice
A. Sucrose
BY DR.ABDULRAHMAN ALMUALM
MJDF MCQS WITH ANSWERS
A. N2O has high analgesic property and low anastatic at its minimum anaesthetic
dose. “Low MAC; Max Anaesthetic Concentration”
B. Absolutely contraindicated in pregnancy
C. Has low blood diffusibility and result in hypoxia
D. It is good aesthetic and low MAC
1676. Which local anaesthetic agent is preferred for a confirmed hypersensitive patient
A. Intravascular injection
B. Hypersensitivity
BY DR.ABDULRAHMAN ALMUALM
MJDF MCQS WITH ANSWERS
1679. What to do with instruments after surgically treating a patient with confirmed
diagnosis of hepatitis B**
1682. The first thing to do after surgical removal of impacted 3rd molar in the mandible
is**
1683. A primary molar with relatively un-resorbed roots encompassing the permanent
tooth bud. What extraction technique would you use to avoid the inadvertent removal
of a developing bicuspid
BY DR.ABDULRAHMAN ALMUALM
MJDF MCQS WITH ANSWERS
1684. A young female patient presents with throbbing pain in the left lower posterior
jaw with trismus and associated lymphadenopathy. What would be your diagnosis
A. Tumour
B. Pericoronitis
1685. Patient presents to you with a history of local pain in the lower right posterior
region. Insisting that you extract his lower teeth. The teeth in question are vital without
any pathology. You diagnosis is
A. Odontalgia
B. Referred pain
C. Trigeminal neuralgia
1686. Which of the following are not supplied by the mandibular division of trigeminal**
1687. 30 years old male complains of painless swelling in the buccal mucosa. It has
been present for about six months. He admits “playing with it”. He is concerned
because this might represent cancer. The base is narrow; the most likely diagnosis is
A. Irritation fibroma
1688.A patient is complaining of an open sore on the buccal mucosa. The lesion is
painless, ulcerated, has indurated margins, 1.5 cm in diameter, covered by greyish-
white exudate, enlarged lymph nodes and tender, negative tuberculin test and positive
serology. The diagnosis is**
BY DR.ABDULRAHMAN ALMUALM
MJDF MCQS WITH ANSWERS
1689. An old male presents complaining of having numerous white lesions in the oral
cavity within past few days. Prior to this the family physician prescribed chlorite
tetracycline for an upper
respiratory infection, the patient is taking this antibiotic for the past two weeks; lesions
are relatively non-painful, slightly
elevated, adhere plaques on the lip mucosa, buccal mucosa and the tongue. MOST
LIKELY to be
1691. How can differentiate between a benign epithelial tumour from a carcinomatous
one
A. Soft papillomatous mass, not indurated or not fixed /Move freely/ and pedunculated.
BY DR.ABDULRAHMAN ALMUALM
MJDF MCQS WITH ANSWERS
A. Blood metastasis
B. Does not erode bone
C. Intensive involvement / inveterately characteristic/
D. Radio resistant
1695. Where does the bone resorption show in a necrosis pulp of deciduous molar
1697. 8 years old child, on examination you find 75 with carious exposure. On X-ray
you find 35 missing. Your treatment is
BY DR.ABDULRAHMAN ALMUALM
MJDF MCQS WITH ANSWERS
D. Extraction of 65 and 75
A. Class I
B. Class II
C. Class III
D. Class IV
1700. Pin Restoration with which material has the best retention
A. Amalgam
B. Gold inlay
C. Composite
D. Glass Ionomer
BY DR.ABDULRAHMAN ALMUALM
MJDF MCQS WITH ANSWERS
A. Clicking
B. Locking
C. Pain in the muscles of mastication
A. Trigeminal nerve
B. Glossopharyngeal
C. Facial nerve
D. Recurrent laryngeal
1704. Which impression material should NOT be kept in water within on hour: “in
another paper was: 30 mins before pouring”
A. Polyether
B. Condensation silicone
C. Polyvinyl silicone
1705. High copper amalgam lasts longer than low copper amalgam because of**
BY DR.ABDULRAHMAN ALMUALM
MJDF MCQS WITH ANSWERS
A. Heat
B. Chemical sterilisation
C. Flame
D. Boiling
E. Autoclave
A. ZOE
BY DR.ABDULRAHMAN ALMUALM
MJDF MCQS WITH ANSWERS
B. Zinc Polycarboxylate
C. GIC
A. Polycarboxylate
B. Zinc phosphate
C. Silicate phosphate
D. GIC
1716. A patient with reasonable oral hygiene has a small proximal caries on the
premolar. The patient requests an aesthetic filling. Your preparation will be
BY DR.ABDULRAHMAN ALMUALM
MJDF MCQS WITH ANSWERS
1719. The use of nickel chromium in base plate should be judiciously considered
because
1720. A seven year old boy fell of his bicycle 2 weeks ago and broke his maxillary
central incisor. The pulp horn is visible as a pin point. The tooth is vital. Your treatment
will be
A. Pulpectomy
B. Place calcium hydroxide and fill with composite resin
C. Calcium hydroxide pulpotomy
BY DR.ABDULRAHMAN ALMUALM
MJDF MCQS WITH ANSWERS
1721. During mouth preparation for RPD on tooth adjacent to edentulous area. There
is dentine exposure
A. Restoration is required
B. Proceed with rest seat preparation and fabrication if involved area is not more than
2mm
1722. After making an impression to reline an RPD the dentist notes that the indirect
retainers are not resting on the tooth. To avoid this what process should have
undertaken initially
1723. Which is the only dental tissue that lose its formative cells as it matures
A. Enamel
B. Dentine
C. Pulp
D. Cementum
1724.The muscle responsible for maintaining the bolus of food while chewing is
A. Buccinator
B. Orbicularis oris
A. Mylohyoid
BY DR.ABDULRAHMAN ALMUALM
MJDF MCQS WITH ANSWERS
B. Genioglossus
C. Digastric
1726. Which of the following is the most significant factor regarding filler in composite
for increased strength
1727.A patient has a small incisal fracture of the maxillary incisor. Which is the best
material to resist fracture at the acid etched tooth composite interface
A. Micro-filled composite
B. Hybrid composite
C. GIC
D. Silicate
A. Periods of stress
B. Occlusal pre-maturities during mandibular closure
A. I, II and III
B. None of the above
BY DR.ABDULRAHMAN ALMUALM
MJDF MCQS WITH ANSWERS
C. I and II
D. II and III
1731. While doing RCT you gave dressing with a paper point wetted with..??...solution.
The patient arrives the next day with severe
pain. There is no swelling but the tooth is tender to percussion. You will**
1732. The area of the posterior palatal seal includes which of the following
A. [left][right]
B. Hamular notch
A. Alcohol
B. Citric acid
C. Water
D. Organic acid
BY DR.ABDULRAHMAN ALMUALM
MJDF MCQS WITH ANSWERS
A. Serum
B. Vesicle
C. Vesicle fluid and saliva
A. Pulpal radiology
B. History and vitality test
C. X-ray and history
A. Cirrhosis of liver
B. Hypertension
1739. Children born with cleft palate, microdontia and glossoptosis have
A. Christian disease
BY DR.ABDULRAHMAN ALMUALM
MJDF MCQS WITH ANSWERS
B. Trenches-Collins Syndrome
C. Pierre-Robin Syndrome
1740. Which of the following penicillin are readily destructed by stomach acid
A. Methicillin
B. Cloxacillin
C. Phenoxy methyl
D. Penicillin G
A. Urine
B. Lungs
A. Multiple myeloma
B. Paget’s disease
C. Hyperparathyroidism
D. Chronic renal failure
BY DR.ABDULRAHMAN ALMUALM
MJDF MCQS WITH ANSWERS
Anaerobic Bacteria/
A. It effects chemotaxis
B. Produces leukous toxins
C. Destroys collagen
D. It is immuno-suppressive
A. MA
B. Exposure time
C. Developing time
D. Rinsing time
BY DR.ABDULRAHMAN ALMUALM
MJDF MCQS WITH ANSWERS
1750. A patient has developed a sever chest pain and difficulties in breathing while in
the dental chair. Your initial response is
1751. On inspection of lateral boarder of the tongue at the base, which structure would
you expect to find
A. Filiform papillae
B. Fungiform papillae
C. Taste buds
D. Lymph nodes
E. Circumvallate papillae
1752. Delayed eruptions of at least part of the dentition is a recognised feature in**
BY DR.ABDULRAHMAN ALMUALM
MJDF MCQS WITH ANSWERS
A. Dentino-Genesis imperfecta
B. Anhidrotic ectodermal dysplasia
C. Rickets
1754. A 10 year old boy presents with non-vital, non-mobile tooth. Treatment is
A. Pulpectomy with calcium hydroxide
B. Pulpectomy with Zinc oxide eugenol
C. Pulpotomy with formocresol
D. No treatment is required if tooth is asymptomatic
1755.A patient suffers a blow to his maxillary central incisor without resulting in
fracture. The pulp
A. Immediate necrosis
B. Becomes non-vital but only if treatment is delayed too long
C. Becomes non vital irrespective of treatment
D. No changes is seen later if fracture does not occur
1756. In the case f malignant melanoma occurring intra orally, which of the following
is true
BY DR.ABDULRAHMAN ALMUALM
MJDF MCQS WITH ANSWERS
1759. Patient presents with caries in many teeth. you will advise that
A. Fluoride toothpaste does not effectively prevent caries and topical fluoride is
required.
BY DR.ABDULRAHMAN ALMUALM
MJDF MCQS WITH ANSWERS
A. Angioneurotic oedema
B. Nervousness
1763. A physician refers a nine year old boy to you to confirm diagnosis. The boy has
a fever of 40°C and coughing. When you focus your light into his eyes he turns away.
Intra-orally there are white spots surrounded by red margins. The disease and lesions
are
1764. In periodontal scalers and curettes; the blade is formed by which two surfaces
1765. Which of the following is NOT TRUE in regards to lateral periodontal cyst**
BY DR.ABDULRAHMAN ALMUALM
MJDF MCQS WITH ANSWERS
1766. Middles aged woman gives a history of intermittent unilateral pain in the sub
mandibular region, most probable cause is
A. Oculomotor
B. Ansa cervicalis
C. Abducens
1769. After an inferior alveolar nerve block; the patient develops paralysis of eyelid,
upper lip and lower lip on that side. This means that the L.A was deposited in
BY DR.ABDULRAHMAN ALMUALM
MJDF MCQS WITH ANSWERS
1772. The pulpal floor of the Class II cavity for a mandibular first premolar should be**
I. Insufficient condensation
II. First proximal increment was too large
III. Neglecting to wedge the matrix
IV. Hand manipulation instead of mechanical
V. Debris contamination
A. I, II, III
B. II, III, IV
C. I, II, V
D. None of the above
E. All of the above
1774. What are the dangers of using air as a cooler during cavity cutting
A. Hypersensitivity
B. Odontoblast is drawn into the tubule
BY DR.ABDULRAHMAN ALMUALM
MJDF MCQS WITH ANSWERS
1777. Which of the following is MOST useful in differentiating between apical abscess
and periodontal
A. Percussion
B. Vitality tests
C. Cold tests
D. Heat tests
1778. What is the ideal length for a post in post-core in an endodontically treated tooth
BY DR.ABDULRAHMAN ALMUALM
MJDF MCQS WITH ANSWERS
1780. How many mg of fluoride ions are obtained from 2.2 mg tablet of NaF
A. 0.5mg
B. 1 mg
C. 1.5mg
D. 10mg
A. An external force
B. An internal force to oppose external load
C. Deformity opposed the applied load
A. Age
B. Parafunctional
C. History of the tooth /abrasion, erosion, caries/
D. All of the above
1783. Self polymerising acrylic resins differs from heat cured resins because they
exhibit
BY DR.ABDULRAHMAN ALMUALM
MJDF MCQS WITH ANSWERS
1786. Where would you expect to find the Mylohyoid on relation to periphery of
complete denture
A. In lingual pits
B. In buccal fissures
C. Poor oral hygiene
BY DR.ABDULRAHMAN ALMUALM
MJDF MCQS WITH ANSWERS
1791. Porcelain must not be contaminated by handling between which two stages
1792. What is the relationship of the retentive portion of the partial dentures retainers
to the survey line of abutment
A. Gingival /Below/
B. Occlusal /Above/
C. No relation
1793. Which of the following liquids is not suitable for prolonged immersion of cobalt
chrome partial dentures
A. Alkaline peroxidase
BY DR.ABDULRAHMAN ALMUALM
MJDF MCQS WITH ANSWERS
B. Hypochlorite solutions
C. Soap solutions
D. Water
A. The line running from the tragus of the nose to ala of the ear
B. A guide used to orient the occlusal plane
C. Parallel to Frankfurt horizontal plane
D. A guide to the occluding face height in complete denture.
BY DR.ABDULRAHMAN ALMUALM
MJDF MCQS WITH ANSWERS
A. Reverse Overjet
B. TMJ problems
C. Cranio mandibular skeletal relationship
1801. What is important requisite for fillers in dental composite restorative resins in
load bearing area
1802. Sensitivity to hot and cold foods soon after cavity preparation and placement of
GIC and composite resin in an upper incisor tooth is due to
BY DR.ABDULRAHMAN ALMUALM
MJDF MCQS WITH ANSWERS
1803. After completing pulp extirpation, debridement and placing a dressing; apical
periodontitis is because
1806. When you tries to seat a crown on tooth you find a discrepancy of 0.3mm at the
margin; you will
BY DR.ABDULRAHMAN ALMUALM
MJDF MCQS WITH ANSWERS
A. Is anionic
B. Used in 0.02% concentration
C. Used in 0.12 concentration
D. Penetrates the gingival crevice/pocket
1809. Which of the following is NOT complication of radiation to head and neck area**
A. Xerostomia
B. Mucositis
C. Increased caries
D. Heightened taste sensation
E. Increased risk of osteomyelitis
1810. A female patient is diagnosed with Addison’s disease which of the following
does not confirm this
A. Weakness, lassitude
B. Anorexia, nauseas, fatigue
C. Hypotension
D. Bony expansion
E. Amenorrhea
1811. Which of the following conditions is not associated with periodontal destruction
in primary teeth
BY DR.ABDULRAHMAN ALMUALM
MJDF MCQS WITH ANSWERS
A. Down’s syndrome
B. Steven Johnson’s syndrome
C. Hypophosphatasia
D. Papillon-Lefebvre syndrome
E. Cyclic neutropenia
1813. Which of the following is the best index to evaluate gingival health
1815. Patient is resistant to caries but has periodontal disease. In this case, sucrose
in diet is important because
BY DR.ABDULRAHMAN ALMUALM
MJDF MCQS WITH ANSWERS
A. Glucans
B. Levans
C. Fructans
D. Sucrose
A. Induce vomiting
B. Gives a lot of fluids
C. Gives a lot of fluids and sodium bicarbonates
D. Ask patient not to eat for 45 minutes
E. Gives milk, calcium tablets or magnesium tablets
A. Reduces the size of the beam, so it is easy to visualise the central X ray.
B. Avoids unnecessary exposure to radiation of surrounding tissues of the patient
1820. In calculus formatio, the epitaxic concept is one of the theories. Which of the
following is true
BY DR.ABDULRAHMAN ALMUALM
MJDF MCQS WITH ANSWERS
A. External resorption
B. Internal resorption
C. Necrosis of the pulp
D. Ankylosis
1825. When the developing solution is correctly mixed and x ray film is being
developed for normal time; but the solution is too warm, the outcome film will be
BY DR.ABDULRAHMAN ALMUALM
MJDF MCQS WITH ANSWERS
A. Too light
B. Too dark
C. Fogged
A. S. mutans only
B. No micro organism
C. S. mutans and S. salivavis
D. Lactobacilli and S. mutans
1829. The papillae that are few in numbers, associated with MOST taste buds,
associated with Von Ebner’s glands are**
A. Fungiform
B. Circumvallate
C. Foliate
D. Filiform
BY DR.ABDULRAHMAN ALMUALM
MJDF MCQS WITH ANSWERS
1830. In class II preparation it is difficult to place the gingival seat when preparation is
extended too gingivally because the
A. It is normal
B. Typically associated with tachycardia
C. Associated with blood pressure
D. Usually subsides spontaneously
E. Typically associated with constricted and un-reactive pupil
A. Denaturation of protein
BY DR.ABDULRAHMAN ALMUALM
MJDF MCQS WITH ANSWERS
1835. The maxillary and mandibular teeth get their blood supply from**
1839. A 65 year old patient needs extraction of 44; he has taken insulin in the morning.
What preoperative advice you should
BY DR.ABDULRAHMAN ALMUALM
MJDF MCQS WITH ANSWERS
give
BY DR.ABDULRAHMAN ALMUALM
MJDF MCQS WITH ANSWERS
1843. A patient 37 year old; with paroxysmal pain on the left eye that he thinks is
related to his maxillary posterior teeth. The pain comes in recurrent bursts and
aggravated by stress and alcohol. Oral exam is negative. The probable diagnosis is
A. Migraine
B. Cluster headache
C. Trigeminal neuralgia
D. Temporal neuritis
1844. A mandibular permanent first molar had to be extracted, this will affect
A. Adjacent teeth
B. Teeth in the same quadrant
C. Both arches the same side
D. Full mouth
1845. The places for new erupted mandibular molars are created by
1846. A patient comes with a firm, painless swelling of lower lobe of parotid which has
grown progressively for the past year. He complains of paresthesia for the past 2
weeks. This is most likely to be
A. Pleomorphic adenoma
B. Carcinoma of the parotid
C. Lymphoma of parotid
1847. What is the histopathology of the pathogenesis of the plaque following 21 days
of plaque accumulate
BY DR.ABDULRAHMAN ALMUALM
MJDF MCQS WITH ANSWERS
A. Hereditary
B. Drug induced
C. Plaque induced
D. Leukaemia
1851. A 13 year old has enlarged gingivae; gives a history of Dilantin sodium what is
you treatment
BY DR.ABDULRAHMAN ALMUALM
MJDF MCQS WITH ANSWERS
C. Stop medication
1852. A patient has improperly formed DEJ, reduction in size of pulp chamber,
chipping and attrition of enamel that would MOSTLY be
A. Fluorosis
B. Amelogenesis imperfecta
C. Dentinogenesis imperfecta
A. Hypodontia or anodontia
1855. During extraction of maxillary third molar, the tuberosity is fractured; however, it
remains in place attached to the mucoperiosteum. Which of the following procedures
should be employed**
BY DR.ABDULRAHMAN ALMUALM
MJDF MCQS WITH ANSWERS
A. Inform the patient and her physician of your findings and instruct the patient to return
in six months
B. Surgically excise the entire lesion since you know it is not malignant
C. Dismiss the patient with instructions for warm saline rinses for re- examination
D. Repeat the biopsy
A. Fontanelles
BY DR.ABDULRAHMAN ALMUALM
MJDF MCQS WITH ANSWERS
A. Viral infection
B. Diabetes
C. Measles
D. Rubella
E. Candidosis
A. Neurofibroma
B. Necrosis of bone produced by ionizing radiation
A. Antibiotic therapy
B. Poor surgical techniques
C. Aspirin
D. Codeine
BY DR.ABDULRAHMAN ALMUALM
MJDF MCQS WITH ANSWERS
A. Leucopoenia
B. Neutropenia
C. Leukocytosis
D. Lymphocytosis
E. Eosinophilia
A. Herpes simplex
B. Whooping cough
C. Bacterial endocarditis
A. Diabetes mellitus
B. Addison’s disease
C. Multiple myeloma
D. Squamous cell carcinoma
E. Bright’s disease
F. Cushing’s disease
1871. Patient has fainted, the signs are, blanched face, weak pulse, moist skin,
shallow respiration; your first management is
A. 1 ml adrenaline subcutaneously
B. Mouth to mouth respiration
C. Nitro glycerine sub lingually
BY DR.ABDULRAHMAN ALMUALM
MJDF MCQS WITH ANSWERS
A. Oedema
B. Haemorrhage
C. Acute infection
1873. Patient who has WBC count of just over 100000 is most likely suffering from**
A. Leucopoenia
B. Leukaemia
C. Polycythemia
1875. In regards to the conditions where you have to prescribe antibiotic prior to dental
treatment
A. Rheumatic fever
B. Sub-acute bacterial endocarditis
C. By pass
D. Valve replacement
E. Uncontrolled diabetes
BY DR.ABDULRAHMAN ALMUALM
MJDF MCQS WITH ANSWERS
A. Coxsackie virus
A. Vitamin K
A. Excitement
B. Shock
C. Improper eye sight
D. Leaning
E. Sever headache
F. Vomiting
G. Euphonia
H. Fixed dilated pupils
1880. A young patient presented with rheumatic fever and suspected allergy to
penicillin. The antibiotic of choice is
BY DR.ABDULRAHMAN ALMUALM
MJDF MCQS WITH ANSWERS
A. Chloromycetin
B. Sulphonamide
C. Buffered penicillin
D. Erythromycin
E. Achromycin
A. Adrenal suppression
A. Adrenal suppression
B. Delayed healing
C. Osteoporosis
D. All of the above
1883. Esophagitis, herpes simplex, colitis during 5 weeks. You will find the same signs
of
A. Multiple myeloma
B. Erythema multiforme
C. AIDS
BY DR.ABDULRAHMAN ALMUALM
MJDF MCQS WITH ANSWERS
1886. Steam under pressure sterilisation is the best method to kill microorganisms.
How does it work
1887. Patient with morphine coma; what is the medication of choice to reverse its act
A. Bradykinin
B. Epinephrine
C. Amphetamine
D. Naloxone
1889. When comparing the mesio distal length of second deciduous molar with the
length of 2nd premolar; we will find the deciduous tooth is
A. Longer
B. Shorter
BY DR.ABDULRAHMAN ALMUALM
MJDF MCQS WITH ANSWERS
A. History
1891. How do you treat a child with severe Von Willebrand’s disease**
1894. Painless bluish lump filled with fluid on the lips; MOST likely is
A. Smoker’s keratosis
B. Squamous cell carcinoma
C. Mucocele
D. Fibroma
BY DR.ABDULRAHMAN ALMUALM
MJDF MCQS WITH ANSWERS
E. Fibro-epithelial polyp
A. Tzanck cells
B. Test dose of corticosteroid
C. Test of anti body
D. Histological immunofluorescence
E. Serological test for auto antibody
A. Mosaic pattern
A. Edentulous ridge
B. Short lingual frenum
C. Short labial frenum
A. Elevation
B. Fixation
C. Invasion
D. Verrucoid appearance
E. Pain
BY DR.ABDULRAHMAN ALMUALM
MJDF MCQS WITH ANSWERS
1899. Blow to the mandible resulted in deviation to the left on opening; x-rays show
unilateral fracture, where would you expect the fracture**
A. Osteoporosis
B. Osteopetrosis
A. Adrenalin 1mp IV
BY DR.ABDULRAHMAN ALMUALM
MJDF MCQS WITH ANSWERS
BY DR.ABDULRAHMAN ALMUALM
MJDF MCQS WITH ANSWERS
1909. If the focal spot to film distance is increased from 20cm to 40cm, the intensity of
radiation is reduced by
A. ½
B. ¼
C. 1/3
D. 1/5
A. Vitamin C
A. Preservation of pulp
B. Immobilisation
C. Root canal treatment
D. Calcium hydroxide treatment
1913. In full dentures; porosity in the most thickest area is due to**
A. Gaseous porosity
B. Shrinkage porosity
BY DR.ABDULRAHMAN ALMUALM
MJDF MCQS WITH ANSWERS
1914. The most common cause of fracture at the isthmus of class II dental amalgam
restoration is
A. Delayed expansion
B. Inadequate depth at the isthmus area
C. Inadequate width at the isthmus area
D. Moisture contamination of the amalgam during placement
A. Barbiturate
1918. For a 5 years old child who lives in a NON WATER FLUORIDATED are. What
is the recommended intake of fluoride
A. 0.25mg
B. 0.10mg
C. 0.50mg
D. 1.00mg
BY DR.ABDULRAHMAN ALMUALM
MJDF MCQS WITH ANSWERS
A. Heart disease
B. Asthma
C. Mental retardant
D. Sickle cell anaemia
A. Chromogenic bacteria
A. Labial
B. Palatal
A. Chronic lesion
A. Pleomorphic oedema
A. A kind of clinical study in which neither the participants nor the person administering
treatment know which treatment any particular subject is receiving. Usually the
comparison is between an experimental drug and a placebo or standard comparison
treatment. This method is believed to achieve the most accuracy because neither the
doctor nor the patient can affect the observed results with their psychological bias.
1925. After 4 to 7 days, what type of cells you would find predominately in gingivitis
BY DR.ABDULRAHMAN ALMUALM
MJDF MCQS WITH ANSWERS
A. Leukocytes
B. Plasma cells
1926. What the age of patient who has all incisors, some premolars and some canine
erupted; note that no 2nd molars showing
A. 8 years
B. 11 years
C. 13 years
D. 14 years
A. +2
B. -2
C. +8
D. -8
BY DR.ABDULRAHMAN ALMUALM
MJDF MCQS WITH ANSWERS
A. Aerobes
B. Aerobes to mainly anaerobes
A. A test that measures the rate at which red blood cells settle through a column of
liquid. A non-specific index of inflammation
A. Scarlet fever
B. Pericarditis
C. Pancreatitis
D. Carbuncle
BY DR.ABDULRAHMAN ALMUALM
MJDF MCQS WITH ANSWERS
1937. Which of the following has proven to be the MOST important in community
preventive program
A. 0.25mg
B. 0.50mg
C. 1.00mg
D. 0mg
A. Candidal infection
A. Electrons
1942. In the mouth of new born baby; what sort of bacteria you expect to find
A. None
BY DR.ABDULRAHMAN ALMUALM
MJDF MCQS WITH ANSWERS
A. Transcription
1945. To obtain the MOST accurate X rays of teeth; the tooth film distance should be
(Close/far) as anatomical restriction will permit. What is TRUE in this regard
A. Hormonal disturbance
A. Defective neutrophils
BY DR.ABDULRAHMAN ALMUALM
MJDF MCQS WITH ANSWERS
A. Hepatotoxic reaction
A. Diazepam
1952. Which lymph node is involved in carcinoma of the lip: “or the first metastasis of
carcinoma of lips”
A. Submental node
B. Submandibular node
1953. Which of the following could cause the overall cellular damage to be greater
1954. Which of the following conditions would be considered for antibiotic prophylaxes
1955. All of the following should be considered for systemic antibiotic except
BY DR.ABDULRAHMAN ALMUALM
MJDF MCQS WITH ANSWERS
1956. The tissue response to oral hygiene after periodontal treatment is BEST
assessed by
A. Face bow
B. Kinematic face bow
C. Articulator
1961. Incisal colour differs from gingival colour in that the gingival part
BY DR.ABDULRAHMAN ALMUALM
MJDF MCQS WITH ANSWERS
1963. The silver bromide crystals in x rays films after being expressed to radiation
forms
A. Latent image
A. PA skull x ray
B. Occipitomental radiograph
C. Town’s view
A. No mottling
B. Mottling in almost all permanent teeth except some molars
C. Mottling in permanent premolars only
1967. Which of the following is not considered in the estimation of gingival index
A. Nasmyth’s membrane
BY DR.ABDULRAHMAN ALMUALM
MJDF MCQS WITH ANSWERS
1968. When examining intra orally between the side of the tongue and the lateral
border of the mandible, you expect to
A. HIV patient
1971. When there is a fracture on condyle, the muscle responsible for elevation of
condyle is
A. Initial stage
B. Proliferation stage
C. Histodifferentiation stage
D. Morphology stage
BY DR.ABDULRAHMAN ALMUALM
MJDF MCQS WITH ANSWERS
A. There is contraction
B. There is negligible expansion
C. There is definite expansion
D. There is no change
E. None of the above
BY DR.ABDULRAHMAN ALMUALM
MJDF MCQS WITH ANSWERS
1977. The flow of the following percentage is allowable for impression compound (type
I) at the oral temp of 37º
A. 6%
B. 10%
C. 2%
D. 20%
E. None of the above
1979. Generally there is ???? zinc oxide eugenol impression pastes between flow are
A. Working time
B. Accelerator
C. Setting time
D. Composition
E. None of the above
BY DR.ABDULRAHMAN ALMUALM
MJDF MCQS WITH ANSWERS
A. Hydrophilic
B. Hydrophobic
C. Water-loving impression material
D. Potassium alginates
E. None of the above
1984. The effect of temperature rise above 100ºC on heat cured denture base acrylic
resins is
BY DR.ABDULRAHMAN ALMUALM
MJDF MCQS WITH ANSWERS
A. The stronger the restoration which contains fewer matrixes alloys and fewer voids
B. The weaker the restoration is
C. The more matrixes alloys
D. The more voids
E. None of the above
BY DR.ABDULRAHMAN ALMUALM
MJDF MCQS WITH ANSWERS
A. It may vary widely vary widely and has no influence on the final restoration
B. The degassing procedure is not important
C. It is the Achilles heel of direct gold restoration
D. Clinical tech are more important than the physical properties of restorative gold
E. All of the above
BY DR.ABDULRAHMAN ALMUALM
MJDF MCQS WITH ANSWERS
1992. The effectiveness of the acid etch is dependent on which of the following factors
A. Material must be used to clean the surface of the tooth prior to etching
B. The effectiveness of the itchant
C. The chemical and physical nature of the tooth
D. The area and surface of the enamel to be itched
A. Hypertension
BY DR.ABDULRAHMAN ALMUALM
MJDF MCQS WITH ANSWERS
B. Angina pectoris
C. Myocardial infection
1997.A patient with history of angina suffers an attack while in the dental chair. Prompt
relief can be anticipated in MOST instances from
A. Von Recklinghausen
B. Paget’s
C. Periapical granuloma
2000.You notice radiolucent area close to the apex of central incisor, on a second x
ray the radiolucent area moves; it is likely to be
A. Cyst
B. Abscess
C. Granuloma
BY DR.ABDULRAHMAN ALMUALM
MJDF MCQS WITH ANSWERS
D. Incisive foramen
2001Treatment of Osteoradionecrosis is
A. Antibiotic coverage
B. Conservative treatment including antibiotic coverage and resection of jaw segment.
C. Conservative treatment with sequestrectomy
A. Osteosarcoma
B. Scleroderma
A. Cortical bone
B. Spongy bone
C. Immature bone
D. Cribriform plate perforated by nutrient carnally
A. Occlusally
B. Lingually
C. Buccally
D. Gingivally
E. Interproximally
2005. A periapical x ray of 11 and 12 region shows the vimen, floor of the nasal fossa
and the median palatine suture. The other feature that can be seen is
BY DR.ABDULRAHMAN ALMUALM
MJDF MCQS WITH ANSWERS
A. Maxillary sinus
B. Incisive foramen
C. Zygomatic process
D. Wall of maxillary sinus
BY DR.ABDULRAHMAN ALMUALM
MJDF MCQS WITH ANSWERS
A. Nickel chrome
2014. Initiation of curing process in self cure acrylic resin is achieved by**
A. Benzyl peroxide
BY DR.ABDULRAHMAN ALMUALM
MJDF MCQS WITH ANSWERS
A. Where performing an endodontic treatment on existing root canal filling may lead
to fracture of the root
B. When root canal treatment is faulty
C. When there is danger of involving other structures
BY DR.ABDULRAHMAN ALMUALM
MJDF MCQS WITH ANSWERS
D. When the bony defect is so extensive that the edges of the incisors will collapse
E. None of the above
2023. To achieve optimum cavity preparation which of the following factors of internal
anatomy must be considered
A. Outline form
BY DR.ABDULRAHMAN ALMUALM
MJDF MCQS WITH ANSWERS
B. The age and shape of pulp chamber; in addition to the direction of individual root
canals.
C. Internal external relationship
D. Intra-coronal preparation
E. None of the above
BY DR.ABDULRAHMAN ALMUALM
MJDF MCQS WITH ANSWERS
2027. Which of the following varieties should be made in the proximal occlusal cavity
preparation in deciduous teeth compared to permanent ones
2028 10 years old boy looses permanent mandibular molar; what is affected
2030. During teeth eruption, the reduced enamel epithelium merges with the oral
epithelium and consequently….????
BY DR.ABDULRAHMAN ALMUALM
MJDF MCQS WITH ANSWERS
A. Down growth of oral epithelium which replaces the reduced enamel epithelium
B. Proliferation of inner enamel epithelium
C. Proliferation of outer enamel epithelium
D. Down growth of oral epithelium which undermines the reduced enamel epithelium
E. Gradual transformation of the reduced enamel epithelium
A. Classifications
B. X rays
C. Plaster models
2032. As a general practitioner; you decide at an initial appointment that you can not
handle a child due to lack of co- operation. Which of the following approaches would
seem to be your alternative
2033. The most common reason to refer a child to a pedodontist is problem with
A. Rampant caries
B. Behaviour management
C. Endodontic treatments in primary teeth
D. Space maintainers
A. Gingival inflammation
B. Vasodilation
BY DR.ABDULRAHMAN ALMUALM
MJDF MCQS WITH ANSWERS
C. Wider spaces
D. Necrosis of bone
2035. A patient who has lost several teeth in an otherwise healthy mouth, can get
A. TMJ dysfunction
B. Changes in the vertical dimension
C. Change in the interocclusal dimension
A. Depth
B. Width
C. Length
2039. 8 years child has a badly broken deciduous molar what is the best material to
restore it
A. Amalgam
B. Gold
BY DR.ABDULRAHMAN ALMUALM
MJDF MCQS WITH ANSWERS
C. Composite
D. GIC
A. Mesio version
B. Disto version
C. Lingo version
D. Torsion
A. 60%
BY DR.ABDULRAHMAN ALMUALM
MJDF MCQS WITH ANSWERS
A. Expand
BY DR.ABDULRAHMAN ALMUALM
MJDF MCQS WITH ANSWERS
B. Contract
C. Contract and expand
D. Expand and contract
A. Orbicularis oris
A. Impact
B. Fatigue
A. Substitute for the missing wall so adequate condensation forces can be applied
B. Permit re-establishment of proper contact lines
C. Restrict extrusion of amalgam and prevent formation of an “overhang”
BY DR.ABDULRAHMAN ALMUALM
MJDF MCQS WITH ANSWERS
A. Hybrid composite
A. Cohesion
B. Atmospheric pressure
C. Retention in the defect
D. Patient support it with the tongue
2060. In a fixed moveable bridge where should the moveable connectors “non rigid”
be placed
BY DR.ABDULRAHMAN ALMUALM
MJDF MCQS WITH ANSWERS
2063. What is the minimal labial reduction for porcelain metal crowns
A. 1mm
B. 1.5mm
C. 0.5mm
A. To protect alloy from oxidation, and distribute metallic oxides as they are formed
A. In compression
B. In tension
BY DR.ABDULRAHMAN ALMUALM
MJDF MCQS WITH ANSWERS
2068. A 29 year old lady presents with mandibular second molar associated with
radiolucency of 1 cm diameter and paraesthesia of mental nerve. There is no other
symptoms
A. A slow progression
B. Rapid progression
C. Cyclic or burst progression “active and inactive phases”
D. Intermittent progress
A. Cemental tears
B. Bone loss
C. Mobility
D. True pocket formation
BY DR.ABDULRAHMAN ALMUALM
MJDF MCQS WITH ANSWERS
A. Bleeding
B. True pocket formation and apical migration of attached gingiva
A. Down’s syndrome
B. Cleidocranial dysplasia “dysostosis”
2075. Which of the following does not carry a risk of infection from hepatitis B patient
A. HBs Ag antigens
B. HBs Ag
C. HBe Ag
BY DR.ABDULRAHMAN ALMUALM
MJDF MCQS WITH ANSWERS
2078. A patient 8 years old has 3 of first premolars erupted with swelling on the ridge
of the un-erupted premolar. X ray shows a fully developed crown and ¾ roots
development with no other pathology. What is your management
2079. Loss of the first deciduous molar in 10 years old child required
2080. Palatal root displaced into the antrum while extracting; what is your decision to
retrieve it
A. Odontogenic keratocyte
BY DR.ABDULRAHMAN ALMUALM
MJDF MCQS WITH ANSWERS
B. Central haemangioma
C. Radicular cyst
D. Osteomyelitis
2082. The MOST frequent retained deciduous teeth in permanent dentition are
A. Liven
B. Fructose
C. Glucan
D. Glycogen
2084. Benzodiazepine and diazepam in 5-10mg oral dose used for oral sedation in
dentistry DOES NOT give
A. Necrotic pulp
B. Carious exposure
BY DR.ABDULRAHMAN ALMUALM
MJDF MCQS WITH ANSWERS
C. Mechanical exposure
D. Periapical disease
2086. After you have successfully treated an Angle’s Class II division I malocclusion.
The ideal Class I incisor relationship has been produced and 14, 24 were extracted.
The arches are now well aligned. What molar occlusion will there be at the end of
treatment when all spaces are closed
A. Immunofluorescence
B. Microscopy
C. Serology
2089. Metallic Plato backing the intra oral films are for**
BY DR.ABDULRAHMAN ALMUALM
MJDF MCQS WITH ANSWERS
A. 5 minutes at 20ºC
B. At least 10 minutes
C. Until it clears up
D. 2 minutes at 40ºC
A. 5 minutes at 20ºC
B. At least 10 minutes
C. Until it clears up
D. 2 minutes at 40ºC
A. 100-120 nm
B. 200-300 nm
C. 400-430 nm
D. 470 nm or 450-500 nm
BY DR.ABDULRAHMAN ALMUALM
MJDF MCQS WITH ANSWERS
A. Sulphur oxides
B. Oxygen
C. Chlorides
D. Over trituration
A. The intensity of the blow was too low to cause pulp death
2098. Patient presents with fever of 39ºC, pain, swelling of upper lip and nose.
Radiograph shows an enlargement of periodontal ligament space of 11 which has a
large restoration without a base. What would your treatment be
BY DR.ABDULRAHMAN ALMUALM
MJDF MCQS WITH ANSWERS
2099. Cervical finish line of full veneer crown preparation should be placed
2102 Which of the following will NOT be used in determination of vertical dimension
A. Aesthetic
BY DR.ABDULRAHMAN ALMUALM
MJDF MCQS WITH ANSWERS
B. Phonetics
C. Gothic arch tracing
D. Swallowing
2107. A lateral incisor labial to the arch needs to be restored in normal alignment with
PFM retraction. How will the tooth appear
A. Too wide
B. Too short
BY DR.ABDULRAHMAN ALMUALM
MJDF MCQS WITH ANSWERS
C. To narrow
D. To long
A. Length
B. Cross section
C. Material
D. Degree of taper
E. All of the above
2110. In vital pulp therapy; what is the optimum depth for a pin hole in a tooth
A. 4-5mm
B. Approximately 2mm
C. Less than 2mm
D. 1-1.5mm
A. Acute angle
B. Right angle
BY DR.ABDULRAHMAN ALMUALM
MJDF MCQS WITH ANSWERS
C. Obtuse angle
D. 45 angle
2113. An irregular shaped void on surface of a gold cast would indicate that
A. Soluble in chloroform
B. Too weak for narrow canals
A. Reduces new caries and hamper the progress of freshly established caries
B. Reduces new caries and hamper the progress of existing caries
BY DR.ABDULRAHMAN ALMUALM
MJDF MCQS WITH ANSWERS
2117. What control tooth or teeth should be used when testing a suspected pulpally
involved tooth
2119. Cast crown fits on die but not on tooth; discrepancy is about 0.3mm what would
you do
A. The wider the gap the better the chance of secondary caries
B. Seal the margin with fissure sealant would prevent further breakdown
C. Secondary caries may develop
BY DR.ABDULRAHMAN ALMUALM
MJDF MCQS WITH ANSWERS
A. Coronoid process
B. Condyle
C. Masseter muscle
A. Brightness
B. Saturation of hue
C. Value
2124. Acrylic self-cure special trays; how long should have been made prior taking
impression
A. 12 hrs
B. Immediately after fabricating it
C. After been left in water for an hour
D. Wait for an hour before pouring
A. Placement of dentures
B. Plaque accumulation
2126. Movable component of the non-rigid connector in a fixed bridge is placed. Which
of the following is TRUE
BY DR.ABDULRAHMAN ALMUALM
MJDF MCQS WITH ANSWERS
2127. When lateral incisor is lost; patient has Class II Division II type with deep bite.
Which of the following is contra indicated
A. Formed of, or having, two laminae, or thin plates. Which is the distal attachment of
superior hard lateral plate
2130. Which of the following DOES NOT cause depression of the mandible
BY DR.ABDULRAHMAN ALMUALM
MJDF MCQS WITH ANSWERS
2132. How much would you reduce a cusp to be replaced with amalgam onlay
A. 2 mm to achieve a good retention form
B. 2mm to achieve a good resistance form
C. 1mm
2133. How long it would take to notice significant reduction in radiolucency after
finishing a root filing for a tooth with a periapical lesion
A. 6 months
B. 1 month
C. 3 months
A. Class II Division I
B. Tongue thrust
3135. When treating a tooth with a non-vital pulp with a fistula presented; fistula should
be treated by
A. Surgical incision
B. Antibiotic coverage
C. The usual root canal procedures for non-vital teeth and no special procedures for
fistula
BY DR.ABDULRAHMAN ALMUALM
MJDF MCQS WITH ANSWERS
2137. 27 years old female; shows sudden oedematous rash and collapses after an
injection of barbiturates. Your management is
2139. 50 years old man presented after a full mouth extraction complaining that he
“bled all night”. Which of the following pre existing conditions could be responsible for
the post operative bleeding
A. Mitosis in osteoblast
B. Mitosis of osteoblast
C. Appositional growth in cartilage epiphysis
D. Interstitial growth in cartilage epiphysis
BY DR.ABDULRAHMAN ALMUALM
MJDF MCQS WITH ANSWERS
2142. Bone graft method that has shown the greatest osteogenetic potential is
2143. A patient states that for ALMOST a year now, she has had a rubbery, firm,
painless nodule within the substance of parotid gland. This MOST likely is
A. Mucocele
B. Lymph node
C. Benign mixed tumour
D. Squamous cell carcinoma
E. Sialolith with encapsulations
BY DR.ABDULRAHMAN ALMUALM
MJDF MCQS WITH ANSWERS
2147. Child with less than normal number of teeth, mandibular lateral incisor is larger
than usual; on x rays it shows with two roots and two roots canals; your diagnosis is**
A. Dilaceration
B. Gemination
C. Fusion
D. Concrescence
E. Taurodontism
A. Nasal floor
B. Cranial vault
C. Occlusal plane
D. Naso maxillary complex
E. Anterior cranial base
2149. 7 years child with Class I malocclusion, slight version of maxillary Class I;
adequate arch length. What is your management
A. Oral screen
B. Head cap therapy
BY DR.ABDULRAHMAN ALMUALM
MJDF MCQS WITH ANSWERS
A. Incompetent lips
A. Mobility
B. Dull pain on closing
C. Presence of true pocket
D. Apical migration of gingival epithelium
E. Presence of subgingival calculus
F. C, D & E
A. Double flap
B. Stripping procedure
C. Full thickness flap
D. Apically positioned flap
E. Split thickness flap
BY DR.ABDULRAHMAN ALMUALM
MJDF MCQS WITH ANSWERS
2155. Examination reveals area of gingival recession, exposed wide area of dental
roots. Which is the procedure of choice to obtain coverage of the root surface
A. Distal fluting
B. Long attached gingiva
2158. Two conditions of enamel facilitate post eruptive uptake of fluoride element
BY DR.ABDULRAHMAN ALMUALM
MJDF MCQS WITH ANSWERS
A. Length
B. Cross section
C. Material
D. Degree of taper
E. Under cut area
Ans e
A. Infectious mononuclears
2163. The FIRST advantage of using 100000 rpm and over rotors is
2164. How long it would take to see the dentinal bridge after direct pulp capping by
using Calcium hydroxide**
BY DR.ABDULRAHMAN ALMUALM
MJDF MCQS WITH ANSWERS
A. 6-8 weeks
B. 4 weeks
C. 6-8 months
D. 4 months
2165. The OPTIMUM crown to root ratio for abutment tooth is**
A. 2:3
B. 1:1
2168. Patient has been coming to your clinic for several times complaining about
soreness under the denture; what would you do
2169. What is the difference between arcon and non arcon articulator
2170. Purplish lesions on the buccal mucosa that have been there since birth; the
diagnosis is
BY DR.ABDULRAHMAN ALMUALM
MJDF MCQS WITH ANSWERS
A. Haemangioma
2172. What is contraindicated to the use of calcium hydroxide for pulp capping
2174. Patient comes to you complaining of pain in a tooth, the tooth is filled with
composite long time ago; what would you do
BY DR.ABDULRAHMAN ALMUALM
MJDF MCQS WITH ANSWERS
A. Acromegaly
B. Paget’s disease
C. Giant cell lesion
D. Primordial cysts
E. Dental cysts
2177. For fissure and sealant treatment to be a part of the primarily retentive care
A. 4 mm indicates periodontitis
2180. Which of the following elements is not found in normal periodontal membrane
A. Fibroblast
B. Epithelial cells
C. Erythrocytes
D. Vest cells of malaise
E. Inflammatory plasma cells and lymphocytes
2181. Which of the following situations make periodontal disease more sever
BY DR.ABDULRAHMAN ALMUALM
MJDF MCQS WITH ANSWERS
2182. The auxiliary occlusal rest on tooth for partial denture should be placed
2183 A vital tooth has a crown cemented to a pin retained amalgam cored; where does
failure occur
A. Angioneurotic oedema
B. Hypotension
C. Respiratory depression
D. Hypertension
Ans D
2185. Swallowing will aid in the diagnosis of
A. Branchial cyst
BY DR.ABDULRAHMAN ALMUALM
MJDF MCQS WITH ANSWERS
2187. On X ray you found the Gutta Percha cone extending 1mm beyond the apex
without any symptoms; what would you do
A. Remove restoration material until you are able to withdraw the Gutta Percha cone
B. Apiectomy
C. Leave as is until any complications occur
2188. On X ray you found the cement of previous root canal treatment is extending
1mm beyond the apex without any symptoms; what would you do
2189. What is the main purpose of using corticosteroids in pulpal obturation material
BY DR.ABDULRAHMAN ALMUALM
MJDF MCQS WITH ANSWERS
2192. The main of damaged gingival tissues after placing rubber dam is
A. Frictional resistance
2195. How much under cut area a clasp arm should engage**
BY DR.ABDULRAHMAN ALMUALM
MJDF MCQS WITH ANSWERS
2199. You have patient with Class II division 2; which of the following is contraindicated
A. Cantilever bridge
B. Maryland bridge
2200. How will cover buccal bicuspid for lower premolar when making a metallic
porcelain crown
BY DR.ABDULRAHMAN ALMUALM
MJDF MCQS WITH ANSWERS
A. Ductile
B. Hard
C. High sensitivity
Ans A
2203. What sort of material do you use for the fabrication of Maryland bridges
2204. When the neck of the condyle is fractured; what muscles determine the
movement of the superior segment
A. Lateral pterygoid
B. Medial pterygoid
C. Temporalis
D. Mylohyoid
Ans A
2205. Patient with prosthetic heart valves, with INR value of 3.0; requires surgery,
what is the your management
BY DR.ABDULRAHMAN ALMUALM
MJDF MCQS WITH ANSWERS
2206. Chronic oral antral fistula for some time after the extraction of maxillary first
molar. What is your management
A. Surgical closure
B. Anti-biotic and nasal decongestant
C. Wash the antrum
Ans B
A. Always
B. Never
C. 10 to 15%
Ans C
2208. The MOST common sites for squamous carcinoma in the oral cavity are**
2209. A patient has painful lesions on her buccal mucosa. Biopsy report shows
acantholysis and supra basilare; your diagnosis is
BY DR.ABDULRAHMAN ALMUALM
MJDF MCQS WITH ANSWERS
A. Pemphigus vulgaris
B. Bulla lichen planus
C. Erythema multiform
D. Systemic lupus erythematosus
Ans A
A. Milliampere
B. Time
C. KvP Kilovoltage
D. Object-film distance
E. Focal spot-object distance
Ans E
BY DR.ABDULRAHMAN ALMUALM
MJDF MCQS WITH ANSWERS
A. Metastases is common
B. Erodes bone
C. More common in oriental races
D. Cannot occur in oral mucosa according to definition (ans is D
BY DR.ABDULRAHMAN ALMUALM
MJDF MCQS WITH ANSWERS
A. 6
B. 5.5
C. 4
D. 4.5
2220. When it is acceptable for patient to hold radiographic film packet in the patient’s
mouth
2222. Posterior vital molar with core the best material to restore it is
BY DR.ABDULRAHMAN ALMUALM
MJDF MCQS WITH ANSWERS
A. Amalgam
2226. When restoring with composite resins, why do we do the cavo surface beveling
A. Aesthetic
A. Cervical third
BY DR.ABDULRAHMAN ALMUALM
MJDF MCQS WITH ANSWERS
2230. When preparing Class II cavity, you notice a hard dark brown spot on the
adjacent tooth just below the contact point; MOST LIKELY it is
A. Demineralised enamel
A. Necrosis
A. One year
BY DR.ABDULRAHMAN ALMUALM
MJDF MCQS WITH ANSWERS
2237. Which muscle acts on the disto lingual contour of lower denture
A. Mentalis
B. Masseter
C. Mylohyoid
D. Buccinator
2238. The MOST common curvature of palatal root of maxillary first molar is
A. Distal
B. Mesial
C. Buccal
D. Palatal
2240. 58 years old male has had a 60 yo WM course of radiation given for carcinoma
of tongue. Patient complains of pain associated with poor dentition. The dental
management would be
A. Immediate extraction of any poor teeth under local anaesthetic with antibiotic
coverage
B. Segmental dental clearance and closure to eliminate problems
C. No dental treatment may be due to neuronic of neoplasms
BY DR.ABDULRAHMAN ALMUALM
MJDF MCQS WITH ANSWERS
2242. Occasional sensitivity in a shallow class I amalgam restoration after two days
would be managed by
A. Pulpalgia
B. Internal resorption
C. Hypercalcification within root canals
D. All of the above
BY DR.ABDULRAHMAN ALMUALM
MJDF MCQS WITH ANSWERS
2245. When should not contaminate metallic framework during fabrication of porcelain
fused to metal crown
2246. “Pop off” of a porcelain veneer from under the lying gold crown is due to**
A. Building bone around the fundus of alveolar bone and deposition of cementum
B. Increased interocclusal distance
C. Formation of dentine
2249. Initial condylar guidance of 25 degree was wrong is changed to 45 degree. What
changes will you make to achieve balanced occlusion
BY DR.ABDULRAHMAN ALMUALM
MJDF MCQS WITH ANSWERS
2250. Good oral hygiene and fluoridation is LEAST useful in preventing caries of
A. Regeneration of cementum
B. Long junctional epithelium
A. Mobility
BY DR.ABDULRAHMAN ALMUALM
MJDF MCQS WITH ANSWERS
A. Diabetes
B. Viral infection
A. 2-5 hours
B. 5-12 hours
C. 12-30 hours
D. 30-48 hours
E. 48-96 hours
2259. Ulcers, necrosis and plasma cells at the basal membrane with atrophic thin
areas, reduced rete pegs will be diagnosed as
BY DR.ABDULRAHMAN ALMUALM
MJDF MCQS WITH ANSWERS
A. Desquamative gingivitis
A. Tzanck cells
B. Intraepithelial vesicles
C. Histopathology like aphthous ulcer
D. Scarring of the conjunctiva
2261. In syphilis
A. The spirochetes disseminate rapidly throughout the body within 24hour after
contact
B. Both the primary chancre and the secondary mucous patch stages of the disease
are highly infectious
C. Only the lesions of the primary and secondary stages are contagious
D. All of the above
BY DR.ABDULRAHMAN ALMUALM
MJDF MCQS WITH ANSWERS
A. Thyroiditis
B. Pancreatitis
C. Osteomyelitis
D. Scarlatina
E. Pneumonia
2268. Patient with weak pulse, moist skin and dyspnoea; what is the first thing to do
BY DR.ABDULRAHMAN ALMUALM
MJDF MCQS WITH ANSWERS
2269. Single retroclined upper incisor in 9 years old, space is sufficient. What is your
management
2271. In which of the following conditions vesicles/bullae are never seen prior to
ulceration
A. HSV 1
B. Aphthous ulcer
C. Pemphigus
2272. Patient complains of finger-like growth on the lateral aspect of the tongue. The
lesion is painless and of normal colour. The MOST PROBABLE diagnosis is
BY DR.ABDULRAHMAN ALMUALM
MJDF MCQS WITH ANSWERS
A. Folate papillae
B. Filiform papillae
C. Neurofibroma
D. Papilloma
A. Collagen synthesis
B. Clotting factor production
C. Epithelial integrity
D. Cellular energy production
2275. Child presented to you with sore throat, fever and joint swelling; the MOST
probable diagnosis is
A. Rheumatic fever
B. Rheumatic arthritis
C. Osteoarthritis
A. Urea
B. Uric acid
C. Allantoin
BY DR.ABDULRAHMAN ALMUALM
MJDF MCQS WITH ANSWERS
2277. Patient shows a lesion on the tongue adjacent to sharp tooth. You “rounded off”
the sharp area and recall patient after one month to see the lesion turning smaller in
size. What your next step would be
A. Keep observing
B. Perform an excision biopsy
C. Prescribe Kenalog and Orabase
C. Osteosarcoma
D. External lymphoma
E. Kaposi sarcoma
A. Prescribe antibiotics
B. Debridement and antimicrobial rinses
C. Gingivoplasty
D. Flap surgery
A. Clotting time
B. Bleeding time
C. Prothrombin time
D. Sedimentation rate
E. Complete blood cell count
BY DR.ABDULRAHMAN ALMUALM
MJDF MCQS WITH ANSWERS
A. Garr’s osteomyelitis
B. Condensing osteitis
C. Torus
2282. You want to place a post on an endodontically treated tooth which has a good
silver point “Ag point”; there is no evidence of failure of the previous root filing. What
would you do
A. Remove and replace the Ag point with Gutta Percha before the post preparation.
2283. The placement of metal stops at a location remote to direct retainers to increase
retention is termed
A. Indirect retainers
2284. The hamular notch is important in full dentures construction because it aids in
the setting position of the artificial teeth
2285. When setting up teeth for complete dentures having bilateral balanced
occlusion, separation of posterior teeth during protrusion is done by
2286. When patient bites in protrusion you notice that posterior teeth do not meet,
what would you do to solve this
BY DR.ABDULRAHMAN ALMUALM
MJDF MCQS WITH ANSWERS
A. A systemic allergy
B. Allergy because of denture
C. Psychogenic
2291. What is the MOST COMMON configuration of the mesial buccal canal of upper
first molar**
BY DR.ABDULRAHMAN ALMUALM
MJDF MCQS WITH ANSWERS
2293. Why would you invest the wax pattern as soon as possible in an indirect inlay
fabrication
A. Minimise distortion
B. Avoid contraction
C. Avoid expansion
2294. Upon palpation which of the following areas would be found to have overlying
mucosa
A. I and II
B. I, II, III
C. I, II, V
D. None of the above
E. All of the above.
Ans C
2296 Patient needs a minimal preparation metal crown for his LL6 the best cement
is
a) GIC
b) zinc phosphates
c) RMGIC
d) zinc polycarboxylate cement
BY DR.ABDULRAHMAN ALMUALM
MJDF MCQS WITH ANSWERS
e) resin cement
Ans e
2297.What is the best type of restoration for restoring buccal root caries in the lower
mandibular molar in elderly patient:
a) Glass ionomer
b) Amalgam
c) Composite
d) Porcelain veneer crown
e) Zinc phosphate cement
ans A
2300.The liquid which is present in the adhesive dentin systems to wet the dentin is:
a) Water
b) Acetone
c) Polymers liquid
d) Ketones
Ans B
2301.During the try-in stage of complete Denture you need to do slight grinding of
back teeth.
Which teeth cusps should be reduced
a) Reduce teeth randomly
b) Upper buccal and lower lingual
c) Upper palatal and lower buccal
d) Upper palatal and lower lingual
e) Upper buccal and lower buccal
ans B
2302.the obturation technique which gives the best hermetic seal for gutta-percha in
endodontic treatment
a) warm lateral compaction
b) warm vertical compaction
c) thermoplasticised gutta-percha technique
d) cold lateral compaction
BY DR.ABDULRAHMAN ALMUALM
MJDF MCQS WITH ANSWERS
e) chlorpercha technique
Ans C
2303.The use of latex gloves does has the following effect when a polyvinyl siloxane
impression is taken
a) Retard the set of the impression material
b) Enhance the set of the impression material
c) Result in porosities in the impression material
d) Has no effect on impression
e) Latex gloves stick to the polyvinyl siloxane impression material
Ans A
2305.Female patient with missing upper lateral incisors, gaps between centrals and
canines
and pointed canine the best treatment option would be:
a) Move the canines by orthodontic means to close gaps
b) Provide fixed-fixed bridge
c) Provide single crowns over the canines to close the gaps to a limit
d) Close the gaps with tooth colored restorations
e) Restorative and orthodontic intervention
f) flumazenil
Ans e
BY DR.ABDULRAHMAN ALMUALM
MJDF MCQS WITH ANSWERS
Ans A
2308.First line treatment for patient with NCTL in palatal surfaces of upper front
teeth:
a) Direct Composite palatal veneers
b) Full coverage crowns
c) Gold veneers
d) Indirect composite palatal veneers
e) Indirect porcelain veneers
Ans A
2311.Placement of maxillary anterior teeth in complete denture too far superiorly and
anteriorly might result in difficulty in pronouncing :
a) F and V sounds
b) S sound
c) T and S sounds
d) D and T sounds
e) Th and V sounds
Ans A
2312.You are going to do full clearance and immediate denture fabrication, the teeth
that
should be removed last are:
a) Canines and 1st molars
b) Canines and second molars
BY DR.ABDULRAHMAN ALMUALM
MJDF MCQS WITH ANSWERS
BY DR.ABDULRAHMAN ALMUALM
MJDF MCQS WITH ANSWERS
d) Post width
e) Post design
ans A
2321.A patient lost his two lower central incisors in a fight; what do you recommend
a) Fixed fixed Resin bonded bridge
b) Two separate cantilever bridge
c) Fixed fixed conventional
d) Spring cantilever
e) 6 units bridge from canine to canine |ans is C?
2322.A 90-year old gentleman presented to clinic who is edentulous and has
dentures
upper and lower full for 15-years. Denture bit uncomfortable, tooth structures little bit
worn out, freeway space 2-4mm, polished surfaces satisfactory and occlusal wear
minimal. How would you proceed?
a) Copy denture
b) Hard reline
c) Soft reline
d) Construction of new denture
e) Construction of new denture with thermo-press material
Ans A
2323.When preparing the Apical Zone, the use of the files sequentially from apex to
backwards. what is the best distance to achieve good apical seal.
a) 0.5
b) 1
BY DR.ABDULRAHMAN ALMUALM
MJDF MCQS WITH ANSWERS
c) 1.5
d) 2
e) Flush apex
Ans B
2324.Copy denture is indicated for a patient with:
a) Oral Cancer
b) TMJ problems
c) Artificial limbs
d) Hemophilic patient
e) Heavy smoking habit
Ans B
2325.The best way to achieve retention in patient with mandibular free end saddle is:
a) Occlusal rest mesially
b) RPI system, gingival approaching
c) Occlusal rest distally
d) Wide lingual coverage
Ans B
2326.The best way to achieve retention in patient with maxillary free end saddle is:
a) Occlusal rest mesially
b) Occlusal rest distally
c) gingival approaching clasp
d) Attachments
e) RPI
E?
2328.A 18 years old girl lost her upper canine in accident, she has good oral hygiene
and minimal restored
dentition; to restore the tooth we use:
a) Implant
b) Resin based fixed fixed bridge
c) Resin based cantilever bridge
d) Fixed fixed conventional bridge
e) Conventional cantilever bridge
BY DR.ABDULRAHMAN ALMUALM
MJDF MCQS WITH ANSWERS
e) Beechwood creosote
2331. 50 years old patient who started smoking since he was 20, he smoked 20
cigarettes for
15 years before he reduced the number to only 10 cigarettes per day. His pack/year
value
is:
a) 10
b) 20
c) 22.5
d) 15
e) 25
Ans C
2333. The maxillary cast partial denture major connector design with the
greatest potential to cause speech problems is
A. a thick narrow major connector.
B. an anterior and a posterior bar
C. a thin broad palatal strap.
D. narrow horseshoe shaped.
Ans A
2334. Which of the following structures lies inferior to the mylohyoid muscle at
the level of the mandibular second molar?
A. Lingual artery.
BY DR.ABDULRAHMAN ALMUALM
MJDF MCQS WITH ANSWERS
B. Lingual vein.
C. Lingual nerve.
D. Submandibular duct.
Ans B
2339. Roots of the permanent maxillary central incisors are completed by what
age?
A. 8 years.
BY DR.ABDULRAHMAN ALMUALM
MJDF MCQS WITH ANSWERS
B. 10 years.
C. 12 years.
D. Later than 12 years.
Ans B
2344.
What clinical evidence would support a diagnosis of acute dento-alveolar
abscess? 1. 2. 3. 4.
1. A negative reaction to the electric vitality tester.
2.A positive reaction of short duration to cold.
3.A positive reaction to percussion.
4.Presence of a draining fistula.
A. (1) (2) (3)
B. (1) and (3)
C. (2) and (4)
D. (4) only
E. All of the above.
Ans B
BY DR.ABDULRAHMAN ALMUALM
MJDF MCQS WITH ANSWERS
2347. In teeth with complete pulp necrosis, the periapical area is involved if
1. there is pain to thermal stimuli.
2. there is pain on percussion.
3. the tooth throbs when the patient is lying down.
4. the radiograph shows an apical radiolucency.
A. (1) (2) (3)
B. (1) and (3)
C. (2) and (4)
D. (4) only
E. All of the above.
Ans C
2348. Radiographs of the mandibular incisor teeth of a 45 year old healthy black
female patient reveal periapical radiolucencies. The teeth are vital and
asymptomatic. You would
A. perform a biopsy of the radiolucent lesion.
B. perform endodontic therapy on the four incisors.
C. place a drain in the affected area.
D. observe periodically
Ans D
BY DR.ABDULRAHMAN ALMUALM
MJDF MCQS WITH ANSWERS
4. fibroblasts.
5. polymorphonuclear leucocytes.
A. (1) (2) (3) (4)
B. (1) (3) (4)
C. (1) (3) (4) (5)
D. (2) and (5)
E. All of the above.
Ans C
Ans C
2352. Which of the following is NOT a sign or symptom of the myofascial pain
dysfunction syndrome?
A. Pain.
B. Muscle tenderness.
C. Limitation of jaw motion.
D. "Clicking" or "popping" noise in the joints.
E. Radiographic changes of the join
Ans E
BY DR.ABDULRAHMAN ALMUALM
MJDF MCQS WITH ANSWERS
E. dentigerous cyst.
Ans B
2356. Tetracycline will cause crown discolouration when prescribed at the age
of
1. 6 months in utero.
2. 2 years.
3. 7 years.
4. 14 years.
A. (1) (2) (3)
B. (1) and (3)
C. (2) and (4)
D. (4) only
E. All of the above.
Ans A
BY DR.ABDULRAHMAN ALMUALM
MJDF MCQS WITH ANSWERS
Ans B
Ans B
BY DR.ABDULRAHMAN ALMUALM
MJDF MCQS WITH ANSWERS
BY DR.ABDULRAHMAN ALMUALM
MJDF MCQS WITH ANSWERS
BY DR.ABDULRAHMAN ALMUALM
MJDF MCQS WITH ANSWERS
BY DR.ABDULRAHMAN ALMUALM
MJDF MCQS WITH ANSWERS
BY DR.ABDULRAHMAN ALMUALM
MJDF MCQS WITH ANSWERS
2382. Which of the following combinations of milliamperage and kilovoltage will give
Xradiation with the maximum penetration?
A. 10kVp - 65ma.
B. 85kVp - 5ma.
BY DR.ABDULRAHMAN ALMUALM
MJDF MCQS WITH ANSWERS
C. 90kVp - 10ma. .
D. 65kVp - 15ma. .
E. 75kVp - 40ma.
Ans is C
True
False
Ans True
2386. The occlusal parameter that is most useful to differentiate between an overbite
of dental or skeletal origin is the
A. mandibular curve of Spee.
B. mandibular curve of Wilson.
C. molar sagittal relationship.
D. mandibular anterior lack of space.
E. maxillary curve of Wilson.(
ans is A)
.2388 What is the time for the lower permanent canine to erupt?
A- 9-11 years.
2389. You want to make root canal treatment in the first lower permanent molar, what
nerves you need to anaesthetise?
A- Inferior Dental.
BY DR.ABDULRAHMAN ALMUALM
MJDF MCQS WITH ANSWERS
A. Azithromycin
. B. Cephalexin.
C. Clindamycin.
D. Erythromycin.
Ans B
2392 Which of the following penicillins is most effective against Gram-negative organisms?
A. Nafcillin.
B. Ampicillin
C. . C. Methicillin.
D. D. Penicillin V.
E. E. Phenethicillin.
Ans B
2393 The most common cause of persistent post-operative sensitivity following the
placement of posterior composite resin restorations is
A. hyperocclusion.
B. microleakage
. C. acidic primers.
D. residual caries.
Ans B
2394 Space closure is LEAST likely to occur following the premature loss of the primary
A. maxillary second molar.
B. mandibular second molar.
C. maxillary first molar.
D. mandibular canine.
E. maxillary central incisor.
Ans e
2395 A 14 year old boy presents with bilateral pearly-white thickening of the buccal mucosa
which has been present since birth. His brother has similar lesions. The most likely diagnosis
is A. leukoplakia.
B. lichen planus.
C. mucous patches.
D. white sponge nevus.
Ans D
2396 The white appearance of the oral mucosa following extended local application of
acetylsalicylic acid is the result of
A. hyperparakeratosis.
B. acanthosis.
C. coagulation necrosis
. D. edema.
BY DR.ABDULRAHMAN ALMUALM
MJDF MCQS WITH ANSWERS
Ans C
.2398 Sharpey’s fibres run in 5 directions: crestally, apically, horizontally, obliquely and
interradicularly? A- True.
2399The lamina dura appears as a dark thin line on X- Ray? A-False, it appears as a white
line arround the teeth indicating low rate of turn over in the bone.
2401-The first formed cementum is the acellular one? A-True, and the cellular forms later.
2406Anaemia, ischemia, or poisoning CO in blood can cuses hypoxia and cell death?
A- True.
BY DR.ABDULRAHMAN ALMUALM
MJDF MCQS WITH ANSWERS
2408In the mandibulr first permanent molar, the buccal cusps are smaller and less pointed
that the lingual cusps?
True
Ans A
Pemiphigius is intraepthilial ,intercellular IgG and C3 ,heals without scaring ,skin lesion
involved
Ans C
2415. Pt with lower incisor radiolucency.. in x ray it is Acceptable percentage it will be?
A: not more than 50%
B: not more than 20%
C: not more than 10%
BY DR.ABDULRAHMAN ALMUALM
MJDF MCQS WITH ANSWERS
A. Analysis of data
B. Program planning
C. Program operation
D. Surveying
E. Evaluation
Ans is D
2417. 59. Patient come to you with the request of replacing her old partial denture with a
new one. Which denture design is best for taste and phonetics while eating?
a. Relieving the anterior area
b. Relieving the posterior area
c. Relieving the center area
Ans C.
2418. 72. The Department of Health publication “A First Class Service” can be defined as a
framework through which NHS organisations are accountable for continuously improving
the quality of their services and safeguarding high standards of care by creating an
environment in which excellence in clinical care will flourish.”. What does the statement
indicates?
a. Clinical Quality Assurance
b. Clinical Audit
c. Clinical governance
d. Clinical Operational Service
Ans c
BY DR.ABDULRAHMAN ALMUALM